Вы находитесь на странице: 1из 124

© 2014 Pearson Education, Inc., Upper Saddle River, NJ. All rights reserved.

This material is protected under all copyright laws as they currently


exist. No portion of this material may be reproduced, in any form or by any means, without permission in writing from the publisher.

7–1.  As the top plate is pulled to the right with a constant U


velocity U, the fluid between the plates has a linear velocity
distribution as shown. Determine the rate of rotation of a fluid
element and the shear-strain rate of the element located at y. u h
y

Solution U

We consider steady flow of an ideal fluid.


Referring to the velocity profile shown in Fig. a, u
h
u U U
= ;    u = y y
y h h

And
(a)
v = 0

. We or
m W ina g

b)
ed e n
The rate of rotation or average angular velocity of the fluid element is

in
no W iss ea s

itt id tio
is e D t w
t p or em ch
d th g. in t la
1 0v 0u 1 u U U

an on in rs h
vz = a - b = a0 - b = - = b Ans.

k g rn to rig
2 0x 0y 2 h 2h 2h
or in a uc y
w d le tr p

er ld
e lu nt ns co

The rate of shear strain is


th inc de f i es

# 0v 0u U
of rk ( stu e o tat

gxy = + = Ans.
ity o g us d S

0x 0y h
te is ss th ite
in f th se for Un
gr w in e
th t o a ly by
y ar d le d
ro p an o te
st ny s d s ec
de f a rse de ot

s
ill o u vi pr
w le co ro is
sa eir is p rk
th d wo
an his

e
T

Ans:
U
vz =
2h
#
gxy =
U
h

722
© 2014 Pearson Education, Inc., Upper Saddle River, NJ. All rights reserved. This material is protected under all copyright laws as they currently
exist. No portion of this material may be reproduced, in any form or by any means, without permission in writing from the publisher.

7–2.  A flow is defined by its velocity components y


u = 1 4x2 + 4y2 2 m>s and v = ( -8xy) m>s, where x and y
are in meters. Determine if the flow is irrotational. What is
C B
the circulation around the rectangular region?

0.4 m

x
O A
0.3 m

Solution
We consider ideal fluid flow.
0v 0

. We or
= ( -8xy) = -8y
0x 0x

m W ina g

b)
ed e n
in
no W iss ea s

itt id tio
is e D t w
0u 0

t p or em ch
( 4x2 + 4y2 ) = 8y

d th g. in t la
=
0y 0y

an on in rs h
k g rn to rig
1 0v 0u 1
vz = a - b = ( - 8y - 8y) = -8y
or in a uc y
w d le tr p

er ld
2 0x 0y 2
e lu nt ns co
th inc de f i es

Since vz ≠ 0, the flow is rotational. Along edge OA, y = 0. Then


of rk ( stu e o tat
ity o g us d S

u OA = 4x2 + 4 ( 02 ) = ( 4x2 ) m>s


te is ss th ite
in f th se for Un

Along edge AB, x = 0.3 m. Then


gr w in e
th t o a ly by

vAB = - 8(0.3)y = ( - 2.4y) m>s


y ar d le d
ro p an o te
st ny s d s ec

Along edge BC, y = 0.4 m. Then


de f a rse de ot

s
ill o u vi pr

u BC = 4x2 + 4 ( 0.42 ) = ( 4x2 + 0.64 ) m>s


w le co ro is
sa eir is p rk
th d wo

Along edge CO, x = 0. Then


an his

e
T

vCO = - 8(0)y = 0

Here, uOA and vAB are directed in the positive sense of dx and dy, respectively.

uOA # dx and vAB # dy, are positive. However, uBC # dx and vCO # dy
C C C C
Thus,

are negative since uBC and vCO are directed in the negative sense of dx and dy,
respectively.

V # ds = uOA # dx + vAB # dy - uBC # dx - vCO # dy


C C C C C
Γ =

0.3 m 0.4 m 0.3 m

L0 L0 L0
= 4x2dx + ( -2.4y)dy - ( 4x2 + 0.64 ) dx - 0

4 3 0.3 m 0.4 m
4 0.3 m
= x ` - 1.2y2 ` - a x3 + 0.64xb `
3 0 0 3 0

= -0.384 m2 >s Ans.

Ans:
- 0.384 m2 >s

723
© 2014 Pearson Education, Inc., Upper Saddle River, NJ. All rights reserved. This material is protected under all copyright laws as they currently
exist. No portion of this material may be reproduced, in any form or by any means, without permission in writing from the publisher.

7–3.  A uniform flow V is directed at an angle u to the


horizontal as shown. Determine the circulation around the 0.3 m
C B
rectangular region.
0.5 m
V
A
O

Solution
We consider ideal fluid flow. 30˚
The component of V along edges OA and BC is

u = V cos u

. We or
m W ina g

b)
ed e n
in
no W iss ea s
30˚

itt id tio
is e D t w
t p or em ch
and the component of U along edges AB and CO is

d th g. in t la
30˚

an on in rs h
v = V sin u
k g rn to rig
or in a uc y
w d le tr p

er ld
30˚
e lu nt ns co

Here, u and v are directed in the same sense as dsOA and d AB , respectively. Thus,
th inc de f i es

u # ds BC and v # ds CO are negative since u and v are directed in the opposite


C C
of rk ( stu e o tat
ity o g us d S

sense to that of ds BC and ds CO, respectively. Thus, the circulation can be determined as
te is ss th ite
in f th se for Un

V # ds = u # ds OA + v # ds AB - u # ds BC - v # ds CO
C C C C C
gr w in e

Γ =
th t o a ly by
y ar d le d
ro p an o te

0.3 m 0.5 m 0.3 m 0.5 m

L0 L0 L0 L0
st ny s d s ec

= V cos udsOA + V sin udsAB - V cos udsBC - V sin udsCO


de f a rse de ot

s
ill o u vi pr

= 0 Ans.
w le co ro is
sa eir is p rk
th d wo

Note: The irrotational flow always produces Γ = 0. In this case, this result is to be
an his

expected since the flow is irrotational.


T

Ans:
Γ = 0

724
© 2014 Pearson Education, Inc., Upper Saddle River, NJ. All rights reserved. This material is protected under all copyright laws as they currently
exist. No portion of this material may be reproduced, in any form or by any means, without permission in writing from the publisher.

*7–4.  The velocity within the eye of a tornado is defined


by vr = 0, vu = (0.2r) m>s, where r is in meters. Determine
the circulation at r = 60 m and at r = 80 m.

60 m
80 m

Solution
We consider ideal fluid flow.
Since vu is always tangent to the circle, v # ds = vu ds. For r = 60 m,
vu = 0.2(60) m>s = 12 m>s and ds = rdu = 60du.

. We or
2p 2p

m W ina g
V # ds =
C L0 L0

b)
ed e n
in
no W iss ea s
Γr = 60 m = 12(60du) = 720u  2p 2
0 = 1440p m >s Ans.

itt id tio
vuds =

is e D t w
t p or em ch
d th g. in t la
an on in rs h
For r = 80 m, vu = 0.2(80) m>s = 16 m>s, and ds = rdu = 80du.
k g rn to rig
or in a uc y
w d le tr p

er ld
2p 2p
e lu nt ns co

V # ds =
C L0 L0
Γr = 80 m = vuds = 16(80du) = 1280u  2p 2
0 = 2560p m >s Ans.
th inc de f i es
of rk ( stu e o tat
ity o g us d S
te is ss th ite
in f th se for Un
gr w in e
th t o a ly by
y ar d le d
ro p an o te
st ny s d s ec
de f a rse de ot

s
ill o u vi pr
w le co ro is
sa eir is p rk
th d wo
an his

e
T

725
© 2014 Pearson Education, Inc., Upper Saddle River, NJ. All rights reserved. This material is protected under all copyright laws as they currently
exist. No portion of this material may be reproduced, in any form or by any means, without permission in writing from the publisher.

7–5.  Consider the fluid element that has dimensions in polar D C


coordinates as shown and whose boundaries are defined by v  dv
the streamlines with velocities v and v + dv. Show that the  dr
vorticity for the flow is given by z = - 1 v>r + dv>dr 2 .
v
A B

r
u

Solution
We consider ideal fluid flow.
The circulation of the flow around element ABCD can be determined from

V # ds
C

. We or
Γ =

m W ina g

b)
ed e n
in
no W iss ea s

itt id tio
is e D t w
t p or em ch
d th g. in t la
= vSAB + (v + dv)( - SCD)

an on in rs h
k g rn to rig
= v(r∆u) + (v + dv) 3 - (r + dr)∆u 4
or in a uc y
w d le tr p

er ld
e lu nt ns co

= - vdr∆u - rdv∆u - dvdr∆u


th inc de f i es

Neglect the second order terms


of rk ( stu e o tat
ity o g us d S

Γ = - vdr∆u - rdv∆u = - ∆u(vdr + rdv)


te is ss th ite

The area of the element, again, neglecting higher-order terms, is


in f th se for Un
gr w in e
th t o a ly by

A = (r∆u)dr
y ar d le d
ro p an o te

Thus, the vorticity is


st ny s d s ec
de f a rse de ot

Γ - ∆u(vdr + rdv)
s
ill o u vi pr

z = =
A (r∆u)dr
w le co ro is
sa eir is p rk
th d wo

v dv
= -a + b (Q.E.D)
an his

r dr
e
T

726
© 2014 Pearson Education, Inc., Upper Saddle River, NJ. All rights reserved. This material is protected under all copyright laws as they currently
exist. No portion of this material may be reproduced, in any form or by any means, without permission in writing from the publisher.

7–6.  Determine the stream and potential functions for the y


two-dimensional flow field if V0 and u are known.

V0

x
Solution
We consider ideal fluid flow.
The velocity components are
u = V0 cos u0   v = V0 sin u0
0u 0v
Since the continuity equation + = 0 + 0 = 0 is satisfied, the establishment
0x 0y
of a stream function is possible using the velocity components,
0c 0c
= u;    = V0 cos u0

. We or
0y 0y

m W ina g

b)
ed e n
in
no W iss ea s

itt id tio
is e D t w
Integrating this equation with respect to y,

t p or em ch
d th g. in t la
c = (V0 cos u0)y + f(x) (1)

an on in rs h
k g rn to rig
Also, or in a uc y
w d le tr p

er ld
0c 0
e lu nt ns co

- = v;    - 3 ( V0 cos u0 ) y + f(x) 4 = V0 sin u0


0x 0x
th inc de f i es
of rk ( stu e o tat

0
3f(x) 4 = -V0 sin u0
ity o g us d S

0x
te is ss th ite

Integrating this equation with respect to x,


in f th se for Un
gr w in e

f(x) = ( - V0 sin u0)x + C


th t o a ly by
y ar d le d
ro p an o te

Setting C = 0 and substituting this result into Eq. (1),


st ny s d s ec

c = ( V0 cos u0 ) y - ( V0 sin u0 ) x
de f a rse de ot

s
ill o u vi pr

c = V0 3 ( cos u0 ) y - ( sin u0 ) x 4
w le co ro is

Ans.
sa eir is p rk
th d wo

1 0v 0u 1
Since, vz = a - b = (0 - 0) = 0, the flow is indeed irrotational. Thus, the
an his

2 0x 0y 2
T

potential function exists.


Using the velocity components,
0f 0f
= u;   = V0 cos u0
0x 0x
Integrating this equation with respect to x,
f = ( V0 cos u0 ) x + f(y) (1)
Also,
0f 0
= v;    3 V0 cos u0x + f(y) 4 = V0 sin u0
0y 0y
0
3f(y) 4 = V0 sin u0
0y
Integrating this equation with respect to y,
f(y) = ( V0 sin u0 ) y + C
Setting C = 0 and substituting this result into Eq. 1,
Ans:
f = ( V0 cos u0 ) x + ( V0 sin u0 ) y c = V0 3(cos u0)y - (sin u0)x4
f = V0 3 ( cos u0 ) x + ( sin u0 ) y 4 Ans. f = V0 3(cos u0)x + (sin u0)y4

727
© 2014 Pearson Education, Inc., Upper Saddle River, NJ. All rights reserved. This material is protected under all copyright laws as they currently
exist. No portion of this material may be reproduced, in any form or by any means, without permission in writing from the publisher.

7–7.  A two-dimensional flow is described by the stream


function c = 1 xy3 - x3y 2 m2 >s, where x and y are in
meters. Show that the continuity condition is satisfied and
determine if the flow is rotational or irrotational.

Solution
We consider ideal fluid flow.
0c 0
u = = ( xy3 - x3y ) = ( 3xy2 - x3 ) m>s
0y 0y
0c 0
v = - = ( xy3 - x3y ) = - ( y3 - 3x2y ) m>s = ( 3x2y - y3 ) m>s
0x 0x

Then,
0u 0

. We or
= ( 3xy2 - x3 ) = ( 3y2 - 3x2 ) s-1

m W ina g

b)
ed e n
0x 0x

in
no W iss ea s

itt id tio
is e D t w
t p or em ch
d th g. in t la
0v 0
= ( 3x2y - y3 ) = ( 3x2 - 3y2 ) s-1

an on in rs h
0y 0y
k g rn to rig
or in a uc y
w d le tr p
0u 0

er ld
e lu nt ns co

= ( 3xy2 - x3 ) = 6xy s-1


0y 0y
th inc de f i es
of rk ( stu e o tat

0v 0
( 3x2y - y3 ) = 6xy s-1
ity o g us d S

=
0x 0x
te is ss th ite
in f th se for Un

This gives,
gr w in e

0u 0v
th t o a ly by

+ = 3y2 - 3x2 + 3x2 - 3y2 = 0


0x 0y
y ar d le d
ro p an o te
st ny s d s ec

Thus, the flow field satisfies the continuity condition,


de f a rse de ot

s
ill o u vi pr

1 0v 0u
w le co ro is

vz = a - b
sa eir is p rk

2 0x 0y
th d wo

1
an his

( 6xy - 6xy ) = 0
e

=
2
T

The flow field is irrotational since vz = 0. Ans.

Ans:
irrotational

728
© 2014 Pearson Education, Inc., Upper Saddle River, NJ. All rights reserved. This material is protected under all copyright laws as they currently
exist. No portion of this material may be reproduced, in any form or by any means, without permission in writing from the publisher.

*7–8.  If the stream function for a flow is c = (3x + 2y),


where x and y are in meters, determine the potential
function and the magnitude of the velocity of a fluid particle
at point (1 m, 2 m).

Solution
We consider ideal fluid flow.
0c 0
u = = (3x + 2y) = 2 m>s
0y 0y

. We or
0c 0

m W ina g
v = - = - (3x + 2y) = - 3 m>s

b)
ed e n
in
no W iss ea s
0x 0x

itt id tio
is e D t w
t p or em ch
d th g. in t la
Since u and v are constant, the magnitude of the flow velocity at any point in the

an on in rs h
k g rn to rig
flow field is the same and is given by or in a uc y
w d le tr p

er ld
e lu nt ns co

V = 2u2 + v2 = 2(2 m>s)2 + ( - 3 m>s)2


th inc de f i es

= 3.606 m>s = 3.61 m>s Ans.


of rk ( stu e o tat
ity o g us d S

Applying,
te is ss th ite

0f 0f
in f th se for Un

u = ; 2 =
gr w in e

0x 0x
th t o a ly by
y ar d le d

Integrating with respect to x,


ro p an o te
st ny s d s ec

f = 2x + f(y)
de f a rse de ot

s
ill o u vi pr

Substituting this result into,


w le co ro is
sa eir is p rk

0f 0
th d wo

v = ;   - 3 = [2x + f(y)]


0y 0y
an his

0
T

-3 = 0 + [f(y)]
0y

Integrating with respect to y,


f(y) = - 3y + C

Setting C = 0, thus

f = 2x - 3y Ans.

729
© 2014 Pearson Education, Inc., Upper Saddle River, NJ. All rights reserved. This material is protected under all copyright laws as they currently
exist. No portion of this material may be reproduced, in any form or by any means, without permission in writing from the publisher.

7–9.  The velocity profile of a very thick liquid flowing y


along the channel of constant width is approximated as u  (3y2) mm/s
u = 13y2 2 mm>s, where y is in millimeters. Determine the
stream function for the flow and plot the streamlines for
c0 = 0, c1 = 1 mm2 >s, and c2 = 2 mm2 >s. 10 mm

Solution
We consider ideal fluid flow.
The x and y components of the constant flow velocity are

u = ( 3y2 ) mm>s    v = 0

. We or
0c 0c

m W ina g

b)
ed e n
in
;  3y2 =

no W iss ea s
u =

itt id tio
is e D t w
t p or em ch
0y 0y

d th g. in t la
an on in rs h
Integrating with respect to y,
k g rn to rig
or in a uc y
w d le tr p
c = y3 + f(x)

er ld
e lu nt ns co
th inc de f i es

0c 0
v = - ;   0 = - 3 y3 + f(x) 4
of rk ( stu e o tat

0x 0x
ity o g us d S

0
te is ss th ite

0 = [f(x)]
in f th se for Un

0x
gr w in e
th t o a ly by

Integrating with respect to x


y ar d le d

f(x) = C
ro p an o te
st ny s d s ec

Thus,
de f a rse de ot

s
ill o u vi pr

c = y3 + C
w le co ro is
sa eir is p rk
th d wo

Setting C = 0,
an his

c = y3 Ans.
T

c0 = 0, y = 0
c1 = 1, y = 1
c2 = 2, y = 1.26

1.26 c2 = 2
1 c1 = 1
x
0 c0 = 0

Ans:
c = y3

730
© 2014 Pearson Education, Inc., Upper Saddle River, NJ. All rights reserved. This material is protected under all copyright laws as they currently
exist. No portion of this material may be reproduced, in any form or by any means, without permission in writing from the publisher.

7–10.  The velocity profile of a very thick liquid flowing y


along the channel of constant width is approximated as u  (3y2) mm/s
u = 13y2 2 mm>s, where y is in millimeters. Is it possible to
determine the potential function for the flow? If so, what is it?
10 mm

Solution

. We or
m W ina g

b)
ed e n
We consider ideal fluid flow.

in
no W iss ea s

itt id tio
is e D t w
t p or em ch
The x and y components of flow velocity are

d th g. in t la
an on in rs h
u = ( 3y2 ) mm>s   v = 0
k g rn to rig
or in a uc y
w d le tr p

er ld
Here,
e lu nt ns co

0v
th inc de f i es

= 0
0x
of rk ( stu e o tat
ity o g us d S

0u 0
= ( 3y2 ) = (6y) rad>s
te is ss th ite

0y 0y
in f th se for Un
gr w in e

1 0v 0u 1
th t o a ly by

vz = a - b = (0 - 6y) = -3y
2 0x 0y 2
y ar d le d
ro p an o te
st ny s d s ec

Since vz ≠ 0, the flow is rotational. Thus, the potential function cannot be established
de f a rse de ot

since it requires the flow to be irrotational.


s
ill o u vi pr
w le co ro is
sa eir is p rk
th d wo
an his

e
T

Ans:
f cannot be established.

731
© 2014 Pearson Education, Inc., Upper Saddle River, NJ. All rights reserved. This material is protected under all copyright laws as they currently
exist. No portion of this material may be reproduced, in any form or by any means, without permission in writing from the publisher.

7–11.  The liquid confined between two plates is assumed A 1.2 m/s
to have a linear velocity distribution as shown. Determine
the stream function. Does the potential function exist?
10 mm

B
0.2 m/s

Solution 1.2 m s

. We or
m W ina g

b)
ed e n
We consider ideal fluid flow.

in
no W iss ea s

itt id tio
is e D t w
u

t p or em ch
From the geometry of Fig. a, the x component of velocity is

d th g. in t la
an on in rs h
u - 0.2 1.2 - 0.2

k g rn to rig
= ;  u = (100y + 0.2) m>s
or in a uc y 0.01 m
y 0.01
w d le tr p

er ld
y
e lu nt ns co

Also, since the velocity distribution is directed along the x axis, v = 0.


th inc de f i es
of rk ( stu e o tat

0c 0c
ity o g us d S

u = ;  100y + 0.2 =
0y 0y 0.2 m s
te is ss th ite

(a)
in f th se for Un

Integrating with respect to y,


gr w in e
th t o a ly by

c = 50y2 + 0.2y + f(x)


y ar d le d
ro p an o te

Substituting this result into,


st ny s d s ec
de f a rse de ot

0c 0
s
ill o u vi pr

v = - ;  0 = - 3 50y2 + 0.2y + f(x) 4


0x 0x
w le co ro is
sa eir is p rk

0
th d wo

[f(x)] = 0
0x
an his

e
T

Integrating with respect to x,


f(x) = C
Setting this constant equal to zero,
c = 50y2 + 0.2y Ans.

Here
0v 0u 0
= 0;  = (100y + 0.2) = 100 rad>s
0x 0y 0y
Thus,
1 0v 0u 1
vz = a - b = (0 - 100) = -50 rad>s
2 0x 0y 2
Since vz ≠ 0, the flow is rotational. Therefore, it is not possible to establish the
potential function.

Ans:
c = 50y2 + 0.2y, f cannot be established.

732
© 2014 Pearson Education, Inc., Upper Saddle River, NJ. All rights reserved. This material is protected under all copyright laws as they currently
exist. No portion of this material may be reproduced, in any form or by any means, without permission in writing from the publisher.

*7–12.  The liquid confined between two plates is assumed A 1.2 m/s
to have a linear velocity distribution as shown. If the
pressure at the top surface of the bottom plate is 600 N>m2,
detemine the pressure at the bottom surface of the top 10 mm
plate. Take r = 1.2 Mg>m3.

B
0.2 m/s

Solution 1.2 m s

We consider ideal fluid flow.


u
From the geometry of Fig. a, the x component of velocity is
u - 0.2 1.2 - 0.2
= ;  u = (100y + 0.2) m>s 0.01 m
y 0.01
y
Also, since the velocity distribution is directed along the x axis, v = 0. Here,
0v 0u 0
= 0;  = (100y + 0.2) = 100 rad>s
0x 0y 0y 0.2 m s

. We or
(a)

m W ina g

b)
ed e n
1 0v 0u 1

in
no W iss ea s

itt id tio
vz = a - b = (0 - 100) = - 50 rad>s

is e D t w
t p or em ch
2 0x 0y 2

d th g. in t la
an on in rs h
Since vz ≠ 0, the flow is rotational. Thus, the Bernoulli equation can not be applied
k g rn to rig
or in a uc y
at points A and B. Instead, we will first apply the Euler equation along the x axis,
w d le tr p

er ld
e lu nt ns co

0u 0 0u 0
with = 100y + 0.2 = 0 and = (100y + 0.2) = 100 rad>s,
th inc de f i es

0x 0x 0y 0y
of rk ( stu e o tat
ity o g us d S

1 0p 0u 0u
- = u + v
te is ss th ite

r 0x 0x 0y
in f th se for Un

1 0
gr w in e

- 3 - rgy + f(x) 4 = 0 + 0 = 0
th t o a ly by

r 0x
y ar d le d
ro p an o te

0
st ny s d s ec

[f(x)] = 0
de f a rse de ot

0x
s
ill o u vi pr

Integrating this equation with respect to x,


w le co ro is
sa eir is p rk

f(x) = C
th d wo
an his

Thus,
T

p = - rgy + C

At point B, y = 0 and p = 600 N>m2. Then,


N
600 = - 1.2 ( 103 )( 9.81 m>s2 ) (0) + C
m2
N
C = 600
m2
Thus,
N
p = ( - rgy + 600)
m2
At point A, y = 0.01 m. Then,
N
pA = 3 - 1.2 ( 103 )( 9.81 m>s2 ) (0.01 m) + 600 4
m2
N
= 482.28 = 482 Pa Ans.
m2

733
© 2014 Pearson Education, Inc., Upper Saddle River, NJ. All rights reserved. This material is protected under all copyright laws as they currently
exist. No portion of this material may be reproduced, in any form or by any means, without permission in writing from the publisher.

7–13.  A two-dimensional flow has a y component of


velocity of v = (4y) ft>s, where y is in feet. If the flow is
ideal, determine the x component of velocity and find the
magnitude of the velocity at the point x = 4 ft, y = 3 ft.
The velocity of the flow at the origin is zero.

Solution
We consider ideal fluid flow.
In order to satisfy the continuity condition,
0u 0v
+ = 0
0x 0y
Here,
0v 0
= (4y) = 4 s-1
0y 0y

. We or
Then,

m W ina g

b)
ed e n
in
no W iss ea s

itt id tio
is e D t w
t p or em ch
0u

d th g. in t la
+ 4 = 0
0x

an on in rs h
k g rn to rig
0u or in a uc y
w d le tr p
= -4

er ld
e lu nt ns co

0x
th inc de f i es

Integrating with respect to x,


of rk ( stu e o tat
ity o g us d S

u = - 4x + f(y)
te is ss th ite

Since ideal flow is irrotational,


in f th se for Un
gr w in e

0v 0u
th t o a ly by

- = 0
0x 0y
y ar d le d
ro p an o te

0v 0(4y)
st ny s d s ec

and since = = 0,
de f a rse de ot

0x 0x
s
ill o u vi pr

0u
= 0
w le co ro is

0y
sa eir is p rk
th d wo

u = g(x)
an his

Thus,
T

u = ( - 4x) ft>s Ans.

At x = 4 ft and y = 3 ft,

u = - 4(4) = - 16 ft>s   v = 4(3) = 12 ft>s

V = 2u2 + v2 = 2 ( - 16 ft>s ) 2 + ( 12 ft>s ) 2 = 20 ft>s Ans.

Ans:
u = ( - 4x) ft>s
V = 20 ft>s

734
© 2014 Pearson Education, Inc., Upper Saddle River, NJ. All rights reserved. This material is protected under all copyright laws as they currently
exist. No portion of this material may be reproduced, in any form or by any means, without permission in writing from the publisher.

7–14.  A two-dimensional flow field is defined by its


components u = (3y) m>s and v = (9x) m>s, where x and
y are in meters. Determine if the flow is rotational or
irrotational, and show that the continuity condition for the
flow is satisfied. Also, find the stream function and the
equation of the streamline that passes through point
(4 m, 3 m). Plot this streamline.

Solution y

We consider ideal fluid flow. 3


0v 0
= (9x) = 9 rad>s
0x 0x
0u 0
= (3y) = 3 rad>s x
0y 0y
3.61 4
0u 0
= (3y) = 0
0x 0x
0v 0
= (9x) = 0

. We or
0y 0y

m W ina g

b)
ed e n
in
no W iss ea s

itt id tio
Thus,

is e D t w
t p or em ch
d th g. in t la
1 0v 0u 1

an on in rs h
vz = a - b = (9 - 3) = 3 rad>s

k g rn to rig
2 0x 0y 2
or in a uc y
w d le tr p

er ld
e lu nt ns co

Since vz ≠ 0, the flow is rotational. Also, Ans.


th inc de f i es

0u 0v
of rk ( stu e o tat

+ = 0 + 0 = 0
0x 0y
ity o g us d S
te is ss th ite

The flow satisfies the continuity condition. Thus,


in f th se for Un

0c 0c
gr w in e

u = ;  3y =
th t o a ly by

0y 0y
y ar d le d
ro p an o te

Integrating with respect to y,


st ny s d s ec

3 2
de f a rse de ot

y + f(x)
s

c =
ill o u vi pr

2
w le co ro is

Substituting this result into


sa eir is p rk
th d wo

0c 0 3
v = - ;    9x = - c y2 + f(x) d
an his

0x 0x 2
T

0
- 9x = 0 + 3f(x) 4
0x
Integrating with respect to x,
9
f(x) = - x2 + C
2
Thus, setting C = 0,
3 9 1
c = y2 + a - x2 + C b = ( 3y2 - 9x2 ) Ans.
2 2 2
From the slope of the stream function,
dy v 9x 3x
= = =
dx u 3y y
y x

L3 m L4 m
ydy = 3xdx

y2 y 3 x Ans:
` = x2 ` rotational
2 3m 2 4m
1
c = 1 3y2 - 9x2 2
y2 = 3x2 - 39 2

y = 23x2 - 39 Ans. y = 23x2 - 39

735
© 2014 Pearson Education, Inc., Upper Saddle River, NJ. All rights reserved. This material is protected under all copyright laws as they currently
exist. No portion of this material may be reproduced, in any form or by any means, without permission in writing from the publisher.

7–15.  Water flow through the horizontal channel is y


defined by the stream function c = 2 1 x2 - y2 2 m2 >s. If the B
pressure at B is atmospheric, determine the pressure at
point (0.5 m, 0) and the flow per unit depth in m2 >s.

c0
1.5 m

c  0.5 m2/s

Solution
We consider ideal fluid flow. x
The velocity components are A

0c 0c 1.5 m
u = = ( -4y) m>s  v = - = ( - 4x) m>s
0y 0x
0u 0v
The continuity equation + = 0 + 0 = 0 is indeed satisfied.
0x 0y
At point A (0.5 m, 0),

. We or
uA = 0 vA = - 4(0.5) = - 2 m>s

m W ina g

b)
ed e n
in
no W iss ea s

itt id tio
is e D t w
t p or em ch
Thus,

d th g. in t la
VA = vA = - 2 m>s

an on in rs h
k g rn to rig
At point B (1.5 m, 1.5 m),
or in a uc y
w d le tr p

er ld
e lu nt ns co

u B = - 4(1.5) = -6 m>s  vB = - 4(1.5) = - 6 m>s


th inc de f i es
of rk ( stu e o tat

Thus,
ity o g us d S

VB = 2uB2 + vB2 = 2 ( - 6 m>s ) 2 + ( - 6 m>s ) 2 = 272 m>s = 8.485 m>s


te is ss th ite
in f th se for Un
gr w in e

1 0v 0u 1
th t o a ly by

Since vz = a - b = 3 - 4 - ( -4) 4 = 0, the flow is irrotational. Thus,


2 0x 0y 2
y ar d le d
ro p an o te

Bernoulli’s equation can be applied between two points on the different streamlines
st ny s d s ec

such as points A and B.


de f a rse de ot

s
ill o u vi pr

pA VA2 pB VB2
w le co ro is

= + gzA = = + gzB
sa eir is p rk

rw 2 rw 2
th d wo

Since the flow occurs in the horizontal plane, zA = zB. Also, pB = patm = 0.
an his

e
T

pA ( 2 m>s ) 2 ( 8.485 m>s ) 2


+ = 0 +
1000 kg>m3 2 2

pA = 34 ( 103 ) Pa = 34 kPa Ans.

The flow per unit depth is

c2 - c1 = 0.5 m2 >s - 0 = 0.5 m2 >s Ans.

Ans:
pA = 34 kPa
c2 - c1 = 0.5 m2 >s

736
© 2014 Pearson Education, Inc., Upper Saddle River, NJ. All rights reserved. This material is protected under all copyright laws as they currently
exist. No portion of this material may be reproduced, in any form or by any means, without permission in writing from the publisher.

*7–16.  A flow field is defined by the stream function y


c = 2 1 x2 - y2 2 m2 >s, where x and y are in meters.
Determine the flow per unit depth in m2 >s that occurs
through AB, CB, and AC as shown.
C B

3m

A x
4m

Solution
We consider ideal fluid flow.
At point A, x = 0, y = 0. Thus,
cA = 2 ( 02 - 02 ) = 0

. We or
At point B, x = 4 m, y = 3 m. Thus

m W ina g

b)
ed e n
in
no W iss ea s
cB = 2 ( 42 - 32 ) = 14 m2 >s

itt id tio
is e D t w
t p or em ch
d th g. in t la
At point C, x = 0, y = 3 m

an on in rs h
k g rn to rig
cC = 2 ( 02 - 32 ) = - 18 m2 >s
or in a uc y
w d le tr p

er ld
e lu nt ns co

The flow rates per unit depth through AB, BC, and AC are
th inc de f i es

qAB = cB - cA = 14 m2 >s - 0 = 14 m2 >s Ans.


of rk ( stu e o tat
ity o g us d S

qBC = cB - cC = 14 m >s - 2
( - 18 m >s ) = 32 m >s
2 2
Ans.
te is ss th ite

qAC = cA - cC = 0 - ( - 18 m2 >s ) = 18 m2 >s Ans.


in f th se for Un
gr w in e
th t o a ly by

Note that the flow satisfies the continuity condition through ABC since
y ar d le d

ΣV # A = 0 - qBC + qAB + qAC = - 32 m2 >s + 14 m2 >s + 18 m2 >s = 0


ro p an o te
st ny s d s ec
de f a rse de ot

s
ill o u vi pr
w le co ro is
sa eir is p rk
th d wo
an his

e
T

737
© 2014 Pearson Education, Inc., Upper Saddle River, NJ. All rights reserved. This material is protected under all copyright laws as they currently
exist. No portion of this material may be reproduced, in any form or by any means, without permission in writing from the publisher.

7–17.  A fluid has the velocity components shown. y


Determine the stream and potential functions. Plot the
streamline for c0 = 0, c1 = 1 m2 >s, and c2 = 2 m2 >s.

4 m/s

3 m/s

Solution x

We consider ideal fluid flow.


Here, the flow velocity has constant x and y components. ψ2 = 2 m2 s
y ψ1 = 1 m2 s
u = 4 m>s   v = 3 m>s ψ0 = 0
Applying
0c 0c
u = ;   4 = 0.5

. We or
0y 0y

m W ina g

b)
ed e n
0.25

in
no W iss ea s
Integrating with respect to y,

itt id tio
is e D t w
t p or em ch
d th g. in t la
x
c = 4y + f(x)

an on in rs h
k g rn to rig
Substituting this result into or in a uc y
w d le tr p

er ld
e lu nt ns co

0c 0
v = - ;   3 = 34y + f(x) 4
th inc de f i es

0x 0x
of rk ( stu e o tat

0
ity o g us d S

3 = -0 - 3f(x) 4
0x
te is ss th ite
in f th se for Un

0
gr w in e
th t o a ly by

3f(x) 4 = -3
0x
y ar d le d
ro p an o te

Integrating with respect to x,


st ny s d s ec

f(x) = - 3x + C
de f a rse de ot

s
ill o u vi pr

Setting C = 0, we get
w le co ro is
sa eir is p rk

c = 4y + ( - 3x)
th d wo

c = 4y - 3x Ans.
an his

e
T

Applying
0f 0f
u = ;    4 =
0x 0x
Integrating with respect to x,
f = 4x + f(y)
Substituting this result into
0f 0
v = ;  3 = 34x + f(y) 4
0y 0y
0
3 = 0 + 3f(y) 4
0y
Integrating with respect to y,
f(y) = 3y + C
Setting C = 0, we get
f = 4x + 3y Ans.

Ans:
c = 4y - 3x
f = 4x + 3y

738
© 2014 Pearson Education, Inc., Upper Saddle River, NJ. All rights reserved. This material is protected under all copyright laws as they currently
exist. No portion of this material may be reproduced, in any form or by any means, without permission in writing from the publisher.

7–18.  A two-dimensional flow field is defined by its


components u = 1 2x2 2 ft>s and v = 1 - 4xy + x2 2 ft>s,
where x and y are in feet. Determine the stream function,
and plot the streamline that passes through point (3 ft, 1 ft).

Solution
We consider ideal fluid flow.
0u 0v
Since the continuity equation + = 4x + ( - 4x) = 0 is satisfied, then the
0x 0y
establishment of stream function is possible.
Using the definition of velocity components with respect to the stream function,
0c 0c
= u;   = 2x2
0y 0y

. We or
Integrating this equation with respect to y,

m W ina g

b)
ed e n
in
no W iss ea s

itt id tio
c = 2x2y + f(x)

is e D t w
(1)

t p or em ch
d th g. in t la
Also,

an on in rs h
k g rn to rig
0c 0 or in a uc y
- = v;   - 3 2x2y + f(x) 4 = -4xy + x2
w d le tr p

er ld
e lu nt ns co

0x 0x
th inc de f i es

0
- 4xy - 3f(x) 4 = -4xy + x2
of rk ( stu e o tat

0x
ity o g us d S

0
te is ss th ite

3f(x) 4 = -x2
0x
in f th se for Un
gr w in e

Integrating this equation with respect to x,


th t o a ly by
y ar d le d

1
ro p an o te

f(x) = - x3 + C
st ny s d s ec

3
de f a rse de ot

s
ill o u vi pr

Substituting this result into Eq. (1),


w le co ro is

1 3
sa eir is p rk

c = 2x2y - x + C
th d wo

3
an his

Here, C is an arbitary constant that we will set equal to zero. The stream function
T

can be expressed as
1 3
c = 2x2y - x Ans.
3

For the streamline passing through point (3 ft, 1 ft),


1 3
c = 2(3)2(1) - (3)
3
Thus,
1 3
2x2y - x = 9
3
x3 + 27
y =
6x2

739
© 2014 Pearson Education, Inc., Upper Saddle River, NJ. All rights reserved. This material is protected under all copyright laws as they currently
exist. No portion of this material may be reproduced, in any form or by any means, without permission in writing from the publisher.

7–18. Continued

The plot of stream function is shown in Fig. a.


x(m) 0 1 2 3 4 5 6 7 8
y(m) ∞ 4.67 1.46 1 0.948 1.01 1.125 1.26 1.40

dy 6x2 ( 3x2 ) - ( x3 + 27 ) (12x)


= = 0
dx ( 6x2 ) 2
6x4 - 324x = 0
6x ( x3 - 54 ) = 0

. We or
m W ina g

b)
ed e n
in
no W iss ea s
x = 3.780 ft

itt id tio
is e D t w
t p or em ch
d th g. in t la
The corresponding

an on in rs h
k g rn to rig
3.7803 + 27 or in a uc y
y = = 0.945
w d le tr p

er ld
6 ( 3.7802 )
e lu nt ns co
th inc de f i es
of rk ( stu e o tat
ity o g us d S

y (m)
te is ss th ite
in f th se for Un
gr w in e
th t o a ly by

8
y ar d le d
ro p an o te
st ny s d s ec

7
de f a rse de ot

s
ill o u vi pr

6
w le co ro is
sa eir is p rk
th d wo

5
an his

e
T

2
(3.78, 0.945)

x (m)
1 2 3 4 5 6 7 8

(a)

Ans:
1
c = 2x2y - x3
3

740
© 2014 Pearson Education, Inc., Upper Saddle River, NJ. All rights reserved. This material is protected under all copyright laws as they currently
exist. No portion of this material may be reproduced, in any form or by any means, without permission in writing from the publisher.

7–19.  The stream function for a flow field is defined by


c = 1 4>r 2 2 sin 2u. Show that continuity of the flow is
satisfied, and determine the r and u velocity components of
fluid particles at point r = 2 m, u = (p>4) rad. Plot the
streamline that passes through this point.

Solution
We consider ideal fluid flow.
Using the r and u velocity components with respect to stream function

1 0c 1 4 8
vr = = a 2 b(2 cos 2u) = 3 cos 2u
r 0u r r r
0c 8 8
vu = - = - a - 3 sin 2u b = 3 sin 2u
0r r r
vr 0vr 1 0vu 8 24
The continuity equation + + = 4 cos 2u + a - 4 cos 2u b

. We or
r 0r r 0u r r

m W ina g

b)
ed e n
in
no W iss ea s
16

itt id tio
is e D t w
t p or em ch
+ cos 2u = 0 is indeed satisfied.

d th g. in t la
r4

an on in rs h
k g rn to rig
p
At point r = 2 m and u = , or in a uc y
4
w d le tr p

er ld
e lu nt ns co

8 p
vr = 3 cos c 2 a b d = 0 Ans.
th inc de f i es

2 4
of rk ( stu e o tat
ity o g us d S

8 p
vu = sin c 2 a b d = 1 m>s Ans.
te is ss th ite

3 4
2
in f th se for Un

4
gr w in e

p
th t o a ly by

c = sin c 2 a b d = 1
22 4
y ar d le d
ro p an o te
st ny s d s ec

Therefore, the stream function that passes through this point is


de f a rse de ot

4
ill o u vi pr

1 = sin 2u
w le co ro is

r2
sa eir is p rk
th d wo

r 2 = 4 sin 2u
an his

The plot of this streamline is shown in Fig. a.


T

y
π
θ =
4

r=2m

x
π
θ =
4

Ans:
vr = 0
(a) vu = 1 m>s

741
© 2014 Pearson Education, Inc., Upper Saddle River, NJ. All rights reserved. This material is protected under all copyright laws as they currently
exist. No portion of this material may be reproduced, in any form or by any means, without permission in writing from the publisher.

*7–20.  A flow field has velocity components


u = (x - y) ft>s and v = -(x + y) ft>s, where x and y are
in feet. Determine the stream function, and plot the
streamline that passes through the origin.

Solution
We consider ideal fluid flow.
0c 0c
u = ; x - y =
0y 0y
Integrating this equation with respect to y,
y2
c = xy - + f(x)
2
Substituting this result into

. We or
m W ina g
y2

b)
0c

ed e n
0

in
no W iss ea s

itt id tio
is e D t w
v = - ;   - (x + y) = - c xy - + f(x) d

t p or em ch
d th g. in t la
0x 0x 2

an on in rs h
0
k g rn to rig
x + y = y - 0 + 3f(x) 4or in a uc y
0x
w d le tr p

er ld
e lu nt ns co

0
th inc de f i es

3f(x) 4 = x
0x
of rk ( stu e o tat
ity o g us d S

Integrating with respect to x,


te is ss th ite

x2
in f th se for Un

f(x) = + C
2
gr w in e
th t o a ly by

Thus, setting C = 0,
y ar d le d
ro p an o te

y2 x2
st ny s d s ec

c = xy - + Ans.
de f a rse de ot

2 2
s
ill o u vi pr

Evaluate c(x, y) at the origin, x = y = 0. This equation gives c = 0 - 0 + 0 = 0


w le co ro is
sa eir is p rk

Then, for c = 0,
th d wo

x2 y2
an his

- + xy = 0
2 2
T

x2 - y2 + 2xy = 0
The plot of this equation is shown in Fig. a.
y c= 0

c= 0

(a)

742
© 2014 Pearson Education, Inc., Upper Saddle River, NJ. All rights reserved. This material is protected under all copyright laws as they currently
exist. No portion of this material may be reproduced, in any form or by any means, without permission in writing from the publisher.

7–21.  A flow is described by the stream function


c = (8x - 4y) m2 >s, where x and y are in meters. Determine
the potential function, and show that the continuity
condition is satisfied and that the flow is irrotational.

Solution
We consider ideal fluid flow.
0c 0
u = = (8x - 4y) = -4 m>s
0y 0y
0c 0
v = = - (8x - 4y) = - 8 m>s
0x 0x
Applying
0f 0f
u = ; -4 =

. We or
0x 0x

m W ina g

b)
ed e n
in
no W iss ea s

itt id tio
is e D t w
t p or em ch
Integrating with respect to x,

d th g. in t la
f = - 4x + f(y)

an on in rs h
k g rn to rig
Substituting this result into
or in a uc y
w d le tr p

er ld
e lu nt ns co

0f 0
th inc de f i es

v = ; - 8 = [ - 4x + f(y)]
of rk ( stu e o tat

0y 0y
ity o g us d S

0
-8 = 0 + f(y)
te is ss th ite

0y
in f th se for Un
gr w in e

0
th t o a ly by

3f(y) 4 = - 8
0y
y ar d le d
ro p an o te
st ny s d s ec

Integrating with respect to y,


de f a rse de ot

s
ill o u vi pr

f(y) = - 8y + C
w le co ro is

Thus,
sa eir is p rk
th d wo

f = - 4x + ( -8y + C)
an his

Omitting the integration constant,


T

f = - 4x - 8y Ans.
Here,
0u 0
= ( -4) = 0
0x 0x
0v 0
= ( -8) = 0
0y 0y
0u 0
= ( -4) = 0
0y 0y
0v 0
= ( - 8) = 0
0x 0x
Then,
0u 0v
+ = 0 + 0 = 0
0x 0y
The flow field satisfies the continuity condition. Also,
1 0v 0u 1
vz = a - b = (0 - 0) = 0
2 0x 0y 2
Ans:
The flow field is irrotational since vz = 0. f = - 4x - 8y

743
© 2014 Pearson Education, Inc., Upper Saddle River, NJ. All rights reserved. This material is protected under all copyright laws as they currently
exist. No portion of this material may be reproduced, in any form or by any means, without permission in writing from the publisher.

7–22.  The stream function for a flow field is defined by


c = 2r 3 sin 2u. Determine the magnitude of the velocity of
fluid particles at point r = 1 m, u = (p>3) rad, and plot the
streamlines for c1 = 1 m2 >s and c2 = 2 m2 >s.

Solution y (m)

We consider ideal fluid flow.


The velocity components are
1 0c 1 c = 2 m2 s
vr = ; vr = 3 2r 3(2 cos 2u) 4 = ( 4r 2 cos 2u ) m>s
r 0u r
0c
vu = - ; vu = ( - 6r 2 sin 2u ) m>s
0r

vr 0vr 1 0vu
The continuity equation + + = 4r cos 2u + 8r cos 2u
r 0r r 0u

. We or
m W ina g

b)
ed e n
c = 1 m2 s

in
no W iss ea s
+ ( - 12r cos 2u) = 0 is indeed satisfied.

itt id tio
is e D t w
t p or em ch
d th g. in t la
an on in rs h
At point r = 1 m, u = p>3 rad, x (m)

k g rn to rig
or in a uc y
w d le tr p

er ld
e lu nt ns co

p (a)
vr = 4 ( 12 ) cos c 2 a b d = - 2 m>s
th inc de f i es

3
of rk ( stu e o tat
ity o g us d S

p
vu = - 6 ( 12 ) sin c 2 a b d = - 5.196 m>s
3
te is ss th ite
in f th se for Un

Thus, the magnitude of the velocity is


gr w in e
th t o a ly by

V = 2vr2 + vu2 = 2 ( - 2 m>s ) 2 + ( - 5.196 m>s ) 2 = 5.57 m>s Ans.


y ar d le d
ro p an o te
st ny s d s ec

For c = 1 m2 >s,
de f a rse de ot

1
ill o u vi pr

1 = 2r 3 sin 2u r3 =
w le co ro is

2 sin 2u
sa eir is p rk
th d wo

p p p p 5p p
u(rad) 0
an his

12 6 4 3 12 2
T

r(m) ∞ 1.00 0.833 0.794 0.833 1.00 ∞

For c = 2 m2 >s
1
2 = 2r 3 sin 2u r3 =
2 sin 2u
p p p p 5p p
u(rad) 0 12 6 4 3 2
12
r(m) ∞ 1.260 1.049 1.00 1.049 1.260 ∞

Ans:
5.57 m>s

744
© 2014 Pearson Education, Inc., Upper Saddle River, NJ. All rights reserved. This material is protected under all copyright laws as they currently
exist. No portion of this material may be reproduced, in any form or by any means, without permission in writing from the publisher.

7–23.  An ideal fluid flows into the corner formed by the y


two walls. If the stream function for this flow is defined by
c = 1 5 r 4 sin 4u 2 m2 >s, show that continuity for the flow is
satisfied. Also, plot the streamline that passes through point
r = 2 m, u = (p>6) rad, and find the magnitude of the
velocity at this point.

45
x

Solution
We consider ideal fluid flow.
p
For the stream f unction passing through point r = 2 m, u = rad,
6
p
c = 5 ( 24 ) sin c 4 a b d = 4023 m2 >s
6
y (m)
Thus, the stream function passing through this point is

. We or
m W ina g

b)
ed e n
in
no W iss ea s
4023 = 5r 4 sin 4u

itt id tio
is e D t w
t p or em ch
d th g. in t la
r 4 sin 4u = 823

an on in rs h
k g rn to rig
The plot of this streamline is shown in Fig. a or in a uc y
w d le tr p

er ld
e lu nt ns co

p p p p 5p p
th inc de f i es

u(rad) 0 24 12 8 6 4
24
of rk ( stu e o tat

r(m) ∞ {2.29 {2.0 {1.93 {2.0 {2.29 ∞


ity o g us d S

45°
x (m)
te is ss th ite

The radial and transverse components of velocity are


in f th se for Un

1 0c 1
gr w in e

= 3 5r 4(4 cos 4u) 4 = 20r 3 cos 4u


th t o a ly by

vr = (a)
r 0u r
y ar d le d
ro p an o te

0c
st ny s d s ec

vu = - = - 20r 3 sin 4u
de f a rse de ot

0r
s
ill o u vi pr

The continuity equation


w le co ro is
sa eir is p rk

vr 0vr 1 0vu
th d wo

+ +
r 0r r 0u
an his

= 20r 2 cos 4u + 60r 2 cos 4u + ( -80r 2 cos 4u)


T


= 0 is indeed satisfied
At the point r = 2 m, u = p>6 rad,
p
vr = 20 ( 23 ) cos c 4 a b d = -80 m>s
6
p
vu = - 20 ( 23 ) sin c 4 a b d = - 138.56 m>s
6
Thus, the magnitude of the velocity is

V = 2vr2 + vu2 = 2 ( - 80 m>s ) 2 + ( - 138.56 m>s ) 2 = 160 m>s Ans.

Ans:
160 m>s

745
© 2014 Pearson Education, Inc., Upper Saddle River, NJ. All rights reserved. This material is protected under all copyright laws as they currently
exist. No portion of this material may be reproduced, in any form or by any means, without permission in writing from the publisher.

*7–24.  The horizontal flow confined by the walls is defined


4
by the stream function c = c 4r 4>3 sin a u b d m2 >s, where r
3 B
A
is in meters. Determine the magnitude of the velocity at
r
point r = 2 m, u = 45°. Is the flow rotational or irrotational?
45 u
Can the Bernoulli equation be used to determine the
difference in pressure between the two points A and B? O

Solution
We consider ideal fluid flow.
1 0c 1 0 4 4 1 4 4 4 16 1 4
vr = = a4r 3 sin u b = c 4r 3 a cos u b d = r 3 cos u
r 0u r 0u 3 r 3 3 3 3
0c 0 4 4 16 1 4
vu = - = - a4r 3 sin u b = - r 3 sin u
0r 0r 3 3 3

. We or
At the point r = 2 m, u = 45°.

m W ina g

b)
ed e n
in
no W iss ea s

itt id tio
is e D t w
16 1 4

t p or em ch
( 23 ) cos c (45°) d = 3.360 m>s

d th g. in t la
vr =
3 3

an on in rs h
k g rn to rig
16 1 4 or in a uc y
vu = - ( 23 ) sin c (45°) d = - 5.819 m>s
w d le tr p

er ld
3 3
e lu nt ns co
th inc de f i es

Thus, the magnitude of the velocity is


of rk ( stu e o tat
ity o g us d S

V = 2vr2 + vu2 = 2 ( 3.360 m>s ) 2 + ( - 5.819 m>s ) 2


te is ss th ite

= 6.72 m>s Ans.


in f th se for Un
gr w in e
th t o a ly by

0f 16 1 4 0f
vr = ; r 3 cos u =
y ar d le d

0r 3 3 0r
ro p an o te
st ny s d s ec

Integrating with respect to r,


de f a rse de ot

s
ill o u vi pr

4 4
f = 4r 3 cos u + f(u)
w le co ro is

3
sa eir is p rk
th d wo

Substituting this result into,


an his

1 0f 16 1 4 1 0 4
T

4
vu = ;    - r 3 sin u = c 4r 3 cos u + f(u) d
r 0u 3 3 r 0u 3

16 1 4 1 4 4 1 0
- r 3 sin u = ( 4r 3 ) c - sin u d + 3f(0) 4
3 3 3 3 r 0u
0
3f(u) 4 = 0
0u
Integrating with respect to u,
f(u) = C

Setting the constant equal to zero,


4 4
f = 4r 3 cos u
3
Since the potential function can be established, the flow is irrotational. Therefore,
the Bernoulli equation is applicable between any two points in the flow, including
points A and B.

746
© 2014 Pearson Education, Inc., Upper Saddle River, NJ. All rights reserved. This material is protected under all copyright laws as they currently
exist. No portion of this material may be reproduced, in any form or by any means, without permission in writing from the publisher.

7–25.  The horizontal flow between the walls is defined by


4
the stream function c = c 4r 4>3 sin a u b d m2 >s, where r is
3 B
A
in meters. If the pressure at the origin O is 20 kPa, determine
r
the pressure at r = 2 m, u = 45°. Take r = 950 kg>m3.
45 u
O

Solution
We consider ideal fluid flow.
1 0c 1 0 4 4 1 4 4 4 16 1 4
vr = = a4r 3 sin u b = c 4r 3 a cos u b d = r 3 cos u
r 0u r 0u 3 r 3 3 3 3
0c 0 4 4 16 1 4
vu = - = - a4r 3 sin u b = - r 3 sin u
0r 0r 3 3 3
At point A, where r = 2 m u = 45°,
16 1 4
( 23 ) cos c (45°) d = 3.360 m>s

. We or
vr =
3 3

m W ina g

b)
ed e n
in
no W iss ea s

itt id tio
is e D t w
t p or em ch
16 1 4

d th g. in t la
vu = - ( 23 ) sin c (45°) d = -5.819 m>s

an on in rs h
3 3
At the origin O, where r = 0,
k g rn to rig
or in a uc y
w d le tr p

er ld
e lu nt ns co

vr = v u = 0
th inc de f i es

Thus, the magnitude of the velocity at these two points is


of rk ( stu e o tat
ity o g us d S

Vo = VA = 2vr2 + vu2 = 2 ( 3.360 m>s ) 2 + ( - 5.819 m>s ) 2


te is ss th ite

= 6.720 m>s
in f th se for Un
gr w in e
th t o a ly by

0f 16 1 4 0f
vr = ; r 3 cos u =
y ar d le d

0r 3 3 0r
ro p an o te
st ny s d s ec

Integrating with respect to r,


de f a rse de ot

4
ill o u vi pr

4
f = 4r 3 cos u + f(u),
w le co ro is

3
sa eir is p rk

Substituting this result into,


th d wo

1 0f 16 1 4 1 0 4
an his

4
vu = ;  - r 3 sin u = c 4r 3 cos u + f(u) d
T

r 0u 3 3 r 0u 3
16 1 4 1 4 4 1 0
- r 3 sin u = ( 4r 3 ) c - sin u d + 3f(u) 4
3 3 3 3 r 0u
0
3f(u) 4 = 0
0u
Integrating with respect to u,
f(u) = C
Setting this constant equal to zero,
4 4
f = 4r 3 cos u
3
Since the potential function can be established, the flow is irrotational. Therefore,
the Bernoulli equation is applicable between any two points in the flow, including
points A and O.
p V2 pO VO2
+ = + 
r 2 r 2
N
20 ( 103 )
p (6.720 m>s)2 m2
+ = + 0
950 kg>m3 2 950 kg>m3 Ans:
p = - 1.448 ( 103 ) Pa = - 1.45 kPa Ans. - 1.45 kPa

747
© 2014 Pearson Education, Inc., Upper Saddle River, NJ. All rights reserved. This material is protected under all copyright laws as they currently
exist. No portion of this material may be reproduced, in any form or by any means, without permission in writing from the publisher.

7–26.  The flat plate is subjected to the flow defined by the


stream function c = 3 8r 1>2 sin (u>2) 4 m2 >s. Sketch the
streamline that passes through point r = 4 m, u = p rad,
and determine the magnitude of the velocity at this point.
u
r

Solution
We consider ideal fluid flow.
For the stream function passing point r = 4 m and u = p rad,
1 p
c = 8 ( 42 ) sin = 16
2
Thus, the stream function passing through this point is
1 u

. We or
16 = 8r 2 sin
2

m W ina g

b)
ed e n
in
no W iss ea s

itt id tio
is e D t w
u

t p or em ch
1
r sin = 2

d th g. in t la
2
2

an on in rs h
k g rn to rig
The plot of this function is shown in Fig. a or in a uc y
w d le tr p

er ld
e lu nt ns co

0 p p 2pp 5p 7p 4p 3p 5p 11p
th inc de f i es

u(rad) p
6 3 32 6 6 3 2 3 6
of rk ( stu e o tat
ity o g us d S

r(m) ∞ 59.71 16.0 8.00 5.33 4.29 4.00 4.29 5.33 8.00 16.0 59.71
te is ss th ite

2p
in f th se for Un
gr w in e


th t o a ly by
y ar d le d
ro p an o te
st ny s d s ec
de f a rse de ot

s
ill o u vi pr

x (m)
ψ = 16
w le co ro is
sa eir is p rk
th d wo
an his

e
T

y (m)

(a)

748
© 2014 Pearson Education, Inc., Upper Saddle River, NJ. All rights reserved. This material is protected under all copyright laws as they currently
exist. No portion of this material may be reproduced, in any form or by any means, without permission in writing from the publisher.

7–26. Continued

The radial and transverse components of velocity are


u
4 cos
1 0c 1 1 1 u 2
vr = = c 8r 2 a cos b d =
r 0u r 2 2 1

. We or
r2

m W ina g

b)
ed e n
in
no W iss ea s

itt id tio
is e D t w
t p or em ch
d th g. in t la
u
4 sin
0c 1 2

an on in rs h
1 u
= - a8r - 2 sin b = -
k g rn to rig
vu = -
0r 2 2 or in a uc y r2
1
w d le tr p

er ld
e lu nt ns co
th inc de f i es

u u u
4 cos -2 cos 2 cos
of rk ( stu e o tat

vr 0vr 1 0vu 2 2 2
The continuity equation + + = + ± ≤ + ±- ≤ = 0
ity o g us d S

r 0r r 0u r2
3 3
r2 r2
3
te is ss th ite
in f th se for Un

is indeed satisfied.
gr w in e
th t o a ly by
y ar d le d

At point r = 4 m, u = p rad,
ro p an o te
st ny s d s ec

p
de f a rse de ot

4 cos
s
ill o u vi pr

2
vr = = 0
w le co ro is

1
42
sa eir is p rk
th d wo

p
4 sin
2
an his

vu = - = - 2 m>s
T

1
42
Thus, the magnitude of the velocity is
V = vu = 2 m>s Ans.

Ans:
2 m>s

749
© 2014 Pearson Education, Inc., Upper Saddle River, NJ. All rights reserved. This material is protected under all copyright laws as they currently
exist. No portion of this material may be reproduced, in any form or by any means, without permission in writing from the publisher.

7–27.  An A-frame house has a window A on its right side.


If the stream function that models the flow as this side is
defined as c = (2r 1.5sin 1.5 u) ft 2 >s, show that continuity of
the flow is satisfied, and then determine the wind speed past
the window located at r = 10 ft, u = (p>3) rad. Sketch the
streamline that passes through this point. A

r  10 ft

u  120

Solution
We consider ideal fluid flow.
For the stream function passing through point r = 10 ft, u = p>3 rad,

. We or
c = 2 ( 101.5 ) sin 31.5 ( p>3 ) 4 = 2 ( 101.5 )

m W ina g

b)
ed e n
in
no W iss ea s

itt id tio
is e D t w
t p or em ch
Thus, the stream function passing through this point is

d th g. in t la
an on in rs h
2(101.5) = 2r 1.5 sin 1.5u

k g rn to rig
r 1.5 sin 1.5u = 101.5 or in a uc y
w d le tr p

er ld
e lu nt ns co

The plot of this stream function is shown in Fig. a


th inc de f i es
of rk ( stu e o tat
ity o g us d S

u(rad) 0 p p p p 5p p 7p 2p
12 6 4 3 2
te is ss th ite

12 12 3
in f th se for Un

r(ft) ∞ 18.97 12.60 10.54 10 10.54 12.60 18.97 ∞


gr w in e
th t o a ly by
y ar d le d
ro p an o te

y (ft)
st ny s d s ec
de f a rse de ot

s
ill o u vi pr
w le co ro is
sa eir is p rk
th d wo
an his

e
T

ψ = 2 (101.5)

x (ft)

120° = rad
3

(a)

750
© 2014 Pearson Education, Inc., Upper Saddle River, NJ. All rights reserved. This material is protected under all copyright laws as they currently
exist. No portion of this material may be reproduced, in any form or by any means, without permission in writing from the publisher.

7–27. Continued

The radial and transverse components of velocity are


1 0c 1
= 3 2r 1.5(1.5 cos 1.5u) 4 = 3r 2 cos 1.5u
1
vr =
r 0u r

. We or
0c 1

m W ina g
vu = - = - 3r 2 sin 1.5u

b)
ed e n
in
no W iss ea s
0r

itt id tio
is e D t w
t p or em ch
d th g. in t la
vr 0vr 1 0vu 3 cos 1.5u 1.5 cos 1.5u

an on in rs h
The continuity equation + + = +

k g rn to rig
r 0r r 0u r
1
2
1
r2
or in a uc y
w d le tr p

er ld
e lu nt ns co

4.5 cos 1.5u


+ a- b = 0 is indeed satisfied.
th inc de f i es

1
r2
of rk ( stu e o tat
ity o g us d S

2
At the window, where r = 10 ft, u = 120° = p rad,
te is ss th ite

3
in f th se for Un

2p
gr w in e

1
vr = 3 ( 10 2 ) cos c1.5 a
th t o a ly by

b d = -9.4868 ft>s
3
y ar d le d
ro p an o te

2p
st ny s d s ec

1
v u = - 3 ( 10 2 ) sin c 1.5a bd = 0
de f a rse de ot

3
s
ill o u vi pr
w le co ro is

Thus, the magnitude of the wind velocity is


sa eir is p rk
th d wo

V = vr = 9.49 ft>s Ans.


an his

e
T

Ans:
9.49 ft>s

751
© 2014 Pearson Education, Inc., Upper Saddle River, NJ. All rights reserved. This material is protected under all copyright laws as they currently
exist. No portion of this material may be reproduced, in any form or by any means, without permission in writing from the publisher.

*7–28.  The stream function for a horizontal flow near y


the corner is c = (8xy) m2 >s, where x and y are in meters.
Determine the x and y components of the velocity and the
acceleration of fluid particles passing through point (1 m,
2 m). Show that it is possible to establish the potential
function. Plot the streamlines and equipotential lines that
pass through point (1 m, 2 m).

B
A

Solution
We consider ideal fluid flow.
For the stream function passing through point (1 m, 2 m),

. We or
m W ina g
c = 8(1)(2) = 16

b)
ed e n
in
no W iss ea s

itt id tio
is e D t w
t p or em ch
d th g. in t la
Thus,

an on in rs h
2

k g rn to rig
16 = 8xy  y =
x or in a uc y
w d le tr p

er ld
Using the velocity components,
e lu nt ns co
th inc de f i es

0c 0c
u = = (8x) m>s  v = - = ( -8y) m>s
of rk ( stu e o tat

0y 0x
ity o g us d S
te is ss th ite

0u 0v
The continuity equation + = 8 + ( -8) = 0 is indeed satisfied.
in f th se for Un

0x 0y
gr w in e
th t o a ly by

The acceleration components are


y ar d le d
ro p an o te

0u 0u 0u
st ny s d s ec

ax = + u + v
0t 0x 0y
de f a rse de ot

s
ill o u vi pr

= 0 + 8x(8) + ( - 8y)(0) = (64x) m>s2


w le co ro is
sa eir is p rk

0v 0v 0v
th d wo

ay = + u + v
0t 0x 0y
an his

e
T

= 0 + 8x(0) + ( - 8y)( - 8) = (64y) m>s2


At point (1 m, 2 m),
u = 8(1) = 8 m>s S v = - 8(2) = - 16 m>s = 16 m>s T  Ans.
ax = 64(1) = 64 m>s2 S ay = 64(2) = 128 m>s2 c  Ans.
1 0v 0u 1
Since vz = a - b = (0 - 0) = 0, the flow is irrotational. Therefore, it is
2 0x 0y 2
possible to establish potential function. Using the velocity components,
0f 0f
= u;   = 8x
0x 0x
Integrating this equation with respect to x
f = 4x2 + f(y) (1)

752
© 2014 Pearson Education, Inc., Upper Saddle River, NJ. All rights reserved. This material is protected under all copyright laws as they currently
exist. No portion of this material may be reproduced, in any form or by any means, without permission in writing from the publisher.

7–28. Continued

Also
0f 0
= v; 3 4x2 + f(y) 4 = -8y
0y 0y
0

. We or
3 f(y) 4 = - 8y

m W ina g

b)
0y

ed e n
in
no W iss ea s

itt id tio
is e D t w
t p or em ch
Integrating this equation with respect to y

d th g. in t la
f(y) = - 4y2 + c

an on in rs h
k g rn to rig
Setting C = 0 and substituting this result into Eq. 1 or in a uc y
w d le tr p

er ld
e lu nt ns co

f = 4x2 - 4y2
th inc de f i es
of rk ( stu e o tat

f = 4(x2 - y2) Ans.


ity o g us d S

The potential function passing through point (1 m, 2 m),


te is ss th ite
in f th se for Un

Then
gr w in e
th t o a ly by

f = 4 ( 12 - 22 ) = - 12
y ar d le d
ro p an o te

Thus
st ny s d s ec
de f a rse de ot

- 12 = 4 ( x2 - y2 )   y2 = x2 + 3
s
ill o u vi pr
w le co ro is

The plots of the stream and potential functions are shown in Fig. a.
sa eir is p rk

y (m)
th d wo

For the stream function


an his

x(m) 0 1 2 3 4 5 6 7
T

y(m) ∞ 2 1 0.667 0.5 0.4 0.333


6
For the potential function
y2 = x2 + 3
x(m) 0 1 2 3 4 5 6 5 (Potential function)
y(m) 1.73 2 2.65 3.46 4.36 5.29 6.24
4

2
y = 2x (Stream function)
1

x (m)
0
1 2 3 4 5 6

(a)

753
© 2014 Pearson Education, Inc., Upper Saddle River, NJ. All rights reserved. This material is protected under all copyright laws as they currently
exist. No portion of this material may be reproduced, in any form or by any means, without permission in writing from the publisher.

7–29.  The stream function for horizontal flow near the y


corner is defined by c = (8xy) m2 >s, where x and y are in
meters. Show that the flow is irrotational. If the pressure at
point A (1 m, 2 m) is 150 kPa, determine the pressure at
point B (2 m, 3 m). Take r = 980 kg>m3.

B
A

Solution
We consider ideal fluid flow.
Using the velocity components,

. We or
0c 0c

m W ina g

b)
ed e n
u = = (8x) m>s  v = - = -( -8y) m>s

in
no W iss ea s

itt id tio
is e D t w
0y 0x

t p or em ch
d th g. in t la
an on in rs h
0u 0v

k g rn to rig
The continuity equation + = 8 + ( -8) = 0 is indeed satisfied
0x 0y or in a uc y
w d le tr p

er ld
e lu nt ns co
th inc de f i es

At point A(1 m, 2 m),


of rk ( stu e o tat

u A = 8(1) = 8 m>s  vA = - 8(2) = -16 m>s


ity o g us d S
te is ss th ite

Thus,
in f th se for Un

VA2 = uA2 + vA2 = ( 8 m>s ) 2 + ( - 16 m>s ) 2 = 320 m2 >s2


gr w in e
th t o a ly by

At point B(2 m, 3 m)
y ar d le d
ro p an o te
st ny s d s ec

u B = 8(2) = 16 m>s  vB = - 8(3) = -24 m>s


de f a rse de ot

s
ill o u vi pr

Thus,
w le co ro is

V B2 = u B2 + vB2 = ( 16 m>s ) 2 + ( - 24 m>s ) 2 = 832 m2 >s2


sa eir is p rk
th d wo

1 0v 0u 1
an his

Since vz = a + b = (0 + 0) = 0, the flow is irrotational. Therefore,


2 0x 0y 2
T

Bernoulli’s equation is applicable to two points located on the different streamlines


such as points A and B.
pA VA2 pB VB2
+ + gzA = + + gzB
r 2 r 2
Since the flow is in the horizontal plane, zA = zB = z.

pA VA2 pB VB2
+ + gz = + + gz
r 2 r 2
r
pB = pA + ( VA2 - VB2 )
2
980 kg>m3
pB = 150 ( 103 ) N>m2 + ( 320 m2 >s2 - 832 m2 >s2 )
2
= - 100.88 ( 103 ) N>m2
= - 101 kPa Ans.

Ans:
- 101 kPa

754
© 2014 Pearson Education, Inc., Upper Saddle River, NJ. All rights reserved. This material is protected under all copyright laws as they currently
exist. No portion of this material may be reproduced, in any form or by any means, without permission in writing from the publisher.

7–30.  A flow has velocity components u = 1 2x2 2 ft>s and


v = ( -4xy + 8) ft>s, where x and y are in feet. Determine
the magnitude of the acceleration of a particle located at
point (3 ft, 2 ft). Is the flow rotational or irrotational? Also,
show that continuity of flow is satisfied.

Solution
We consider ideal fluid flow.
0v 0

. We or
= ( -4xy + 8) = - 4y
0x 0x

m W ina g

b)
ed e n
in
no W iss ea s

itt id tio
is e D t w
t p or em ch
0u 0

d th g. in t la
= ( 2x2 ) = 0

an on in rs h
0y 0y

k g rn to rig
0u 0 or in a uc y
w d le tr p

er ld
= ( 2x2 ) = 4x
e lu nt ns co

0x 0x
th inc de f i es

0v 0
of rk ( stu e o tat

= ( -4xy + 8) = - 4x
ity o g us d S

0y 0y
te is ss th ite
in f th se for Un

Thus,
gr w in e
th t o a ly by

1 0v 0u 1
a - b = ( -4y - 0) = -2y
y ar d le d

vz =
ro p an o te

2 0x 0y 2
st ny s d s ec
de f a rse de ot

Since vz ≠ 0, the flow is rotational. Also


s
ill o u vi pr

0u 0v
w le co ro is

+ = 4x + ( - 4x) = 0
sa eir is p rk

0x 0y
th d wo

The flow satisfies the continuity condition.


an his

e
T

0u 0v
Since = = 0 (steady flow)
0t 0t
0u 0u 0u
ax = + u + v
0t 0x 0y
= 0 + ( 2x2 )( 4x ) + ( - 4xy + 8)(0) = 8x3
0v 0v 0v
ay = + u + v
0t 0x 0y
= 0 + 2x2( - 4y) + ( -4xy + 8)( - 4x) = - 8x2y + 16x2y - 32x = 8x2y - 32x
Thus, at x = 3 ft, y = 2 ft
ax = 8 ( 33 ) = 216 ft>s2
ay = 8(32)(2) - 32(3) = 48 ft>s2
The magnitude of the acceleration is
a = 2ax2 + ay2 = 2 ( 216 ft>s2 ) 2 + ( 48 ft>s2 ) 2
= 221.27 ft>s2 = 221 ft>s2 Ans.

Ans:
rotational
a = 221 ft>s2

755
© 2014 Pearson Education, Inc., Upper Saddle River, NJ. All rights reserved. This material is protected under all copyright laws as they currently
exist. No portion of this material may be reproduced, in any form or by any means, without permission in writing from the publisher.

7–31.  The potential function for a flow is


f = 1 x2 - y2 2 ft 2 >s, where x and y are in feet. Determine
the magnitude of the velocity of fluid particles at point A
(3  ft, 1 ft). Show that continuity is satisfied, and find the
streamline that passes through point A.

Solution
We consider ideal fluid flow.
From the velocity components
0f 0f
u = = 2x  v = = - 2y
0x 0y
1 0v 0u 1
Since vz = a - b = (0 - 0) = 0, the flow is indeed irrotational.
2 0x 0y 2

. We or
m W ina g

b)
ed e n
At point (3 ft, 1 ft),

in
no W iss ea s

itt id tio
is e D t w
t p or em ch
d th g. in t la
u = 2(3) = 6 ft>s  v = - 2(1) = - 2 ft>s

an on in rs h
k g rn to rig
Then the magnitude of the velocity is
or in a uc y
w d le tr p

er ld
V = 2u2 + v2 = 2 ( 6 ft>s ) 2 + ( - 2 ft>s ) 2 = 6.32 ft>s
e lu nt ns co

Ans.
th inc de f i es

0u 0v 0u 0y
of rk ( stu e o tat

Here = 2 and = - 2.  Since + = 2 + ( -2) = 0, the potential function


ity o g us d S

0x 0y 0x 0y
satisfies the continuity condition. Using the velocity components.
te is ss th ite
in f th se for Un

0c 0c
= u;  = 2x
gr w in e
th t o a ly by

0y 0y
y ar d le d

Integrating this equation with respect to y,


ro p an o te
st ny s d s ec

c = 2xy + f(x) (1)


de f a rse de ot

s
ill o u vi pr

Also,
w le co ro is
sa eir is p rk

0c 0
- = v;  - 32xy + f(x) 4 = - 2y
th d wo

0x 0x
an his

0
T

2y + 3f(x) 4 = 2y
0x
0
3f(x) 4 = 0
0x
Integrating this equation with respect to x,
f(x) = C
Setting C = 0, and substituting this result into Eq. 1
c = 2xy
For the streamline passing through point (3 ft, 1 ft),
c = 2(3)(1) = 6
Thus,
6 = 2xy
xy = 3 Ans.

Ans:
V = 6.32 ft>s
xy = 3

756
© 2014 Pearson Education, Inc., Upper Saddle River, NJ. All rights reserved. This material is protected under all copyright laws as they currently
exist. No portion of this material may be reproduced, in any form or by any means, without permission in writing from the publisher.

*7–32.  The flow around the bend in the horizontal channel A


can be described as a free vortex for which
vr = 0, vu = (8>r) m>s, where r is in meters. Show that the B
flow is irrotational. If the pressure at point A is 4 kPa, C
determine the pressure at point B.  Take r = 1100 kg>m3. r
2m
u

0.5 m

Solution
We consider ideal fluid flow.
0f 0f
vr = ;  0 =
0r 0r

. We or
m W ina g

b)
ed e n
Integrating with respect to r,

in
no W iss ea s

itt id tio
is e D t w
t p or em ch
d th g. in t la
f = f(u)

an on in rs h
k g rn to rig
Substituting this result into
or in a uc y
w d le tr p
1 0f 8

er ld
1 0
e lu nt ns co

vu = ;  = 3f(u)4
r 0u r r 0u
th inc de f i es
of rk ( stu e o tat

0
3f(u)4 = 8
ity o g us d S

0u
te is ss th ite

Integrating with respect to u,


in f th se for Un
gr w in e

f = f(u) = 8u + C
th t o a ly by
y ar d le d

Since the potential function can be established, the flow is irrotational and Bernoulli’s
ro p an o te
st ny s d s ec

equation can be applied between points A and B. The magnitude of velocity at A


de f a rse de ot

and B is
s
ill o u vi pr

8 8
w le co ro is

VA = (vu)A = = = 3.2 m>s


sa eir is p rk

rA 2.5
th d wo

8 8
an his

VB = (vu)B = = = 4 m>s
rB 2
T

Applying the Bernoulli equation,


pB VB2 pA VA2
+ = +
r 2 r 2
N
4 ( 103 )
pB ( 4 m>s ) 2 m2 ( 3.2 m>s ) 2
+ = +
1100 kg>m3 2 1100 kg>m3 2

pB = 832 Pa Ans.

757
© 2014 Pearson Education, Inc., Upper Saddle River, NJ. All rights reserved. This material is protected under all copyright laws as they currently
exist. No portion of this material may be reproduced, in any form or by any means, without permission in writing from the publisher.

7–33.  The velocity components for a two-dimensional


flow are u = (8y) ft>s and v = (8x) ft>s, where x and y are
in feet. Determine if the flow is rotational or irrotational,
and show that continuity of flow is satisfied.

Solution
We consider ideal fluid flow.
0v 0
= (8x) = 8 rad>s
0x 0x

. We or
0u 0

m W ina g

b)
ed e n
= (8y) = 8 rad>s

in
no W iss ea s

itt id tio
0y 0y

is e D t w
t p or em ch
d th g. in t la
0u 0

an on in rs h
= (8y) = 0
k g rn to rig
0x 0x or in a uc y
w d le tr p

er ld
0v 0
e lu nt ns co

= (8x) = 0
0y 0y
th inc de f i es
of rk ( stu e o tat

Thus,
ity o g us d S
te is ss th ite

1 0v 0u 1
a - b = (8 - 8) = 0
in f th se for Un

vz =
2 0x 0y 2
gr w in e
th t o a ly by

Since vz = 0, the flow is irrotational. Also, Ans.


y ar d le d
ro p an o te
st ny s d s ec

0u 0v
+ = 0 + 0 = 0
de f a rse de ot

0x 0y
s
ill o u vi pr
w le co ro is

The flow satisfies the continuity condition. Ans.


sa eir is p rk
th d wo
an his

e
T

Ans:
irrotational

758
© 2014 Pearson Education, Inc., Upper Saddle River, NJ. All rights reserved. This material is protected under all copyright laws as they currently
exist. No portion of this material may be reproduced, in any form or by any means, without permission in writing from the publisher.

7–34.  The velocity components for a two-dimensional


flow are a u = (8y) ft>s and v = (8x) ft>s where x and y are
in feet. Find the stream function and the equation of the
streamline that passes through point (4 ft, 3 ft). Plot this
streamline.

Solution y

We consider ideal fluid flow. 3


0c 0c
u = ;  8y =
0y 0y

. We or
m W ina g
Integrating with respect to y,

b)
ed e n
in
no W iss ea s

itt id tio
x

is e D t w
t p or em ch
4

d th g. in t la
c = 4y2 + f(x)

an on in rs h
Then,
k g rn to rig
or in a uc y 2.65
0c
w d le tr p
0

er ld
;  8x = - 3 4y2 + f(x)4
e lu nt ns co

v = -
0x 0x
th inc de f i es
of rk ( stu e o tat

0
- 8x = 0 + 3f(x)4
ity o g us d S

0x
te is ss th ite

Integrating with respect to x,


in f th se for Un

f(x) = - 4x2 + C
gr w in e
th t o a ly by

Thus,
y ar d le d
ro p an o te
st ny s d s ec

c = 4y2 + ( - 4x2 + C ) = 4 ( y2 - x2 ) + C
de f a rse de ot

s
ill o u vi pr

Omitting the constant, C,


w le co ro is

c = 4 ( y2 - x2 ) Ans.
sa eir is p rk
th d wo

From the slope of the stream function,


an his

dy v 8x x
T

= = =
dx u 8y y
y x

L3 ft L4 ft
ydy = xdx

y2 y x2 2 x
2 =
2 3 ft 2 4 ft

y2 = x2 - 7

y = { 2x2 - 7 Ans.

Also, at (4 ft, 3 ft),

c = 4 ( (3)2 - (4)2 ) = - 28
Then,
4 ( y2 - x2 ) = - 28

y = { 2x2 - 7 Ans:
c = 4 1y2 - x22
y = { 2x2 - 7

759
© 2014 Pearson Education, Inc., Upper Saddle River, NJ. All rights reserved. This material is protected under all copyright laws as they currently
exist. No portion of this material may be reproduced, in any form or by any means, without permission in writing from the publisher.

7–35.  The stream function for the flow field around the y
90° corner is c = 8r 2 sin 2u. Show that the continuity of
flow is satisfied. Determine the r and u velocity components
of a fluid particle located at r = 0.5 m, u = 30°, and plot the
streamline that passes through this point. Also, determine
the potential function for the flow.

Solution
We consider ideal fluid flow.
From the r and u velocity components to,
1 0c 1
vr = = ( 8r 2 ) (2 cos 2u) = 16r cos 2u

. We or
r 0u r

m W ina g

b)
ed e n
in
no W iss ea s

itt id tio
is e D t w
t p or em ch
0c

d th g. in t la
vu = = - (16r sin 2u) = -16r sin 2u
0r

an on in rs h
k g rn to rig
vr 0vr 1 0vu or in a uc y
The continuity equation + + = 16 cos 2u + 16 cos 2u +
w d le tr p

er ld
r 0r r 0u
e lu nt ns co

( - 32 cos 2u) = 0 is indeed satisfied.


th inc de f i es
of rk ( stu e o tat

At point r = 0.5 m, u = 30°,


ity o g us d S
te is ss th ite

vr = 16(0.5) cos 32(30°) 4 = 4 m>s Ans.


in f th se for Un

vu = - 16(0.5) sin 32(30°) 4 = -6.93 m>s Ans.


gr w in e
th t o a ly by
y ar d le d

y x y x 2xy
ro p an o te

Since sin u = , cos u = , then sin 2u = 2 sin u cos u = 2 a ba b = 2 .


st ny s d s ec

r r r r r
de f a rse de ot

Therefore,
s
ill o u vi pr
w le co ro is

2xy
c = 8r 2 a
sa eir is p rk

b = 16xy
r2
th d wo
an his

At point r = 0.5 m, u = 30°,


T

23
x = r cos u = (0.5 m) cos 30° = m
4
1
y = r sin u = (0.5 m) sin 30° = m
4
Then

23 1
c = 16° ¢ a b = 23
4 4
Thus, the streamline passing through this point is

23 = 16xy

23
y =
16x

760
© 2014 Pearson Education, Inc., Upper Saddle River, NJ. All rights reserved. This material is protected under all copyright laws as they currently
exist. No portion of this material may be reproduced, in any form or by any means, without permission in writing from the publisher.

7–35. Continued

y (m)
1.1
1.0
0.9
0.8
0.7
0.6
0.5
0.4
0.3
0.25
0.2
0.1

. We or
m W ina g

b)
ed e n
in
x (m)

no W iss ea s

itt id tio
is e D t w
t p or em ch
0.1 0.2 0.3 0.4 0.5 0.6 0.7 0.8 0.9 1.0

d th g. in t la
an on in rs h
0.4333

k g rn to rig
(a)or in a uc y
w d le tr p

er ld
e lu nt ns co
th inc de f i es
of rk ( stu e o tat
ity o g us d S

The plot of this streamline is shown in Fig. a


te is ss th ite
in f th se for Un
gr w in e

x(m) 0 0.1 0.2 0.3 0.4 0.5 0.6 0.7 0.8 0.9 1.0
th t o a ly by
y ar d le d

y(m) ∞ 1.08 0.541 0.361 0.271 0.217 0.180 0.155 0.135 0.120 0.108
ro p an o te
st ny s d s ec

The velocity components with respect to stream function are


de f a rse de ot

s
ill o u vi pr

0c 0c
w le co ro is

u = = 16x  v = - - 16y
0y 0x
sa eir is p rk
th d wo

1 0v 0u 1
an his

Since vz = a - b = (0 - 0) = 0, the flow is irrotational. Therefore, it is


2 0x 0y 2
T

possible to established the potential function using the velocity components,


0f 0f
= u;  = 16x
0x 0x
Integrating this equation with respect to x,
f = 8x2 + f(y) (1)
Also,
0f 0
= v;  3 8x2 + f(y)4 = -16y
0y 0y
0
3f(y) 4 = - 16y
0y
Integrating this equation with respect to y
f(y) = - 8y2 + C

Setting C = 0, and substituting this result in Eq. 1


f = 8x2 - 8y2 + C Ans:
vr = 4 m>s
f = 8 ( x2 - y2 ) Ans. vu = - 6.93 m>s
f = 8 1x2 - y 2 2

761
© 2014 Pearson Education, Inc., Upper Saddle River, NJ. All rights reserved. This material is protected under all copyright laws as they currently
exist. No portion of this material may be reproduced, in any form or by any means, without permission in writing from the publisher.

*7–36.  The stream function for a concentric flow is defined y


by c = - 4r 2. Determine the velocity components vr and vu,
and vx and vy. Can the potential function be established? If
so, what is it?

u
x

Solution
We consider ideal fluid flow.
Using the r, u velocity components
1 0c 1
= (0) = 0 Ans.

. We or
vr =
r 0u r

m W ina g

b)
ed e n
in
no W iss ea s

itt id tio
is e D t w
t p or em ch
0c

d th g. in t la
vu = - = - 3 - 4(2r) 4 = 8r Ans.
0r

an on in rs h
k g rn to rig
Since r 2 = x2 + y2, then c = - 4 ( x2 + y2 ) . Using the velocity components,
or in a uc y
w d le tr p

er ld
e lu nt ns co

0c
th inc de f i es

vx = u = = -4(2y) = - 8y Ans.
0y
of rk ( stu e o tat
ity o g us d S

0c
vy = v = - = - 3 - 4(2x) 4 = 8x Ans.
te is ss th ite

0x
in f th se for Un
gr w in e
th t o a ly by

1 0v 0u 1
Since vz = a - b = 38 - ( - 8) 4 = 16 ≠ 0, the flow is rotational.
y ar d le d

2 0x 0y 2
ro p an o te
st ny s d s ec

Therefore, the potential function cannot be established.


de f a rse de ot

s
ill o u vi pr
w le co ro is
sa eir is p rk
th d wo
an his

e
T

762
© 2014 Pearson Education, Inc., Upper Saddle River, NJ. All rights reserved. This material is protected under all copyright laws as they currently
exist. No portion of this material may be reproduced, in any form or by any means, without permission in writing from the publisher.

7–37.  A fluid has velocity components u = (x - y) ft>s


and v = - (x + y) ft>s, where x and y are in feet. Determine
the stream and potential functions. Show that the flow is
irrotational.

Solution
We consider ideal fluid flow.
0u 0v
Since the continuity equation + = 1 + ( - 1) = 0 is satisfied, then the
0x 0y
establishment of a stream function is possible. Using the velocity components,
0c 0c
= u;  = x - y
0y 0x
Integrating this equation with respect to y,
1 2
c = xy - y + f(x) (1)
2
Also,
0c 0 1
= c xy - y2 + f(x) d = -(x + y)

. We or
- = v;  -
0x 0x 2

m W ina g

b)
ed e n
in
no W iss ea s

itt id tio
is e D t w
t p or em ch
0

d th g. in t la
y - 0 + 3f(x) 4 = x + y
0x

an on in rs h
k g rn to rig
0 or in a uc y
w d le tr p
3f(x) 4 = x

er ld
e lu nt ns co

0x
th inc de f i es

Integrating this equation with respect to x,


of rk ( stu e o tat

1 2
ity o g us d S

f(x) = x + C
2
te is ss th ite
in f th se for Un

Setting C = 0 and substituting this result into Eq. 1,


gr w in e
th t o a ly by

1 2 1
y ar d le d

c = xy - y + x2
ro p an o te

2 2
st ny s d s ec
de f a rse de ot

1 2
s
ill o u vi pr

c = ( x - y2 + 2xy ) Ans.
2
w le co ro is
sa eir is p rk

Using the definition of velocity components with respect to the potential function,
th d wo

0f 0f
an his

= u;  = x - y
T

0x 0x
Integrating this equation with respect to x,
1 2
f = x - xy + f(y) (2)
2

Also,
0f 0 1 2
= v;  c x - xy + f(y) d = -(x + y)
0y 0y 2

0
-x + 3f(y) 4 = - x - y
0y
0
3f(y) 4 = - y
0y

763
© 2014 Pearson Education, Inc., Upper Saddle River, NJ. All rights reserved. This material is protected under all copyright laws as they currently
exist. No portion of this material may be reproduced, in any form or by any means, without permission in writing from the publisher.

7–37. Continued

Integrating this equation with respect to y


1 2
f(y) = y + C
2
Setting C = 0 and substituting this result into Eq. 2,
1 2 1
f = x - xy - y2
2 2
1 2
f = ( x - y2 - 2xy ) Ans.
2
Here
0v 0u
= - 1 and = -1
0x 0y
1 0v 0u 1
Since vz = a - b = 3 - 1 - ( - 1) 4 = 0, the flow is irrotational.

. We or
2 0x 0y 2

m W ina g

b)
ed e n
in
no W iss ea s

itt id tio
is e D t w
t p or em ch
d th g. in t la
an on in rs h
k g rn to rig
or in a uc y
w d le tr p

er ld
e lu nt ns co
th inc de f i es
of rk ( stu e o tat
ity o g us d S
te is ss th ite
in f th se for Un
gr w in e
th t o a ly by
y ar d le d
ro p an o te
st ny s d s ec
de f a rse de ot

s
ill o u vi pr
w le co ro is
sa eir is p rk
th d wo
an his

e
T

Ans:
1 2
c = ( x - y2 + 2xy )
2
1 2
f = ( x - y2 - 2xy )
2

764
© 2014 Pearson Education, Inc., Upper Saddle River, NJ. All rights reserved. This material is protected under all copyright laws as they currently
exist. No portion of this material may be reproduced, in any form or by any means, without permission in writing from the publisher.

7–38.  A fluid has velocity components u = (2y) ft>s and


v = (2x - 10) ft>s, where x and y are in feet. Determine
the stream and potential functions.

Solution
We consider ideal fluid flow.
0u 0v
Since the continuity equation + = 0 + 0 = 0 is satisfied, then the
0x 0y
establishment of stream function is possible.

Using the definition of velocity components with respect to stream function,


0c 0c
= u;  = 2y
0y 0y
Integrating this equation with respect to y,
c = y2 + f(x) (1)

Also,
0c 0
- = v;  - = 3 y2 + f(x) 4 = 2x - 10

. We or
0x 0x

m W ina g

b)
ed e n
in
no W iss ea s

itt id tio
is e D t w
0

t p or em ch
d th g. in t la
3f(x) 4 = 10 - 2x
0x

an on in rs h
k g rn to rig
Integrating this equation with respect to x, or in a uc y
w d le tr p

er ld
e lu nt ns co

f(x) = 10x - x2 + C
th inc de f i es

Setting C = 0, and substituting this result into Eq. 1,


of rk ( stu e o tat
ity o g us d S

c = y2 + 10x - x2
te is ss th ite
in f th se for Un

c = y2 - x2 + 10x Ans.
gr w in e
th t o a ly by

1 0v 0u 1
y ar d le d

Since vz = a - b = (2 - 2) = 0, the flow is irrotational. Therefore the


ro p an o te

2 0x 0y 2
st ny s d s ec

potential function exists.


de f a rse de ot

s
ill o u vi pr

Using the definition of velocity components with respect to potential function,


w le co ro is
sa eir is p rk

0f 0f
th d wo

= u;  = 2y
0x 0x
an his

Integrating this equation with respect to x,


T

f = 2xy + f(y) (1)

Also,
0f 0
= v;  = 32xy + f(y) 4 = 2x - 10
0y 0y
0
2x + 3f(y) 4 = 2x - 10
0y
0
3f(y) 4 = - 10
0y
Integrating this equation with respect to y,
f(y) = - 10y + C
Setting C = 0, and substituting this result into Eq. 1,
f = 2xy - 10y
f = 2y(x - 5) Ans.
Ans:
c = y2 - x2 + 10x
f = 2y(x - 5)

765
© 2014 Pearson Education, Inc., Upper Saddle River, NJ. All rights reserved. This material is protected under all copyright laws as they currently
exist. No portion of this material may be reproduced, in any form or by any means, without permission in writing from the publisher.

7–39.  A fluid has velocity components u = (x - 2y) ft>s


and v = - (y + 2x) ft>s, where x and y are in feet.
Determine the stream and potential functions.

Solution
We consider ideal fluid flow.
0u 0v
Since the continuity equation + = 1 + ( - 1) = 0 is satisfied, then the
0x 0y
establishment of the stream function is possible using the velocity components,
0c 0c
= u;  = x - 2y
0y 0y
Integrating this equation with respect to y,

c = xy - y2 + f(x) (1)
Also,
0c 0
- = v;  - 3 xy - y2 + f(x) 4 = -y - 2x
0x 0x

. We or
0

m W ina g

b)
ed e n
-y - 3f(x) 4 = - y - 2x

in
no W iss ea s

itt id tio
is e D t w
0x

t p or em ch
d th g. in t la
0

an on in rs h
3f(x) 4 = 2x
k g rn to rig
0x or in a uc y
w d le tr p

er ld
Integrating this equation with respect to x,
e lu nt ns co
th inc de f i es

f(x) = x2 + C
of rk ( stu e o tat
ity o g us d S

Setting C = 0 and substituting this result into Eq. 1,


te is ss th ite

c = x2 - y2 + xy Ans.
in f th se for Un
gr w in e

1 0v 0u 1
th t o a ly by

Since vz = a - b = 3 - 2 - ( - 2) 4 = 0, the flow is irrotational. Therefore,


2 0x 0y 2
y ar d le d
ro p an o te
st ny s d s ec

the potential function exists.


de f a rse de ot

Using the definition of velocity components with respect to potential function,


s
ill o u vi pr
w le co ro is

0f 0f
sa eir is p rk

= u;  = x - 2y
th d wo

0x 0x
Integrating this equation with respect to x
an his

e
T

1 2
f = x - 2xy + f(y) (1)
2

Also,
0f 0 1
= v;  = c x2 - 2xy + f(y) d = -y - 2x
0y 0y 2
0
- 2x + 3f(y) 4 = - y - 2x
0y
0
3f(y) 4 = - y
0y
Integrating this equation with respect to y
1
f(y) = - y2 + C
2
Setting C = 0 and substituting this result into Eq. 1,
1 2 1
f = x - 2xy - y2 Ans:
2 2
c = x2 - y2 + xy
1 2
f = ( x - y2 ) - 2xy Ans. 1 2
2 f = ( x - y2 ) - 2xy
2

766
© 2014 Pearson Education, Inc., Upper Saddle River, NJ. All rights reserved. This material is protected under all copyright laws as they currently
exist. No portion of this material may be reproduced, in any form or by any means, without permission in writing from the publisher.

*7–40.  A fluid has velocity components


u = 2 1 x2 - y2 2 m>s and v = ( -4xy) m>s, where x and y
are in meters. Determine the stream function. Also show
that the potential function exists, and find this function. Plot
the streamlines and equipotential lines that pass through
point (1 m, 2 m).

Solution
0u 0v
Since the continuity equation + = 4x + ( -4x) = 0 is satisfied, the stream
0x 0y
function can be established.

Using the velocity components


0c 0c

. We or
= u;  = 2x2 - 2y2

m W ina g

b)
ed e n
0y 0y

in
no W iss ea s

itt id tio
is e D t w
t p or em ch
d th g. in t la
Integrating this equation with respect to y,

an on in rs h
k g rn to rig
2 3 or in a uc y
c = 2x2y - y + f(x) (1)
w d le tr p

er ld
3
e lu nt ns co

Also,
th inc de f i es
of rk ( stu e o tat

0c 0 2
- = v;  - 3 2x2y - y3 + f(x) 4 = -4xy
ity o g us d S

0x 0x 3
te is ss th ite

0
in f th se for Un

4xy + 3f(x) 4 = 4xy


gr w in e

0x
th t o a ly by

0
y ar d le d
ro p an o te

3f(x) 4 = 0
0x
st ny s d s ec
de f a rse de ot

Integrating this equation with respect to x,


s
ill o u vi pr
w le co ro is

f(x) = C
sa eir is p rk
th d wo

Setting C = 0 and substituting this result into Eq. 1,


an his

2 3
c = 2x2y -
T

y
3
2
c = y ( 3x2 - y2 ) Ans.
3

1 0v 0u 1
Since vz = a - b = 3 - 4y - ( - 4y) 4 = 0, the flow is irrotational. Thus, the
2 0x 0y 2
potential function exists. Using the velocity components,
0f 0f
= u;  = 2x2 - 2y2
0x 0x

Integrating this equation with respect to x


2 3
f = x - 2xy2 + f(y) (2)
3

767
© 2014 Pearson Education, Inc., Upper Saddle River, NJ. All rights reserved. This material is protected under all copyright laws as they currently
exist. No portion of this material may be reproduced, in any form or by any means, without permission in writing from the publisher.

*7–40. Continued

Also,
0f 0 2 3
= v;  c x - 2xy2 + f(y) d = -4xy
0y 0y 3
0
- 4xy + 3f(y) 4 = -4xy

. We or
0y

m W ina g

b)
ed e n
in
no W iss ea s

itt id tio
is e D t w
0

t p or em ch
d th g. in t la
3f(y) 4 = 0
0y

an on in rs h
k g rn to rig
Integrating this equation with respect to y, or in a uc y
w d le tr p

er ld
e lu nt ns co

f(y) = C
th inc de f i es
of rk ( stu e o tat

Setting C = 0 and substituting this result into Eq. 2,


ity o g us d S

2 3
f = x - 2xy2
te is ss th ite

3
in f th se for Un
gr w in e

2
th t o a ly by

x ( x2 - 3y2 )
f = Ans.
3
y ar d le d
ro p an o te

For stream function and potential functions passing through point (1 m, 2 m),
st ny s d s ec
de f a rse de ot

2 4
s

y(m)
(2) 3 3(1)2 - 22 4 = -
ill o u vi pr

c =
3 3
w le co ro is

7
sa eir is p rk

2 22
th d wo

f = (1) 3 12 - 3 ( 22 ) 4 = -
3 3 6
an his

Thus, the streamline is


T

4 2 5
- = y ( 3x2 - y2 )
3 3 4
3
y - 2
x2 = 3
3y
and the equipotential line is 2
22 2
- = x ( x2 - 3y2 ) 1
3 3
x(m)
x3 + 11
y2 = –6 –5 –4 –3 –2 –1 0 1 2 3 4 5 6
3x
For the streamline
y(m) 1.26 2 3 4 5 6
x(m) 0 {1 {1.67 {2.27 {2.86 {3.45

For the equipotential line


x(m) 0 1 1.77 3 4 5 6 0.25 0.50
y(m) ∞ {2 {1.77 {2.05 {2.50 {3.01 {3.55 {3.83 {2.72

The plot of these two functions is shown in Fig. a

768
© 2014 Pearson Education, Inc., Upper Saddle River, NJ. All rights reserved. This material is protected under all copyright laws as they currently
exist. No portion of this material may be reproduced, in any form or by any means, without permission in writing from the publisher.

7–41.  If the potential function for a two-dimensional flow


is f = (xy) m2 >s, where x and y are in meters, determine
the stream function and plot the streamline that passes
through the point (1 m, 2 m). What are the velocity and
acceleration of fluid particles that pass through this point?

Solution
We consider ideal fluid flow.
0f 0
u = = (xy) = y
0x 0x
0f 0
v = = (xy) = x
0y 0y

. We or
0c 0c

m W ina g

b)
ed e n
u = ;  y =

in
no W iss ea s

itt id tio
is e D t w
0y 0y

t p or em ch
d th g. in t la
y(m)

an on in rs h
Integrating with respect to y,

k g rn to rig
y2
or in a uc y
w d le tr p

er ld
e lu nt ns co

c = + f(x)
2
th inc de f i es

ψ = 1.5
Substituting this result into,
of rk ( stu e o tat
ity o g us d S

0c 0 y2
v = - ;  x = - c + f(x) d
te is ss th ite

0x 0x 2
in f th se for Un
gr w in e

0
th t o a ly by

x = -0 - 3f(x) 4
0x
y ar d le d

3
ro p an o te
st ny s d s ec

0 x(m)
3f(x) 4 = -x
de f a rse de ot

0x
s
ill o u vi pr
w le co ro is

Integrating with respect to x,


sa eir is p rk

(a)
th d wo

x2
f(x) = - + C
an his

2
T

Setting C = 0,
y2 x2
c = + a- b
2 2
1 2
c = ( y - x2 ) Ans.
2
When x = 1 m, and y = 2 m. Then,
1 2
c = ( 2 - 12 ) = 1.5
2

For the streamline defined by c = 1.5, its equation is


1 2
( y - x2 ) = 1.5
2
y2 = x2 + 3

y = 2x2 + 3

The plot of this streamline is shown in Fig. a. Ans:


1 2
c = ( y - x2 )
2

769
© 2014 Pearson Education, Inc., Upper Saddle River, NJ. All rights reserved. This material is protected under all copyright laws as they currently
exist. No portion of this material may be reproduced, in any form or by any means, without permission in writing from the publisher.

7–41. Continued

At c = 1 m, y = 2 m
u = y = 2 m>s

v = x = 1 m>s
V = 2 1 2 m>s 2 2 + 1 1 m>s 2 2 = 2.24 m>s  Ans.

. We or
m W ina g

b)
ed e n
0u 0u 0u

in
no W iss ea s

itt id tio
ax = + u + v = 0 + (2)(0) + 1(1) = 1 m>s2

is e D t w
t p or em ch
0t 0x 0y

d th g. in t la
an on in rs h
k g rn to rig
0v 0v 0v
ay = + u + v = 0 + 2(1) + 1(0) = 2 m>s2 or in a uc y
0t 0x 0y
w d le tr p

er ld
e lu nt ns co
th inc de f i es

a = 2 1 1 m>s2 2 + 1 2 m>s2 2 2 = 2.24 m>s2 Ans.


of rk ( stu e o tat
ity o g us d S
te is ss th ite
in f th se for Un
gr w in e
th t o a ly by
y ar d le d
ro p an o te
st ny s d s ec
de f a rse de ot

s
ill o u vi pr
w le co ro is
sa eir is p rk
th d wo
an his

e
T

770
© 2014 Pearson Education, Inc., Upper Saddle River, NJ. All rights reserved. This material is protected under all copyright laws as they currently
exist. No portion of this material may be reproduced, in any form or by any means, without permission in writing from the publisher.

7–42.  Determine the potential function for the two- y


dimensional flow field if V0 and u are known.

V0

Solution

. We or
We consider ideal fluid flow.

m W ina g

b)
ed e n
in
no W iss ea s
The velocity components are

itt id tio
is e D t w
t p or em ch
d th g. in t la
u = V0 sin u0   v = -V0 cos u0

an on in rs h
k g rn to rig
1 0v 0u 1 or in a uc y
w d le tr p
Since vz = a - b = (0 - 0) = 0, the flow is indeed irrotational. Thus, the

er ld
e lu nt ns co

2 0x 0y 2
th inc de f i es

potential function exists.


of rk ( stu e o tat

Using the velocity components,


ity o g us d S
te is ss th ite

0f 0f
in f th se for Un

= u;    = V0 sin u0
0x 0x
gr w in e
th t o a ly by

Integrating this equation with respect to x,


y ar d le d
ro p an o te
st ny s d s ec

f = V0 sin u0 x + f(y) (1)


de f a rse de ot

s
ill o u vi pr

Also,
w le co ro is

0f 0
= v;   3 ( V0 sin u0 ) x + f(y) 4 = -V0 cos u0
sa eir is p rk

0y 0y
th d wo
an his

0
3f(y) 4 = -V0 cos u0
T

0y

Integrating this equation with respect to y

f(y) = - ( V0 cos u0 ) y + C
Setting C = 0, and substituting this result into Eq. 1,

f = ( V0 sin u0 ) x - ( V0 cos u0 ) y

f = V0 3 ( sin u0 ) x - ( cos u0 ) y 4 Ans.

Ans:
f = V0 3 ( sin u0 ) x - ( cos u0 ) y 4

771
© 2014 Pearson Education, Inc., Upper Saddle River, NJ. All rights reserved. This material is protected under all copyright laws as they currently
exist. No portion of this material may be reproduced, in any form or by any means, without permission in writing from the publisher.

7–43.  The potential function for a flow is


f = 1 x2 - y2 2 ft 2 >s, where x and y are in feet. Determine
the magnitude of the velocity of fluid particles at point (3 ft,
1 ft). Show that continuity is satisfied, and find the
streamline that passes through this point.

Solution
We consider ideal fluid flow.
0f 0
u = = ( x2 - y2 ) = (2x) ft>s
0x 0x
0f 0
v = = ( x2 - y2 ) = ( - 2y) ft>s
0y 0y

. We or
Thus, at x = 3 ft, y = 1 ft,

m W ina g

b)
ed e n
in
no W iss ea s

itt id tio
is e D t w
t p or em ch
u = 2(3) = 6 ft>s

d th g. in t la
an on in rs h
k g rn to rig
v = - 2(1) = - 2 ft>s
or in a uc y
w d le tr p

er ld
Then, the magnitude of the flow velocity is
e lu nt ns co
th inc de f i es

V = 2u2 + v2 = 2 ( 6 ft>s ) 2 + ( - 2 ft>s ) 2 = 6.32 ft>s Ans.


of rk ( stu e o tat
ity o g us d S

Applying
te is ss th ite

0c 0c
in f th se for Un

u = ;  2x =
gr w in e

0y 0y
th t o a ly by

Integrating with respect to y,


y ar d le d
ro p an o te
st ny s d s ec

c = 2xy + f(x)
de f a rse de ot

s
ill o u vi pr

Substituting this result into the second of Eq. (8–8),


w le co ro is
sa eir is p rk

0c 0
th d wo

v = - ;   - 2y = - 32xy + f(x) 4


0x 0x
an his

0
T

2y = 2y + 3f(x) 4
0x
0
3f(x) 4 = 0
0x
Integrating with respect to x,
f(x) = C

Setting C = 0, then

c = 2xy

At x = 3 ft, y = 1 ft,

c = 2(3)(1) = 6

So the streamline through (3 ft, 1 ft) is

6 = 2xy

3
y = Ans.
x

772
© 2014 Pearson Education, Inc., Upper Saddle River, NJ. All rights reserved. This material is protected under all copyright laws as they currently
exist. No portion of this material may be reproduced, in any form or by any means, without permission in writing from the publisher.

7–43. Continued

Here,

0u 0
= (2x) = 2 ft>s
0x 0x
0v 0
= ( -2y) = - 2 ft>s
0y 0y

. We or
Then,

m W ina g

b)
ed e n
in
no W iss ea s

itt id tio
is e D t w
t p or em ch
0u 0v

d th g. in t la
+ = 2 + ( - 2) = 0
0x 0y

an on in rs h
k g rn to rig
or in a uc y
w d le tr p
Thus, the flow field satisfies the continuity condition as required.

er ld
e lu nt ns co
th inc de f i es
of rk ( stu e o tat
ity o g us d S
te is ss th ite
in f th se for Un
gr w in e
th t o a ly by
y ar d le d
ro p an o te
st ny s d s ec
de f a rse de ot

s
ill o u vi pr
w le co ro is
sa eir is p rk
th d wo
an his

e
T

Ans:
V = 6.32 ft>s
3
y = x

773
© 2014 Pearson Education, Inc., Upper Saddle River, NJ. All rights reserved. This material is protected under all copyright laws as they currently
exist. No portion of this material may be reproduced, in any form or by any means, without permission in writing from the publisher.

*7–44.  A fluid has velocity components of u = (10xy) m>s


and v = 5 1 x2 - y2 2 m>s, where x and y are in meters.
Determine the stream function, and show that the continuity
condition is satisfied and that the flow is irrotational. Plot
the streamlines for c0 = 0, c1 = 1 m2 >s, and c2 = 2 m2 >s.

Solution
We consider ideal fluid flow.
0c 0c
u = ;  10xy =
0y 0y
Integrating with respect to y,

c = 5xy2 + f(x)

. We or
Substituting this result into

m W ina g

b)
ed e n
in
no W iss ea s

itt id tio
is e D t w
t p or em ch
0c

d th g. in t la
0
v = - ;  5 ( x2 - y2 ) = - 3 5xy2 + f(x)4

an on in rs h
0x 0x

k g rn to rig
0 or in a uc y
5x2 - 5y2 = - 5y2 - 3f(x)4
w d le tr p

er ld
e lu nt ns co

0x
th inc de f i es

0
3f(x)4 = - 5x2
of rk ( stu e o tat

0x
ity o g us d S

Integrating with respect to x,


te is ss th ite
in f th se for Un

5
f(x) = - x3 + C
gr w in e
th t o a ly by

3
Setting C = 0, then
y ar d le d
ro p an o te
st ny s d s ec

5
de f a rse de ot

c = 5xy2 + a - x3 b
s
ill o u vi pr

3
w le co ro is

5
= x ( 3y - x2 )
2
Ans.
sa eir is p rk

3
th d wo

Here,
an his

e
T

0u 0
= (10xy) = (10y) s-1
0x 0x
0v 0
= 3 5 ( x2 - y2 ) 4 = ( - 10y) s-1
0y 0y
0v 0
= 3 5 ( x2 - y2 ) 4 = (10x) s-1
0x 0x
0u 0
= (10xy) = (10x) s-1
0y 0y
Then,

0u 0v
+ = 10y + ( - 10y) = 0
0x 0y
The flow field satisfies the continuity condition. Applying,

1 0v 0u 1
vz = a - b = (10x - 10x) = 0
2 0x 0y 2

774
© 2014 Pearson Education, Inc., Upper Saddle River, NJ. All rights reserved. This material is protected under all copyright laws as they currently
exist. No portion of this material may be reproduced, in any form or by any means, without permission in writing from the publisher.

7–44. Continued

The flow field is irrotational since vz = 0.

When c = 0,

5
x ( 3y2 - x2 ) = 0
3
1
y = { x

. We or
23

m W ina g

b)
ed e n
in
no W iss ea s
or

itt id tio
is e D t w
t p or em ch
d th g. in t la
x = 0

an on in rs h
k g rn to rig
When c = 1, or in a uc y
w d le tr p

er ld
e lu nt ns co

5
x ( 3y2 - x2 ) = 1
th inc de f i es

3
of rk ( stu e o tat

1 x2
ity o g us d S

y= { +
A 5x 3
te is ss th ite
in f th se for Un

When c = 2,
gr w in e
th t o a ly by

5
x ( 3y2 - x2 ) = 2
y ar d le d

3
ro p an o te
st ny s d s ec

2 x2
y= { +
de f a rse de ot

A 5x 3
s
ill o u vi pr
w le co ro is
sa eir is p rk
th d wo

Ç=2
y
an his

Ç=1
T

Ç=0

30
x

775
© 2014 Pearson Education, Inc., Upper Saddle River, NJ. All rights reserved. This material is protected under all copyright laws as they currently
exist. No portion of this material may be reproduced, in any form or by any means, without permission in writing from the publisher.

7–45.  A fluid has velocity components of u = 1y2 - x22 m>s


and v = (2xy) m>s, where x and y are in meters. If the
pressure at point A (3 m, 2 m) is 600 kPa, determine the
pressure at point B (1 m, 3 m). Also what is the potential
function for the flow? Take g = 8 kN>m3.

Solution
We consider ideal fluid flow.
Applying
0f 0f
u = ;   y2 - x2 =
0x 0x

Integrating with respect to x,

. We or
x3

m W ina g

b)
ed e n
f = xy2 -

in
+ f(y)

no W iss ea s

itt id tio
is e D t w
3

t p or em ch
d th g. in t la
an on in rs h
Substituting this result into,

k g rn to rig
or in a uc y
0f x3
w d le tr p
0

er ld
e lu nt ns co

v = ; 2xy = c xy2 - + f(y) d


0y 0y 3
th inc de f i es
of rk ( stu e o tat

0
ity o g us d S

2xy = 2xy - 0 + 3f(y)4


0y
te is ss th ite

0
in f th se for Un

3f(y)4 = 0
0y
gr w in e
th t o a ly by

Integrating with respect to y,


y ar d le d
ro p an o te
st ny s d s ec

f(y) = C
de f a rse de ot

s
ill o u vi pr

Setting C = 0, we have
w le co ro is

x3
sa eir is p rk

f = xy2 - Ans.
th d wo

3
an his

Since the potential function can be established, the flow is irrotational. Thus, the
T

Bernoulli equation can be applied from point A to B.  The x and y components of


the velocity at these points are

u A = ( 22 - 32 ) m>s = - 5 m>s  vA = 32(3)(2) 4 m>s = 12 m>s

u B = ( 32 - 12 ) m>s = 8 m>s   vB = 32(1)(3) 4 m>s = 6 m>s

Thus, the magnitude of the velocity at these two points is

VA = 2uA2 + vA2 = 2 ( - 5 m>s ) 2 + ( 12 m>s ) 2 = 13 m>s

VB = 2uB2 + vB2 = 2 ( 8 m>s2 ) + ( 6 m>s2 ) = 10 m>s

Applying the Bernoulli equation for ideal fluid from A to B,


pB VB2 pA VA2
+ = +
g 2g g 2g
N
600 ( 103 )
pB (10 m>s)2 m2 (13 m>s)2
+ = +
8(103) N>m3 2 ( 9.81 m>s2 ) 8 ( 103 ) N>m3 2 ( 9.81 m>s2 ) Ans:
x3
N f = xy2 -
pB = 628.13 ( 10 ) 2 = 628 kPa
3
Ans. 3
m
pB = 628 kPa

776
© 2014 Pearson Education, Inc., Upper Saddle River, NJ. All rights reserved. This material is protected under all copyright laws as they currently
exist. No portion of this material may be reproduced, in any form or by any means, without permission in writing from the publisher.

7–46.  The potential function for a horizontal flow is


f = 1 x3 - 5xy2 2 m2 >s, where x and y are in meters.
Determine the magnitude of the velocity at point A (5 m,
2 m). What is the difference in pressure between this point
and the origin? Take r = 925 kg>m3.

Solution
We consider ideal fluid flow.
Since the flow is described by the potential function, the flow is definitely irrotational.
Therefore, the Bernoulli equation can be applied between any two points.
0f 0
u = = ( x3 - 5xy2 ) = 3x2 - 5y2
0x 0x
0f 0

. We or
v = = ( x3 - 5xy2 ) = - 10xy

m W ina g

b)
ed e n
0y 0y

in
no W iss ea s

itt id tio
is e D t w
t p or em ch
d th g. in t la
At point A, x = 5 m, y = 2 m. Thus,

an on in rs h
k g rn to rig
u A = 3 ( 52 ) - 5 ( 22 ) = 55 m>s   vA = -10(5)(2) = -100 m>s
or in a uc y
w d le tr p

er ld
e lu nt ns co

At the origin O, x = 0 and y = 0. Thus,


th inc de f i es
of rk ( stu e o tat

u 0 = 3 ( 02 ) - 5 ( 02 ) = 0      v0 = -10(0)(0) = 0


ity o g us d S

The magnitude of the velocity at A and O is


te is ss th ite
in f th se for Un

VA = 2u A2 + vA2 = 2 ( 55 m>s ) 2 + ( - 100 m>s ) 2 = 114.13 m>s = 114 m>s Ans.


gr w in e
th t o a ly by

V0 = 0
y ar d le d
ro p an o te
st ny s d s ec

Since the flow occurs in the horizontal plane, no change in elevation takes place.
de f a rse de ot

Thus, the elevation term can be excluded. Applying the Bernoulli equation for an
s
ill o u vi pr

ideal fluid from O to A,


w le co ro is
sa eir is p rk

pA VA2 pO VO2
th d wo

+ = +
2 2
an his

r r
e
T

r
pO - pA = ( VA2 - VO2 )
2

925 kg>m3
= ° ¢ 3 ( 114.13 m>s ) 2 - 02 4
2

= 6.024 ( 106 ) Pa = 6.02 MPa Ans.

Ans:
VA = 114 m>s
pO - pA = 6.02 MPa

777
© 2014 Pearson Education, Inc., Upper Saddle River, NJ. All rights reserved. This material is protected under all copyright laws as they currently
exist. No portion of this material may be reproduced, in any form or by any means, without permission in writing from the publisher.

7–47.  A fluid has velocity components of u = (10xy) m>s


and v = 5 1 x2 - y2 2 m>s, where x and y are in meters.
Determine the potential function, and show that the
continuity condition is satisfied and that the flow is
irrotational.

Solution
We consider ideal fluid flow.
0f 0f
u = ;  10xy =
0x 0x

Integrating with respect to x,

f = 5x2y + f(y)

Substituting this result into the second of Eq. (8–12),


0f 0
v = ;  5 ( x2 - y2 ) = 3 5x2y + f(y) 4
0y 0y

. We or
m W ina g
0

b)
ed e n
in
no W iss ea s
5x2 - 5y2 = 5x2 +

itt id tio
3f(y) 4

is e D t w
t p or em ch
0y

d th g. in t la
an on in rs h
k g rn to rig
0
3f(y) 4 = - 5y2 or in a uc y
0y
w d le tr p

er ld
e lu nt ns co
th inc de f i es

Integrating with respect to x,


of rk ( stu e o tat

5
ity o g us d S

f(y) = - y3 + C
3
te is ss th ite
in f th se for Un

Setting C = 0, then
gr w in e
th t o a ly by

5
y ar d le d

f = 5x2y + a - y3 b
ro p an o te

3
st ny s d s ec
de f a rse de ot

s
ill o u vi pr

5
= y ( 3x2 - y2 ) Ans.
w le co ro is

3
sa eir is p rk
th d wo

Here,
an his

0u 0
T

= (10xy) = (10y) s-1


0x 0x

0v 0
= 3 5 ( x2 - y2 ) 4 = ( - 10y) s-1
0y 0y

0v 0
= 3 5 ( x2 - y2 ) 4 = (10x) s-1
0x 0x

0u 0
= (10xy) = (10x) s-1
0y 0y

Then,

0u 0v
+ = 10y + ( - 10y) = 0
0x 0y

The flow field satisfies the continuity condition. Applying,


Ans:
1 0v 0u 1
vz = a - b = (10x - 10x) = 0 5
2 0x 0y 2 f = y ( 3x2 - y2 )
3
The flow field is irrotational since vz = 0. 1
c = ( y2 - x2 )
2

778
© 2014 Pearson Education, Inc., Upper Saddle River, NJ. All rights reserved. This material is protected under all copyright laws as they currently
exist. No portion of this material may be reproduced, in any form or by any means, without permission in writing from the publisher.

*7–48.  A velocity field is defined as u = 2 1 x2 + y2 2 ft>s, y


v = (- 4xy) ft>s. Determine the stream function and the
circulation around the rectangle shown. Plot the streamlines
for c0 = 0, c1 = 1 ft 2 >s, and c2 = 2 ft 2 >s.

0.6 ft

0.5 ft

Solution

. We or
We consider ideal fluid flow.

m W ina g

b)
ed e n
in
no W iss ea s

itt id tio
is e D t w
0u 0v

t p or em ch
d th g. in t la
Since the continuity equation = = 4x + ( -4x) = 0 is satisfied, the stream
0x 0y

an on in rs h
k g rn to rig
function can be established. Using the definition of the velocity components, with
respect to stream function,
or in a uc y
w d le tr p

er ld
e lu nt ns co
th inc de f i es

0c 0c
of rk ( stu e o tat

= u;   = 2 ( x2 + y2 )
0y 0y
ity o g us d S
te is ss th ite

Integrating this equation with respect to y,


in f th se for Un
gr w in e

1 3
th t o a ly by

c = 2 ax2y + y b + f(x) (1)


3
y ar d le d
ro p an o te
st ny s d s ec
de f a rse de ot

Also,
s
ill o u vi pr
w le co ro is

0c 0c
sa eir is p rk

-
= v;   - = - 4xy
th d wo

0x 0x
an his

0 1
T

- c 2 ax2y + y3 b + f(x) d = -4xy


0x 3

0
4xy + 3f(x) 4 = 4xy
0x

0
3 f(x) 4 = 0
0x

Integrating this equation with respect to x,


f(x) = C

Substituting this result into Eq. (1),

1 3
c = 2 ax2y + y b + C
3

C is an arbitary constant. If we set it equal to zero then the stream function can be
expressed as

1 2
c = 2y ax2 + y b Ans.
3

779
© 2014 Pearson Education, Inc., Upper Saddle River, NJ. All rights reserved. This material is protected under all copyright laws as they currently
exist. No portion of this material may be reproduced, in any form or by any means, without permission in writing from the publisher.

*7–48. Continued

y2 y2
For c = 0,        0 = 2y°x2 + ¢  since x2 + ≠ 0, then
3 3

y = 0

For c = 1 ft 2 >s,
y2
1 = 2y °x2 + ¢
3

3 - 2y3
x2 =   0 6 y 6 1.145
6y

. We or
m W ina g

b)
ed e n
in
no W iss ea s
For c = 2 ft 2 >s,

itt id tio
is e D t w
t p or em ch
d th g. in t la
y2

an on in rs h
2 = 2y°x2 + ¢
k g rn to rig
3 or in a uc y
w d le tr p

er ld
e lu nt ns co

6 - 2y3
th inc de f i es

x2 =   0 6 y 6 1.442
of rk ( stu e o tat

6y
ity o g us d S

The plot of these streamlines are shown in Fig. a.


te is ss th ite
in f th se for Un

For c = 1 ft 2 >s For c = 2 ft 2 >s


gr w in e
th t o a ly by

y(ft) 0 0.25 0.50 0.75 1.00 y(ft) 0 0.25 0.50 0.75 1.00 1.25
y ar d le d
ro p an o te

x(ft) {∞ { 1.407 { 0.957 { 0.692 { 0.408 x(ft) {∞ { 1.995 { 1.384 { 1.070 { 0.816 { 0.528
st ny s d s ec
de f a rse de ot

y(ft) 1.145 y(ft) 1.442


s
ill o u vi pr
w le co ro is

x(ft) 0 x(ft) 0
sa eir is p rk
th d wo

y(ft)
an his

e
T

1.50
ψ = 2 ft2/s

1.25

1.0

0.75

ψ = 1 ft2/s
0.5

0.25
ψ =0

x(ft)
–2.0 –1.75 –1.5 –1.25 –1.0 –0.75 –0.5 –0.25 0 0.25 0.5 0.75 1.0 1.25 1.5 1.75 2.0

(a)

780
© 2014 Pearson Education, Inc., Upper Saddle River, NJ. All rights reserved. This material is protected under all copyright laws as they currently
exist. No portion of this material may be reproduced, in any form or by any means, without permission in writing from the publisher.

*7–48. Continued

The circulation can be determined using

V # ds
C
Γ =

0.5 ft 0.6 ft 0.5 ft 0.5 ft

L0 L0 L0 L0
= udx + vdy + u( - dx) + v( -dy)

. We or
0.5 ft 0.6 ft 0.5 ft

L0 L0 L0

m W ina g
2 ( x2 + 0 ) dx + 2 ( x2 + 0.62 ) ( - dx)

b)
ed e n
= - 4(0.5)ydy +

in
no W iss ea s

itt id tio
is e D t w
t p or em ch
d th g. in t la
an on in rs h
0.6 ft

L0 k g rn to rig
+ - 4(0)y( - dy)
or in a uc y
w d le tr p

er ld
e lu nt ns co
th inc de f i es

2 3 0.5 ft 0.6 ft
2 0.5 ft
of rk ( stu e o tat

= x ` - y2 ` - a x3 + 0.72xb ` + 0
3 0 3
ity o g us d S

0 0
te is ss th ite

= - 0.72 ft 2 >s Ans.


in f th se for Un
gr w in e
th t o a ly by
y ar d le d
ro p an o te
st ny s d s ec
de f a rse de ot

s
ill o u vi pr
w le co ro is
sa eir is p rk
th d wo
an his

e
T

781
© 2014 Pearson Education, Inc., Upper Saddle River, NJ. All rights reserved. This material is protected under all copyright laws as they currently
exist. No portion of this material may be reproduced, in any form or by any means, without permission in writing from the publisher.

7–49.  If the potential function for a two-dimensional flow


is f = (xy) m2 >s, where x and y are in meters, determine
the stream function, and plot the streamline that passes
through the point (1 m, 2 m). What are the x and y
components of the velocity and acceleration of fluid
particles that pass through this point?

Solution
We consider ideal fluid flow.
Using the velocity components,

0f 0f
u = = y  v = = x
0x 0y

1 0v 0u 1
Since vz = a - b = (1 - 1) = 0, the flow is indeed irrotational. Also,

. We or
2 0x 0y 2

m W ina g
0u 0v

b)
ed e n
in
no W iss ea s
since the continuity equation + = 0 + 0 = 0 is satisfied, the establishment

itt id tio
is e D t w
t p or em ch
0x 0y

d th g. in t la
of a stream function is possible,

an on in rs h
k g rn to rig
or in a uc y
w d le tr p
0c 0c

er ld
e lu nt ns co

= u;   = y
0y 0y
th inc de f i es
of rk ( stu e o tat

Integrating this equation with respect to y,


ity o g us d S
te is ss th ite

1 2
c = y + f(x) (1)
in f th se for Un

2
gr w in e
th t o a ly by

Also,
y ar d le d
ro p an o te

0c 0 1
st ny s d s ec

- = v;   - c y2 + f(x) d = x
0x 0x 2
de f a rse de ot

s
ill o u vi pr
w le co ro is

0
3f(x) 4 = -x
sa eir is p rk

0x
th d wo
an his

Integrating this equation with respect to x


T

1
f(x) = - x2 + C
2

Setting C = 0 and substituting this result into Eq. 1,


1 2 1
c = y - x2
2 2

1 2
c = ( y - x2 ) Ans.
2

For the streamline passing through point (1 m, 2 m)


1 2 3
c = ( 2 - 12 ) =
2 2

Thus,

3 1
= ( y2 - x2 )
2 2

y2 = x2 + 3

782
© 2014 Pearson Education, Inc., Upper Saddle River, NJ. All rights reserved. This material is protected under all copyright laws as they currently
exist. No portion of this material may be reproduced, in any form or by any means, without permission in writing from the publisher.

7–49. Continued

The plot of this streamline is shown in Fig. a

x(m) 0 1 2 3 4 5 6
y(m) 1.73 2 2.65 3.46 4.36 5.29 6.24

y(m)

. We or
m W ina g

b)
ed e n
in
no W iss ea s

itt id tio
is e D t w
6

t p or em ch
d th g. in t la
an on in rs h
k g rn to rig
5
or in a uc y
w d le tr p

er ld
e lu nt ns co

4
th inc de f i es
of rk ( stu e o tat
ity o g us d S

3
te is ss th ite
in f th se for Un

2
gr w in e
th t o a ly by
y ar d le d

1
ro p an o te
st ny s d s ec
de f a rse de ot

s
ill o u vi pr

x(m)
w le co ro is

0 1 2 3 4 5 6
sa eir is p rk
th d wo

(a)
an his

e
T

At point (1 m, 2 m), the velocity components are


u = 2 m>s  v = 1 m>s Ans.

The acceleration components are


0u 0u 0u
ax = + u + v
0t 0x 0y

= 0 + y(0) + x(1) = x = 1 m>s2 Ans.

0v 0v 0v
ay = + u + v
0t 0x 0y

= 0 + y(1) + x(0) = y = 2 m>s2 Ans.

Ans:
u = 2 m>s, v = 1 m>s
ax = 1 m>s2
ay = 2 m>s2

783
© 2014 Pearson Education, Inc., Upper Saddle River, NJ. All rights reserved. This material is protected under all copyright laws as they currently
exist. No portion of this material may be reproduced, in any form or by any means, without permission in writing from the publisher.

7–50.  A two-dimensional flow is described by the potential


function f = 1 8x2 - 8y2 2 m2 >s, where x and y are in
meters. Show that the continuity condition is satisfied, and
determine if the flow is rotational or irrotational. Also,
establish the stream function for this flow, and plot the
streamline that passes through point (1 m, 0.5 m).

Solution
We consider ideal fluid flow.
Here
0f 02f
= 16x     2 = 16
0x 0x
0f 02f
= - 16y    = -16
0y 0y2

. We or
m W ina g

b)
ed e n
in
02f 02f

no W iss ea s

itt id tio
is e D t w
t p or em ch
Since + = 16+ ( -16) = 0, the potential function f satisfies the continuity

d th g. in t la
2
0x 0y2

an on in rs h
condition.

k g rn to rig
The velocity components can be determined using or in a uc y
w d le tr p

er ld
e lu nt ns co

0f
th inc de f i es

u = ;  u = (16x) m>s


0x
of rk ( stu e o tat
ity o g us d S

0f
v = ;  v = ( - 16y) m>s
te is ss th ite

0y
in f th se for Un

Then
gr w in e
th t o a ly by

0v 0u
y ar d le d

= 0   = 0
ro p an o te

0x 0y
st ny s d s ec
de f a rse de ot

Thus
s
ill o u vi pr
w le co ro is

1 0v 0u
vz = a - b = 0
sa eir is p rk

2 0x 0x
th d wo

Since vz = 0, the flow is indeed irrotational since all flows that can be described by
an his

a potential function are irrotational. Using the definition of velocity components


T

with respect to the stream function,


0c 0c
= u;   = 16x
0y 0y
Integrating this equation with respect to y,
c = 16xy + f(x) (1)
Also,
0c 0
- = v;   - 316xy + f(x) 4 = -16y
0x 0x
0
      - 16y
  - 3f(x) 4 = -16y
0x
0
          3f(x) 4 = 0
0x

784
© 2014 Pearson Education, Inc., Upper Saddle River, NJ. All rights reserved. This material is protected under all copyright laws as they currently
exist. No portion of this material may be reproduced, in any form or by any means, without permission in writing from the publisher.

7–50. Continued

Integrating this equation with respect to x


f(x) = C
Setting C = 0 and substituting this result into Eq. 1,
c = 16xy Ans.
For the streamline passing through point (1 m, 0.5 m),
c = 16(1)(0.5) = 8
Thus,
1

. We or
8 = 16xy;  y =

m W ina g
2x

b)
ed e n
in
no W iss ea s

itt id tio
is e D t w
t p or em ch
The plot of this stream function is shown in Fig. a

d th g. in t la
an on in rs h
k g rn to rig
x(m) 0 0.5 1 1.5 2 2.5 3
or in a uc y
w d le tr p
y(m) ∞ 1 0.5 0.333 0.25 0.2 0.167

er ld
e lu nt ns co
th inc de f i es
of rk ( stu e o tat
ity o g us d S
te is ss th ite

y(m)
in f th se for Un
gr w in e
th t o a ly by

1.5
y ar d le d
ro p an o te
st ny s d s ec
de f a rse de ot

s
ill o u vi pr
w le co ro is

1
sa eir is p rk
th d wo
an his

e
T

0.5

x(m)
0 0.5 1 1.5 2 2.5 3

(a)

Ans:
c = 16xy

785
© 2014 Pearson Education, Inc., Upper Saddle River, NJ. All rights reserved. This material is protected under all copyright laws as they currently
exist. No portion of this material may be reproduced, in any form or by any means, without permission in writing from the publisher.

7–51.  The y component of velocity of a two-dimensional


irrotational flow that satisfies the continuity condition is
v = 1 4x + x2 - y2 2 ft>s, where x and y are in feet. Find
the x component of velocity if u = 0 at x = y = 0.

Solution
We consider ideal fluid flow.
In order for the flow to be irrotational, vz = 0.

1 0v 0u
vz = a - b = 0
2 0x 0y
Here,

. We or
0v 0
= ( 4x + x2 - y2 ) = (4 + 2x) rad>s

m W ina g

b)
ed e n
in
no W iss ea s
0x 0x

itt id tio
is e D t w
t p or em ch
d th g. in t la
Thus,

an on in rs h
k g rn to rig
1 0u or in a uc y
c (4 + 2x) - d = 0
w d le tr p

er ld
2 0y
e lu nt ns co
th inc de f i es

0u
= 4 + 2x
of rk ( stu e o tat

0y
ity o g us d S

Integrating with respect to y,


te is ss th ite
in f th se for Un

u = 4y + 2xy + f(x)
gr w in e
th t o a ly by

In order to satisfy the continuity condition


y ar d le d
ro p an o te

0u 0v
+ = 0
st ny s d s ec

0x 0y
de f a rse de ot

s
ill o u vi pr

Here,
w le co ro is
sa eir is p rk

0u 0 0
= 34y + 2xy + f(x) 4 = 2y + 3f(x) 4
th d wo

0x 0x 0x
an his

0v 0
T

+ ( 4x + x2 - y2 ) = -2y
0y 0y
Then,
0
2y + 3f(x) 4 - 2y = 0
0x
0
3f(x) 4 = 0
0x
Integrating with respect to x,
f(x) = C
Thus,
u = 4y + 2xy + C
= 2y(2 + x) + C
At y = x = 0, u = 0. Then C = 0, and so
u = 2y(2 + x)  Ans.

Ans:
u = 2y(2 + x)

786
© 2014 Pearson Education, Inc., Upper Saddle River, NJ. All rights reserved. This material is protected under all copyright laws as they currently
exist. No portion of this material may be reproduced, in any form or by any means, without permission in writing from the publisher.

*7–52.  The flow has a velocity of V = {(3y + 8)i} ft>s, y


where y is vertical and is in feet. Determine if the flow is
rotational or irrotational. If the pressure at point A is 6 lb>ft2, A
determine the pressure at the origin. Take g = 70 lb>ft3.
3 ft

x
O

Solution
We consider ideal fluid flow.
The x and y components of velocity are
u = (3y + 8) ft>s  v = 0

. We or
Here,

m W ina g

b)
ed e n
in
no W iss ea s

itt id tio
0u 0u 0

is e D t w
t p or em ch
= 0;   = (3y + 8) = 3 rad>s

d th g. in t la
0x 0y 0y

an on in rs h
k g rn to rig
Thus, or in a uc y
w d le tr p

er ld
e lu nt ns co

1 0v 0u 1
vz = a - b = (0 - 3) = - 1.5
th inc de f i es

2 0x 0y 2
of rk ( stu e o tat
ity o g us d S

Since vz ≠ 0, the flow is rotational. Thus, the Bernoulli equation can not be applied
te is ss th ite
in f th se for Un

from O to A. Instead, we will first apply Euler’s equation along the y axis. Here,
gr w in e
th t o a ly by

0v 0v
= = 0.
y ar d le d

0x 0y
ro p an o te
st ny s d s ec
de f a rse de ot

Then,
s
ill o u vi pr

1 0p 0v 0v
w le co ro is

- - g = u + v = 0
sa eir is p rk

r 0y 0x 0y
th d wo

0p
an his

= - rg = - g
T

0y
Integrating with respect to y,
p = - gy + f(x)

Substituting this result into the Euler equation along the x axis, with
0u 0 0u 0
= (3y + 8) = 0 and = (3y + 8) = 3 rad>s,
0x 0x 0y 0y

1 0p 0u 0u
- = u + v
r 0x 0x 0y
1 0
- 3 - gy + f(x) 4 = 0 + 0
r 0x
0
3f(x) 4 = 0
0x

787
© 2014 Pearson Education, Inc., Upper Saddle River, NJ. All rights reserved. This material is protected under all copyright laws as they currently
exist. No portion of this material may be reproduced, in any form or by any means, without permission in writing from the publisher.

7–52. Continued

Integrating with respect to x,


f(x) = C
Thus,
p = - gy + C
lb
At point A, y = 3 ft and p = 6 . Then,
ft 2

. We or
lb lb

m W ina g

b)
ed e n
in
6 = a - 70 2 b(3ft) + C

no W iss ea s

itt id tio
is e D t w
ft 2

t p or em ch
ft

d th g. in t la
an on in rs h
C = 216 lb>ft 2

k g rn to rig
Thus,
or in a uc y
w d le tr p

er ld
e lu nt ns co

p = ( - gy + 216) lb>ft 2
th inc de f i es
of rk ( stu e o tat

At point O, y = 0 Thus,
ity o g us d S

lb
te is ss th ite

pO = 3 - (70)(0) + 2164
in f th se for Un

ft 2
gr w in e
th t o a ly by

lb
= 216 Ans.
y ar d le d

ft 2
ro p an o te
st ny s d s ec
de f a rse de ot

s
ill o u vi pr
w le co ro is
sa eir is p rk
th d wo
an his

e
T

788
© 2014 Pearson Education, Inc., Upper Saddle River, NJ. All rights reserved. This material is protected under all copyright laws as they currently
exist. No portion of this material may be reproduced, in any form or by any means, without permission in writing from the publisher.

7–53.  A tornado has a measured wind speed of 12 m>s a


distance of 50 m from its center. If a building has a flat roof
and is located 10 m from the center, determine the uplift
pressure on the roof. The building is within the free vortex
of the tornado. The density of the air is ra = 1.20 kg>m3.

Solution
We consider ideal fluid flow.
Since the tornado is a free vortex flow, its velocity components are
k
vr = 0    vu =
r
Thus
k

. We or
V = vu =

m W ina g
r

b)
ed e n
in
no W iss ea s

itt id tio
is e D t w
t p or em ch
d th g. in t la
It is required that at r = 50 m, V = 12 m>s. Therefore

an on in rs h
k g rn to rig
k
12 m>s = ;    k = 600 m2 >s
or in a uc y
50 m
w d le tr p

er ld
e lu nt ns co

Then
th inc de f i es
of rk ( stu e o tat

600
ity o g us d S

V = a b m>s
r
te is ss th ite

At r = 10 m,
in f th se for Un
gr w in e
th t o a ly by

600
V = = 60 m>s
y ar d le d

10
ro p an o te
st ny s d s ec

Since free vortex flow is irrotational, Bernoulli’s equation can be applied between
de f a rse de ot

two points on the different streamlines such as two points on two circular streamlines
ill o u vi pr

of radius r = ∞ and r = 10 m. At r = ∞ , V∞ = 0 and pB = 0. Since the flow occurs


w le co ro is
sa eir is p rk

in the horizontal plane, the gravity term can be excluded.


th d wo

p∞ V 2 p V2
an his

+ ∞ = +
ra 2 ra 2
T

p ( 60 m>s ) 2
0 + 0 = +
1.20 kg>m3 2

p = 2160 Pa = - 2.16 kPa Ans.

The negative sign indicates that suction develops.

Ans:
- 2.16 kPa

789
© 2014 Pearson Education, Inc., Upper Saddle River, NJ. All rights reserved. This material is protected under all copyright laws as they currently
exist. No portion of this material may be reproduced, in any form or by any means, without permission in writing from the publisher.

7–54.  Show that the equation that defines a sink will


satisfy continuity, which in polar coordinates is written as
0(vr r) 0(vu)
+ = 0.
0r 0u

Solution
We consider ideal fluid flow.
q
For sink flow, vr = - and vu = 0. Then,
2pr
0 ( vrr ) 0 q 0 q
= ca- b(r) d = a- b = 0
0r 0r 2pr 0r 2p
0vu

. We or
= 0

m W ina g

b)
ed e n
0u

in
no W iss ea s

itt id tio
is e D t w
t p or em ch
d th g. in t la
Thus,

an on in rs h
k g rn to rig
0(vr r) 0vu
+ = 0 + 0 = 0 or in a uc y (Q.E.D)
w d le tr p

er ld
0r 0u
e lu nt ns co
th inc de f i es
of rk ( stu e o tat
ity o g us d S
te is ss th ite
in f th se for Un
gr w in e
th t o a ly by
y ar d le d
ro p an o te
st ny s d s ec
de f a rse de ot

s
ill o u vi pr
w le co ro is
sa eir is p rk
th d wo
an his

e
T

790
© 2014 Pearson Education, Inc., Upper Saddle River, NJ. All rights reserved. This material is protected under all copyright laws as they currently
exist. No portion of this material may be reproduced, in any form or by any means, without permission in writing from the publisher.

7–55.  A source at O creates a flow from point O that is


described by the potential function f = (8 ln r) m2 >s,
where r is in meters. Determine the stream function, and 30
specify the velocity at point r = 5 m, u = 15°. O
u

Solution
We consider ideal fluid flow.
The r and u components of velocity are
0f 0 8
vr = = (8 ln r) = a b m>s
0r 0r r
1 0f 1 0
vu = = (8 ln r) = 0
r 0u r 0u

. We or
m W ina g

b)
ed e n
Applying,

in
no W iss ea s

itt id tio
is e D t w
t p or em ch
d th g. in t la
1 0c 8 1 0c
vr = ;   =

an on in rs h
r 0u r r 0u
k g rn to rig
Integrating this equation with respect to u,
or in a uc y
w d le tr p

er ld
e lu nt ns co

c = 8u + f(r)
th inc de f i es
of rk ( stu e o tat

Substituting this result,


ity o g us d S

0c 0
te is ss th ite

vu = - ;  0 = - 38u + f(r) 4


0r 0r
in f th se for Un
gr w in e

0
th t o a ly by

0 = 0- 3f(r) 4
0r
y ar d le d
ro p an o te
st ny s d s ec

Integrating this equation with respect to r,


de f a rse de ot

f(r) = C
ill o u vi pr
w le co ro is

Thus,
sa eir is p rk
th d wo

c = 8u + C
an his

Setting C = 0,
T

c = 8u Ans.
At r = 5 m, u = 15°,
8
vr = = 1.6 m>s
5
vu = 0

Thus, the magnitude of the velocity is

V = 2vr2 + vu2 = 2 ( 1.6 m>s ) 2 + (0)2 = 1.60 m>s2 Ans.

Ans:
c = 8u
V = 1.60 m>s2

791
© 2014 Pearson Education, Inc., Upper Saddle River, NJ. All rights reserved. This material is protected under all copyright laws as they currently
exist. No portion of this material may be reproduced, in any form or by any means, without permission in writing from the publisher.

*7–56.  Combine a source of strength q with a free


counterclockwise vortex, and sketch the resultant
streamline for c = 0.

Solution
We consider ideal fluid flow.
Superimposing the streamlines of a source and a free vortex,
q
c = u - k ln r
2p
For c = 0,
q

. We or
0 = u - k ln r
2p

m W ina g

b)
ed e n
in
no W iss ea s

itt id tio
is e D t w
t p or em ch
q

d th g. in t la
ln r = u
2pk

an on in rs h
k g rn to rig
q
e ln r = e 2pku or in a uc y
w d le tr p

er ld
e lu nt ns co

q
r = e 2pku
th inc de f i es
of rk ( stu e o tat

This equation represents a logarithmic spiral from the source and its plot is shown
ity o g us d S

in Fig. a.
te is ss th ite
in f th se for Un
gr w in e
th t o a ly by
y ar d le d
ro p an o te
st ny s d s ec
de f a rse de ot

s
ill o u vi pr

source
w le co ro is
sa eir is p rk
th d wo
an his

1
T

(a)

792
© 2014 Pearson Education, Inc., Upper Saddle River, NJ. All rights reserved. This material is protected under all copyright laws as they currently
exist. No portion of this material may be reproduced, in any form or by any means, without permission in writing from the publisher.

7–57.  A free vortex is defined by its stream function


c = ( - 240 ln r) m2 >s, where r is in meters. Determine the
velocity of a particle at r = 4 m and the pressure at points
on this streamline. Take r = 1.20 kg>m3.

r4m

Solution
We consider ideal fluid flow.
The velocity components are
1 0c
vr = ;  vr = 0
r 0u
0c 240
vu = - ;  vu = a b m>s
0r r

. We or
m W ina g

b)
Thus, the velocity is

ed e n
in
no W iss ea s

itt id tio
is e D t w
t p or em ch
d th g. in t la
240
V = vu = a b m>s

an on in rs h
r

k g rn to rig
At r = 4 m,
or in a uc y
w d le tr p

er ld
e lu nt ns co

240
th inc de f i es

V = a b m>s = 60.0 m>s Ans.


of rk ( stu e o tat

4
ity o g us d S

Since a free vortex flow is irrotational, Bernoulli’s equation can be applied


te is ss th ite

between two points on the different streamlines. In this case, the two points are
in f th se for Un

on the circular streamlines r = ∞ where V0 = 0 and p0 = 0 and r = 4 m where


gr w in e
th t o a ly by

V = 60.0 m>s. Since the flow occurs in the horizontal plane, z0 = z.


y ar d le d
ro p an o te

p0 V0 2 p V2
st ny s d s ec

+ + gz0 = + + gz
de f a rse de ot

r 2 r 2
s
ill o u vi pr
w le co ro is

p ( 60.0 m>s ) 2
sa eir is p rk

0 + 0 + gz = + + gz
th d wo

3 2
1.20 kg>m
an his

p = - 2160 Pa = -2.16 kPa Ans.


T

Ans:
V = 60.0 m>s
p = - 2.16 kPa

793
© 2014 Pearson Education, Inc., Upper Saddle River, NJ. All rights reserved. This material is protected under all copyright laws as they currently
exist. No portion of this material may be reproduced, in any form or by any means, without permission in writing from the publisher.

7–58.  Determine the location of the stagnation point for a y


combined uniform flow of 8 m>s and a source having a
strength of 3 m2 >s. Plot the streamline passing through the
stagnation point.
8 m/s

Solution
We consider ideal fluid flow.
This is a case of flow past a half body. The location of the stagnation point P is at
u = p Ans.
Using
q 3 m2 >s 3
r = r0 = = = m Ans.
2pU 2p(8 m>s) 16p

. We or
m W ina g

b)
ed e n
in
no W iss ea s

itt id tio
is e D t w
The equation of the streamline (boundary of a half body) that passes through the

t p or em ch
d th g. in t la
stagnation point P can be determined by applying.

an on in rs h
k g rn to rig
r0(p - u)
r = or in a uc y
w d le tr p

er ld
sin u
e lu nt ns co

3
th inc de f i es

(p - u)
of rk ( stu e o tat

16p
r =
ity o g us d S

sin u
te is ss th ite

3(p - u)
in f th se for Un

y
r =
gr w in e

16p sin u
th t o a ly by

asymptote
y ar d le d

This equation can be written in the form


ro p an o te
st ny s d s ec

3
r sin u = (p - u) 3
de f a rse de ot

16p m
s
ill o u vi pr

16 r
P
w le co ro is

Since y = r sin u, this equation becomes ¨


sa eir is p rk

3 x
th d wo

y = (p - u) source
16p
an his

3
m
T

The half width h of the half body can be determined by setting y = h as u approaches 16
0 or 2p. Thus, 3
m
16
3 3
h = (p - u) = m asymptote
16p 16
The plot of the half body is shown in Fig. a. (a)

Ans:
u = p
3
r = m
16p

794
© 2014 Pearson Education, Inc., Upper Saddle River, NJ. All rights reserved. This material is protected under all copyright laws as they currently
exist. No portion of this material may be reproduced, in any form or by any means, without permission in writing from the publisher.

7–59.  As water drains from the large cylindrical tank, its z


surface forms a free vortex having a circulation of Γ.
Assuming water to be an ideal fluid, determine the equation
z = f(r) that defines the free surface of the vortex.
Hint: Use the Bernoulli equation applied to two points on
the surface. r

Solution
We consider ideal fluid flow.
For a free vortex, the radial and transverse components of velocity are
k
vr = 0  and  vu =

. We or
r

m W ina g

b)
ed e n
in
no W iss ea s

itt id tio
Then

is e D t w
t p or em ch
d th g. in t la
k A

an on in rs h
V = vu =
k g rn to rig
r r
or in a uc y
w d le tr p

er ld
e lu nt ns co

For a circulation Γ,
th inc de f i es

2p z
V # ds =
C L0 r
k
of rk ( stu e o tat

Γ = (rdu) = 2p k r
ity o g us d S
te is ss th ite

Γ
in f th se for Un

k = B
gr w in e

2p
th t o a ly by

Thus,
y ar d le d
ro p an o te
st ny s d s ec

Γ
V =
de f a rse de ot

2pr
s
ill o u vi pr
w le co ro is

Since a free vortex is irrotational flow, Bernoulli’s equation can be applied between
sa eir is p rk

z
th d wo

two points on different streamlines, such as point A and B shown in Fig. a. Point A
an his

(a)
is located at (r = ∞ , 0) where pA = patm = 0 and VA = 0, and point B is located
T

Γ
at (r, z) where pB = patm = 0 and VB = . Establish the datum through point A,
2pr
pA VA2 pB VB2
+ + gzA = + + gzB
rw 2 rw 2
Γ 2
a b
2pr
0 + 0 + g(0) = 0 + + g( -z)
2

Γ2
gz =
8p2r 2
Γ2
z = Ans.
8p2gr 2

Ans:
Γ2
z =
8p2gr 2

795
© 2014 Pearson Education, Inc., Upper Saddle River, NJ. All rights reserved. This material is protected under all copyright laws as they currently
exist. No portion of this material may be reproduced, in any form or by any means, without permission in writing from the publisher.

*7–60.  Pipe A provides a source flow of 5 m2 >s, whereas y


the drain, or sink, at B removes 5 m2 >s. Determine the
stream function between AB, and show the streamline for
c = 0.
5 m/s2 5 m/s2
A B
x

2m 2m

Solution
We consider ideal fluid flow.
When the source and sink are superimposed, Fig. a, the resultant stream function is
q q q 5 m2 >s

. We or
c = u2 - u1 = (u2 - u1) = (u2 - u1) Ans.

m W ina g
2p 2p 2p 2p

b)
ed e n
in
no W iss ea s

itt id tio
is e D t w
t p or em ch
d th g. in t la
Here u2 is defined from A and u1 is defined from B.

an on in rs h
k g rn to rig
Staying in polar coordinates, c = 0 implies u2 - u1 = 0 or u2 = u1. The graph of the
or in a uc y
w d le tr p

er ld
corresponding points is shown in Fig. a, with the direction of flow indicated. Note
e lu nt ns co

that “the” steamline has two distinct segments.


th inc de f i es
of rk ( stu e o tat
ity o g us d S

y
te is ss th ite
in f th se for Un
gr w in e
th t o a ly by
y ar d le d
ro p an o te
st ny s d s ec
de f a rse de ot

s
ill o u vi pr
w le co ro is
sa eir is p rk
th d wo

x
an his

e
T

(a)

796
© 2014 Pearson Education, Inc., Upper Saddle River, NJ. All rights reserved. This material is protected under all copyright laws as they currently
exist. No portion of this material may be reproduced, in any form or by any means, without permission in writing from the publisher.

7–61.  Pipe A provides a source flow of 5 m2 >s, whereas y


the drain at B removes 5 m2 >s. Determine the potential
function between AB, and show the equipotential line for
f = 0.
5 m/s2 5 m/s2
A B
x

2m 2m

Solution
We consider ideal fluid flow.
When the source and sink are superimposed, the resultant potential function is
q q q r2 5 m2 >s r2

. We or
f = ln r2 - ln r1 = ln = ln Ans.

m W ina g
2p 2p 2p r1 2p r1

b)
ed e n
in
no W iss ea s

itt id tio
is e D t w
t p or em ch
d th g. in t la
Here r2 is measured from B and r1 is measured from A.

an on in rs h
k g rn to rig
Staying in polar coordinates, f = 0 implies or in a uc y
w d le tr p

er ld
r2
e lu nt ns co

ln = 0
th inc de f i es

r1
of rk ( stu e o tat

r2
ity o g us d S

= 1
r1
te is ss th ite
in f th se for Un

r1 = r2
gr w in e
th t o a ly by

Thus, the equipotential line for f = 0 is along the y axis as shown in Fig. a.
y ar d le d
ro p an o te

y
st ny s d s ec
de f a rse de ot

s
ill o u vi pr
w le co ro is

f=0
sa eir is p rk
th d wo
an his

x
T

(a)

Ans:
5 m2 >sr2
f = ln
2p r1
The equipotential line for f = 0 is along the y axis.

797
© 2014 Pearson Education, Inc., Upper Saddle River, NJ. All rights reserved. This material is protected under all copyright laws as they currently
exist. No portion of this material may be reproduced, in any form or by any means, without permission in writing from the publisher.

7–62.  A source having a strength of q = 80 ft2 >s is y


located at point A (4 ft, 2 ft). Determine the magnitudes of
the velocity and acceleration of fluid particles at point
B (8 ft, 1 ft).
A

2 ft
x
1 ft

4 ft 4 ft B

Solution
We consider ideal fluid flow.
The radial and transverse components of the velocity are
q
vr = vu = 0

. We or
2pr

m W ina g

b)
ed e n
in
no W iss ea s

itt id tio
is e D t w
Thus, the magnitude of the velocity is

t p or em ch
d th g. in t la
q

an on in rs h
V = vr =

k g rn to rig
2pr
or in a uc y
w d le tr p

er ld
e lu nt ns co

Here, r = 2(8 ft - 4 ft)2 + ( - 1 ft - 2 ft)2 = 5 ft.  Then


th inc de f i es

80 ft 2 >s
of rk ( stu e o tat

V = = 2.546 ft>s = 2.55 ft>s Ans.


ity o g us d S

2p (5 ft)
te is ss th ite

x any y components of the velocity are


in f th se for Un

u = vr cos u v = vr sin u
gr w in e
th t o a ly by

x y
y ar d le d

Here cos u = and sin u = ,


ro p an o te

r r
st ny s d s ec
de f a rse de ot

q x q x q y q y
ill o u vi pr

u = a b = a b  v = a b = a b
w le co ro is

2pr r 2p r 2 2pr r 2p r 2
sa eir is p rk
th d wo

However, r 2 = x2 + y2.  Then


an his

q x q y
T

u = a 2 2
b  v = a 2 b
2p x + y 2p x + y2
x any y components of the acceleration are
0u 0u 0u
ax = + u + v
0t 0x 0y

q x q y2 - x2 q y q -2xy
= 0 + a 2 b • £ § ¶ + a 2 b• £ 2 § ¶
2p x + y 2 2p ( x + y )
2 2 2 2p x + y 2 2p ( x + y2 ) 2

q2 x3 + xy2
= - c d
4p 2
( x2 + y2 ) 3
0v 0v 0v
ay = + u + v
0t 0x 0y

q x q - 2xy q y q x2 - y2
= 0 + a 2 b • £ § ¶ + a b • £ § ¶
2p x + y2 2p ( x2 + y2 ) 2 2p x2 + y2 2p ( x2 + y2 ) 2

q2 y3 + x2y
= - c d
4p 2
( x2 + y2 ) 3

798
© 2014 Pearson Education, Inc., Upper Saddle River, NJ. All rights reserved. This material is protected under all copyright laws as they currently
exist. No portion of this material may be reproduced, in any form or by any means, without permission in writing from the publisher.

7–62. Continued

With respect to point A, the coordinates of point B are B[(8 - 4) ft, ( -1 - 2) ft]
= B ( 4 ft, - 3 ft ) . Then

( 80 ft2 >s ) 2 (4 ft)3 + (4 ft)( - 3 ft)2


ax = - c s = -1.038 ft>s2
4p2 3 (4 ft)2 + ( - 3 ft)2 4 3

. We or
m W ina g

b)
ed e n
in
no W iss ea s

itt id tio
is e D t w
( 80 ft2 >s ) 2 ( -3 ft)3 + (4 ft)2( -3 ft)

t p or em ch
d th g. in t la
ay = - c s = 0.7781 ft>s2
4p2 3 (4 ft)2 + ( - 3 ft)2 4 3

an on in rs h
k g rn to rig
or in a uc y
w d le tr p

er ld
Thus, the magnitude of the acceleration is
e lu nt ns co
th inc de f i es

a = 2ax2 + ay2 = 2 ( - 1.038 ft>s2 ) 2 + ( 0.7781 ft>s2 ) 2 = 1.297 ft>s2 = 1.30 ft>s2 Ans.
of rk ( stu e o tat
ity o g us d S

As an alternative solution,
te is ss th ite

0vr 0vr
a = + vr
in f th se for Un

0t 0r
gr w in e
th t o a ly by

q q 1
y ar d le d

= 0 + a a- 2b b
ro p an o te

2pr 2p r
st ny s d s ec
de f a rse de ot

q2 (80 ft 2 >s)2
s
ill o u vi pr

= 2 2 = = 1.30 ft>s2 Ans.


(2p)2r 3
(2p)2(5 ft)3
w le co ro is
sa eir is p rk
th d wo
an his

e
T

Ans:
V = 2.55 ft>s
a = 1.30 ft>s2

799
© 2014 Pearson Education, Inc., Upper Saddle River, NJ. All rights reserved. This material is protected under all copyright laws as they currently
exist. No portion of this material may be reproduced, in any form or by any means, without permission in writing from the publisher.

7–63.  Two sources, each having a strength of 2 m2 >s, are y


located as shown. Determine the x and y components of
the velocity of fluid particles that pass point (x, y). What is
the equation of the streamline that passes through point
(0, 8 m) in Cartesian coordinates? Is the flow irrotational?

4m 4m

Solution
We consider ideal fluid flow.
When sources (1) and (2) are superimposed, Fig. a, the resultant stream function is
q q q
c = u + u = ( u + u2 )

. We or
2p 1 2p 2 2p 1

m W ina g

b)
ed e n
in
no W iss ea s

itt id tio
is e D t w
From the geometry shown in Fig. a,

t p or em ch
d th g. in t la
y y

an on in rs h
k g rn to rig
u1 = tan-1a b   u2 = tan-1a b
x - 4 x + 4
or in a uc y
w d le tr p

er ld
e lu nt ns co

Then,
th inc de f i es

q y y
of rk ( stu e o tat

c = c tan-1a b + tan-1a b d (1)


ity o g us d S

2p x - 4 x + 4
te is ss th ite

The x and y components of velocity are


in f th se for Un
gr w in e
th t o a ly by

0c q 1 1 1 1
u = = ≥ a b + a b¥
y ar d le d
ro p an o te

0y 2p y 2 x - 4 y 2 x + 4
st ny s d s ec

1 + a b 1 + a b
x - 4 x + 4
de f a rse de ot

s
ill o u vi pr

q
w le co ro is

x - 4 x + 4
= £ + § Ans.
sa eir is p rk

2p (x - 4)2 + y2 (x + 4)2 + y2
th d wo
an his

e
T

0c q 1 y 1 y
v = - = - ≥ £- § + c- d¥
0y 2p y 2
(x - 4) 2 y 2
(x + 4)2
1 + a b 1 + a b
x - 4 x + 4
q y y
= £ + § Ans.
2p (x - 4)2 + y2 (x + 4)2 + y2
Here,

dv q 2y(x - 4) 2y(x + 4)
= £ + §
dx 2p 3 (x - 4)2 + y2 4 2 3(x + 4)2 + y24 2

du q 2y(x - 4) 2y(x + 4)
= £ + §
dy 2p 3 (x - 4)2 + y2 4 2 3 (x + 4)2 + y2 4 2

Substituting these results into


1 0v 0u
vz = a - b = 0
2 0x 0y

800
© 2014 Pearson Education, Inc., Upper Saddle River, NJ. All rights reserved. This material is protected under all copyright laws as they currently
exist. No portion of this material may be reproduced, in any form or by any means, without permission in writing from the publisher.

7–63. Continued

Since vz = 0, the flow is irrotational. For q = 2 m2 >s, the streamline that passes
through point x = 0 and y = 8 m can be determined using Eq. 1,

2 8 8
c = c tan-1a b + tan-1a bd = 0
2p 0 - 4 0 + 4

. We or
m W ina g

b)
ed e n
Thus, the equation of this streamline is

in
no W iss ea s

itt id tio
is e D t w
t p or em ch
d th g. in t la
y y
tan-1a b + tan-1a b = 0

an on in rs h
x - 4 x + 4
k g rn to rig
or in a uc y
w d le tr p
y y

er ld
e lu nt ns co

tan-1a b = - tan-1a b
x - 4 x + 4
th inc de f i es
of rk ( stu e o tat

y y
= -
ity o g us d S

x - 4 x + 4
te is ss th ite

x = 0 Ans.
in f th se for Un
gr w in e
th t o a ly by
y ar d le d

y
ro p an o te
st ny s d s ec
de f a rse de ot

x
ill o u vi pr
w le co ro is
sa eir is p rk

r2
th d wo

r1 y
r
an his

e
T

2 1
x
2 1
4m 4m

Ans:

q x - 4 x + 4
u = £ + §
2p 1 x - 42 2 + y2 1x + 42 2 + y2

q y y
v = £ + §
2p 1 x - 42 2 + y2 1x + 42 2 + y2

The flow is irrotational.


x = 0

801
© 2014 Pearson Education, Inc., Upper Saddle River, NJ. All rights reserved. This material is protected under all copyright laws as they currently
exist. No portion of this material may be reproduced, in any form or by any means, without permission in writing from the publisher.

*7–64.  The source and sink of equal strength q are located y


a distance d from the origin as indicated. Determine the
stream function for the flow, and draw the streamline that
passes through the origin.

d d

Solution
We consider ideal fluid flow.
When the source and sink are superimposed, Fig. a, the resultant stream function is
q q q
c = u - u = ( u - u2 ) Ans.

. We or
2p 1 2p 2 2p 1

m W ina g

b)
ed e n
in
no W iss ea s

itt id tio
is e D t w
Staying in polar coordinates, at the origin u1 = p and u2 = 0.

t p or em ch
d th g. in t la
q q

an on in rs h
c = (p - 0) =
k g rn to rig
2p 2 or in a uc y
w d le tr p

er ld
e lu nt ns co

So the streamline satisfies


th inc de f i es

q q
= ( u - u2 )
of rk ( stu e o tat

2 2p 1
ity o g us d S

u1 = u2 + p
te is ss th ite
in f th se for Un

Thus, the streamline passing through the origin is the straight segment between the
gr w in e

source and the sink, as shown in Fig. a.


th t o a ly by
y ar d le d

y
ro p an o te
st ny s d s ec
de f a rse de ot

s
ill o u vi pr
w le co ro is
sa eir is p rk

q
=
th d wo

2
x
an his

e
T

(a)

802
© 2014 Pearson Education, Inc., Upper Saddle River, NJ. All rights reserved. This material is protected under all copyright laws as they currently
exist. No portion of this material may be reproduced, in any form or by any means, without permission in writing from the publisher.

7–65.  Two sources, each having a strength q, are located y


as shown. Determine the stream function, and show that
this is the same as having a single source with a wall along
the y axis.

y
d d

Solution
We consider ideal fluid flow.
When sources (1) and (2) are superimposed, Fig. a, the resultant stream function is
q q q
( u + u2 )

. We or
c = u + u = Ans.
2p 1 2p 2 2p 1

m W ina g

b)
ed e n
in
no W iss ea s

itt id tio
is e D t w
t p or em ch
In order for the stream function to be the same as that of a single source and a wall

d th g. in t la
along the y axis, a streamline must exist along the y axis. However, by geometry,

an on in rs h
k g rn to rig
along the y axis it is always true that u1 + u2 = {p, so that the value of the stream
or in a uc y
w d le tr p
q q q q

er ld
( {p) { , where + corresponds to the +y axis and -
e lu nt ns co

function is c =
2p 2 2 2
th inc de f i es

corresponds to the ‑y axis.


of rk ( stu e o tat
ity o g us d S

y
te is ss th ite
in f th se for Un

x
gr w in e
th t o a ly by
y ar d le d

r2
ro p an o te

r1
st ny s d s ec

r y
de f a rse de ot

s
ill o u vi pr

2 1
x
w le co ro is

1
2
sa eir is p rk

d d
th d wo
an his

e
T

(a)

Ans:
q
c = ( u + u2 )
2p 1

803
© 2014 Pearson Education, Inc., Upper Saddle River, NJ. All rights reserved. This material is protected under all copyright laws as they currently
exist. No portion of this material may be reproduced, in any form or by any means, without permission in writing from the publisher.

7–66.  A source q is emitted from the wall while a flow


y
occurs towards the wall. If the stream function is described
as c = (4xy + 8u) m2 >s, where x and y are in meters,
determine the distance d from the wall where the stagnation
point occurs along the y axis. Plot the streamline that passes
through this point.

d
x

Solution
We consider ideal fluid flow.
Here, x = r cos u and y = r sin u.  Then in terms of r and u coordinates,

. We or
c = 4(r cos u)(r sin u) + 8u

m W ina g

b)
ed e n
in
no W iss ea s

itt id tio
c = 2r 2 sin 2u + 8u

is e D t w
(1)

t p or em ch
d th g. in t la
The velocity components are

an on in rs h
k g rn to rig
1 0c 1 1 or in a uc y
= 3 2r 2(2 cos 2u) + 8 4 = ( 4r 2 cos 2u + 8 )
w d le tr p
vr =

er ld
e lu nt ns co

r 0u r r
th inc de f i es

0c
of rk ( stu e o tat

vu = - = - (4r sin 2u)


0r
ity o g us d S

At stagnation point p, it is required that these velocity components are equal to zero.
te is ss th ite
in f th se for Un

vu = - 4r sin 2u = 0
gr w in e
th t o a ly by

sin 2u = 0 (since r ≠ 0)
y ar d le d
ro p an o te

2u = 0,  p rad
st ny s d s ec
de f a rse de ot

p
s

u = 0,  rad
ill o u vi pr

2
w le co ro is

p
sa eir is p rk

u = rad is chosen and it gives the direction r of the stagnation point.


th d wo

2
an his

1
vr = ( 4r 2 cos 2u + 8 ) = 0
T

r
1
Since ≠ 0, then
r
4r 2 cos 2u + 8 = 0
p
Substituting u = rad and r = d into this equation,
2
p
4d 2 cos c 2 a b d + 8 = 0
2

d = 22 m Ans.
p
Substituting u = rad and r = 22 m into Eq 1,
2
p p
c = 2 ( 22 ) 2 sin c 2 a b d + 8 a b = 4p
2 2

804
© 2014 Pearson Education, Inc., Upper Saddle River, NJ. All rights reserved. This material is protected under all copyright laws as they currently
exist. No portion of this material may be reproduced, in any form or by any means, without permission in writing from the publisher.

7–66. Continued

Therefore, the streamline passing through the stagnation point is given by


4p - 8u
4p = 2r 2 sin 2u + 8u    r 2 =
2 sin 2u
The plot of this stream function is shown in Fig. a

. We or
m W ina g

b)
ed e n
in
5p

no W iss ea s
p p p p p

itt id tio
u(rad)

is e D t w
t p or em ch
d th g. in t la
12 6 4 3 12 2

an on in rs h
r(m) 3.236 2.199 1.772 1.555 1.447 undef.
k g rn to rig
or in a uc y
w d le tr p

er ld
e lu nt ns co
th inc de f i es
of rk ( stu e o tat

y (m)
ity o g us d S
te is ss th ite

5
in f th se for Un

12
gr w in e
th t o a ly by

3
y ar d le d

4
ro p an o te
st ny s d s ec
de f a rse de ot

6
s
ill o u vi pr
w le co ro is

12
sa eir is p rk
th d wo
an his

x (m)
T

(a)

Ans:
22 m

805
© 2014 Pearson Education, Inc., Upper Saddle River, NJ. All rights reserved. This material is protected under all copyright laws as they currently
exist. No portion of this material may be reproduced, in any form or by any means, without permission in writing from the publisher.

7–67.  Determine the equation of the boundary of the half y


body formed by placing a source of 0.5 m2 >s in the uniform
flow of 8 m>s. Express the result in Cartesian coordinates. 8 m/s

x
A

Solution
We consider ideal fluid flow.
For the flow past a half body,
q 0.5 m2 >s 0.03125

. We or
r0 = = = m

m W ina g
2pU 2p(8 m>s)

b)
ed e n
p

in
no W iss ea s

itt id tio
is e D t w
t p or em ch
d th g. in t la
The equation of the boundary of a half body is given by

an on in rs h
k g rn to rig
0.03125
(p - u)
or in a uc y
r0(p - u)
w d le tr p
p

er ld
e lu nt ns co

r = =
sin u sin u
th inc de f i es
of rk ( stu e o tat

0.03125
r sin u = (p - u)
ity o g us d S

p
te is ss th ite

y
in f th se for Un

Here, y = r sin u and u = tan-1 . Then, this equation becomes


x
gr w in e
th t o a ly by

y
y ar d le d

32py = p - tan-1
ro p an o te

x
st ny s d s ec
de f a rse de ot

y
s

tan-1
ill o u vi pr

= p(1 - 32y)
x
w le co ro is
sa eir is p rk

y
th d wo

= tan [p(1 - 32y)] Ans.


x
an his

e
T

Ans:
y = x tan 3 p ( 1 - 32y ) 4

806
© 2014 Pearson Education, Inc., Upper Saddle River, NJ. All rights reserved. This material is protected under all copyright laws as they currently
exist. No portion of this material may be reproduced, in any form or by any means, without permission in writing from the publisher.

*7–68.  The leading edge of a wing is approximated by the 300 ft/s


half body. It is formed from the superposition of A
the uniform air flow of 300 ft>s and a source. Determine the r  0.3 ft
required strength of the source so that the width of the half u  90
body is 0.4 ft.
O

Solution
We consider ideal fluid flow.
For the flow past a half body,
q q q
r0 = = = ft

. We or
2pU 2p(300 ft>s) 600p

m W ina g

b)
ed e n
in
no W iss ea s

itt id tio
is e D t w
t p or em ch
The equation of the boundary of a half body is given by

d th g. in t la
an on in rs h
r0(p - u)

k g rn to rig
r =
sin u or in a uc y
w d le tr p

er ld
e lu nt ns co

q
(p - u)
th inc de f i es

600p q(p - u)
of rk ( stu e o tat

r = =
sin u 600p sin u
ity o g us d S

q(p - u)
te is ss th ite

r sin u =
in f th se for Un

600p
gr w in e
th t o a ly by

Since y = r sin u, this equation becomes


y ar d le d
ro p an o te

q(p - u)
st ny s d s ec

y =
de f a rse de ot

600p
s
ill o u vi pr

The half width h of the half body can be determined by setting y = h as u approaches
w le co ro is
sa eir is p rk

0 or 2p. Thus,
th d wo

q(p - u) q
an his

h = =
600p 600
T

0.4 ft
Here, h = = 0.2 ft. Then
2
q
0.2 ft =
600
q = 120 ft 2 >s Ans.

807
© 2014 Pearson Education, Inc., Upper Saddle River, NJ. All rights reserved. This material is protected under all copyright laws as they currently
exist. No portion of this material may be reproduced, in any form or by any means, without permission in writing from the publisher.

7–69.  The leading edge of a wing is approximated by the half 300 ft/s
body. It is formed from the superposition of the uniform air A
flow of 300 ft>s and a source having a strength of 100 ft2 >s. r  0.3 ft
Determine the width of the half body and the difference in u  90
pressure between the stagnation point O and point A, where
r = 0.3 ft, u = 90. Take r = 2.35(103) slug>ft3. O

Solution
We consider ideal fluid flow.
For the flow past a half body,
q 100 ft 2 >s 1
r0 = = = ft
2pU 2p(300 ft>s) 6p
The equation of the boundary of a half body is given by
r0(p - u)

. We or
r =

m W ina g

b)
ed e n
in
no W iss ea s
sin u

itt id tio
is e D t w
t p or em ch
d th g. in t la
1
(p - u)

an on in rs h
k g rn to rig
6p
r = or in a uc y
sin u
w d le tr p

er ld
e lu nt ns co

1
th inc de f i es

r sin u = (p - u)
of rk ( stu e o tat

6p
ity o g us d S

Since y = r sin u, this equation becomes


te is ss th ite
in f th se for Un

1
y = (p - u)
gr w in e

6p
th t o a ly by
y ar d le d

The half width h of the half body can be determined by setting y = h as u approaches
ro p an o te
st ny s d s ec

0 or 2p. Thus,
de f a rse de ot

s
ill o u vi pr

1 1
h = (p - u) = ft
w le co ro is

6p 6
sa eir is p rk
th d wo

1
Here, 2h = 2 a ft b = 0.333 ft Ans.
an his

6
e
T

At the stagnation point O, VO = 0. The r and u components of velocity at point A


can be determined using
q 100 ft 2 >s
vr = + U cos u = + ( 300 ft>s ) cos 90° = 53.05 ft>s
2pr 2p(0.3 ft)

vu = - U sin u = - ( 300 ft>s ) sin 90° = - 300 ft>s


Thus, the magnitude of the velocity is

V = 2vr2 + vu2 = 2 ( 53.05 ft>s ) 2 + ( - 300 ft>s ) 2 = 304.65 ft>s


The flow past a half body is irrotational. Thus, the Bernoulli equation for an ideal
fluid is applicable from point O at A. Neglecting the elevation term,
pO VO2 pA VA2
+ = +
r 2 r 2

pO pA ( 304.65 ft>s ) 2
+ 0 = +
2.35 ( 10-3 ) slug>ft 3 2.35 ( 10-3 ) slug>ft 3 2

lb 1 ft 2
∆p = pO - pA = 109.06 a b = 0.757 psi Ans. Ans:
ft 2 12 in.
0.333 ft
∆p = 0.757 psi

808
© 2014 Pearson Education, Inc., Upper Saddle River, NJ. All rights reserved. This material is protected under all copyright laws as they currently
exist. No portion of this material may be reproduced, in any form or by any means, without permission in writing from the publisher.

7–70.  The half body is defined by a combined uniform y


flow having a velocity of U and a point source of strength q.
Determine the pressure distribution along the top boundary
of the half body as a function of u, if the pressure within the
uniform flow is p0. Neglect the effect of gravity. The density
U
of the fluid is r.
r
u
x

Solution
We consider ideal fluid flow.
For the flow past a half body,
q
r0 =

. We or
2pU

m W ina g

b)
ed e n
in
no W iss ea s

itt id tio
is e D t w
t p or em ch
The equation of the boundary of a half body is given by

d th g. in t la
q

an on in rs h
(p - u)
k g rn to rig
r0(p - u) 2pU q(p - u)
or in a uc y
r = = =
w d le tr p

er ld
sin u sin u 2pU sin u
e lu nt ns co
th inc de f i es

The r and u components of velocity at any point on the boundary can be determined
of rk ( stu e o tat

using
ity o g us d S

q q U sin u
te is ss th ite

vr = + U cos u = + U cos u = + U cos u


2pr q(p - u) p - u
in f th se for Un

2pc d
gr w in e

2pU sin u
th t o a ly by
y ar d le d

vu = -U sin u
ro p an o te
st ny s d s ec

Thus, the magnitude of the velocity is


de f a rse de ot

s
ill o u vi pr

2
U sin u
V = 2vr2 + vu2 = 2
w le co ro is

A a p - u + U cos u b + ( -U sin u)
sa eir is p rk
th d wo

U
= 2 sin2u + (p - u) sin 2u + (p - u)2
an his

p - u
T

Since the potential function exists, the flow past a half body is irrotational. The
Bernoulli equation is applicable between any two points in the flow. If point A is an
arbitrary point on the boundary where VA = V and pA = p, and point O is a point
remote from the body where VO = U, then

p0 VO2 pA VA2
+ = +
r 2 r 2

U2
3 sin2u + (p - u) sin 2u + (p - u)2 4
p0 U2 p (p - u)2
+ = +
r 2 r 2
2
rU
p = p0 - 3 sin2u + (p - u) sin 2u 4 Ans.
2(p - u)2

Ans:
rU 2
p = p0 - 3 sin2u + (p - u) sin2u 4
2(p - u)2

809
© 2014 Pearson Education, Inc., Upper Saddle River, NJ. All rights reserved. This material is protected under all copyright laws as they currently
exist. No portion of this material may be reproduced, in any form or by any means, without permission in writing from the publisher.

7–71.  A fluid flows over a half body for which U = 0.4 m>s y
and q = 1.0 m2 >s. Plot the half body, and determine the
magnitudes of the velocity and pressure in the fluid at the
point r = 0.8 m and u = 90o. The pressure within the uniform
flow is 300 Pa. Take r = 850 kg>m3.
U  0.4 m/s

r
u
x
q  1.0 m/s2

Solution
We consider ideal fluid flow.
The location of the stagnation point p can be determined from
q 1.0 m2 >s

. We or
r0 = = = 0.3979 m = 0.398 m

m W ina g
2pU 2p ( 0.4 m>s )

b)
ed e n
in
no W iss ea s

itt id tio
is e D t w
t p or em ch
d th g. in t la
The half width of the half body is

an on in rs h
h = pr0 = p(0.3979 m) = 1.25 m
k g rn to rig
or in a uc y
w d le tr p
The resulting half body is shown in Fig. a. The velocity components of the flow passing

er ld
e lu nt ns co

a half body at point B where r = 0.8 m and u = 90° are


th inc de f i es
of rk ( stu e o tat

q 1.0 m2 >s
(vr)B = + U cos u = + ( 0.4 m>s ) cos 90° = 0.1989 m>s
ity o g us d S

2pr 2p(0.8 m)
te is ss th ite
in f th se for Un

(vu)B = - U sin u = - ( 0.4 m>s ) sin 90° = - 0.4 m>s


gr w in e
th t o a ly by

Thus, the magnitude of the velocity is


y ar d le d
ro p an o te

V = 2(vr)B2 + (vu)B2 = 2 ( 0.1989 m>s ) 2 + ( - 0.4 m>s ) 2 = 0.4467 m>s = 0.447 m>s Ans.
st ny s d s ec
de f a rse de ot

Since the flow passing a half body is irrotational, Bernoulli’s equation can be applied
s
ill o u vi pr

between two points on the different streamlines such as point A within the uniform
w le co ro is
sa eir is p rk

flow and point B. Since the flow occurs in the horizontal plane, the gravity term can be
th d wo

excluded. Here VA = U = 0.4 m>s.


an his

pA VA2 pB VB2
T

+ = +
r 2 r 2
r
pB = pA + ( VA2 - VB2 )
2

850 kg>m3
= 300 Pa + ° ¢ 3 ( 0.4 m>s ) 2 - ( 0.4467 m>s ) 2 4
2

= 283 Pa Ans.

y (m)

1.25 m
p
x (m)
Ans:
1.25 m r0 = 0.398 m
0.398 m h = 1.25 m
V = 0.447 m>s
(a) p = 283 Pa

810
© 2014 Pearson Education, Inc., Upper Saddle River, NJ. All rights reserved. This material is protected under all copyright laws as they currently
exist. No portion of this material may be reproduced, in any form or by any means, without permission in writing from the publisher.

*7–72.  The half body is defined by a combined uniform y


flow having a velocity of U and a point source of strength q.
Determine the location u on the boundary of the half body U
where the pressure p is equal to the pressure p0 within the
uniform flow. Neglect the effect of gravity.
r u
x
Solution
We consider ideal fluid flow.
For the flow past a half body,
q
r0 =
2pU
The equation of the boundary of a half body is given by
q
(p - u)
r0(p - u) 2pU q(p - u)
r = = =
sin u sin u 2pU sin u
The r and u components of velocity at any point on the boundary can be determined
using
q q U sin u
vr = + U cos u = + U cos u = + U cos u
q(p - u)

. We or
2pr p - u
2pc d

m W ina g

b)
ed e n
in
no W iss ea s
2pU sin u

itt id tio
is e D t w
t p or em ch
d th g. in t la
vu = - U sin u

an on in rs h
k g rn to rig
Thus, the magnitude of the velocity is or in a uc y
w d le tr p

er ld
e lu nt ns co
2
U sin u
V = 2vr2 + vu2 = 2
A a p - u + U cos u b + ( -U sin u)
th inc de f i es
of rk ( stu e o tat

U
ity o g us d S

= 2sin2u + (p - u) sin 2u + (p - u)2


p - u
te is ss th ite
in f th se for Un

Since the potential function exists, the flow past a half body is irrotational. The
gr w in e
th t o a ly by

Bernoulli equation is applicable between any two points in the flow. If point A is an
arbitrary point on the boundary where VA = V and pA = pO, and point O is a point
y ar d le d
ro p an o te

remote from the body where VO = U, then


st ny s d s ec
de f a rse de ot

pO VO2 pA VA2
s
ill o u vi pr

+ = +
w le co ro is

r 2 r 2
sa eir is p rk
th d wo

U2
3 sin2u + (p - u) sin 2u + (p - u)2 4
an his

(p - u)2
e

pO pA2
U
+ = +
T

r 2 r 2
rU 2
pO - pA = 3 sin2 u + (p - u) sin 2u 4 = 0
2(p - u)2
rU 2
Since ≠ 0, then
2(p - u)2
sin2u + (p - u) sin 2u = 0
sin2u + 2(p - u) sin u cos u = 0
sin u[sin u + 2(p - u) cos u] = 0
Since sin u ≠ 0, then
sin u + 2(p - u) cos u = 0
tan u + 2(p - u) = 0
Solving by trial and error,
180°
u = 1.9760 rad a b = 113.22° = 113° Ans.
p rad
or
180°
u = 4.3072 rad a b = 246.8° = 247° Ans.
p rad

811
© 2014 Pearson Education, Inc., Upper Saddle River, NJ. All rights reserved. This material is protected under all copyright laws as they currently
exist. No portion of this material may be reproduced, in any form or by any means, without permission in writing from the publisher.

7–73.  The Rankine body is defined by the source and sink,


4 m/s y
each having a strength of 0.2 m2 >s. If the velocity of the
uniform flow is 4 m>s, determine the longest and shortest
dimensions of the body.
x

0.5 m 0.5 m

Solution
We consider ideal fluid flow.
The half length of the Rankine oval is
1
q 2

. We or
b = a a + a2 b

m W ina g
Up

b)
ed e n
in
no W iss ea s

itt id tio
is e D t w
t p or em ch
d th g. in t la
1
0.2 m2 >s 2
b = ca b(0.5 m) + ( 0.5 m ) 2 d = 0.5079 m

an on in rs h
(4 m>s)p
k g rn to rig
or in a uc y
w d le tr p

er ld
Thus, the length of the Rankine oval is
e lu nt ns co

Ans.
th inc de f i es

L = 2b = 2(0.5079 m) = 1.02 m
of rk ( stu e o tat

The half width of the Rankine oval can be determined using


ity o g us d S
te is ss th ite

h2 - a2 2pUh
in f th se for Un

h = tan a b
2a q
gr w in e
th t o a ly by

h2 - ( 0.5 m ) 2 2p(4 m>s)h


y ar d le d
ro p an o te

h = tanc d
2(0.5 m) 0.2 m2 >s
st ny s d s ec
de f a rse de ot

s
ill o u vi pr

h = ( h2 - 0.25 ) tan (40ph)


w le co ro is
sa eir is p rk

Solving numerically, and noting that q>2U = 0.2>[2(4)] = 0.025, we start with
th d wo

h = 0.024 and adjust this until we find that


an his

e
T

h = 0.02423 m
Thus, the width of the Rankine oval is
W = 2h = 2(0.02423) = 0.0485 m Ans.

Ans:
L = 1.02 m
0.0485 m

812
© 2014 Pearson Education, Inc., Upper Saddle River, NJ. All rights reserved. This material is protected under all copyright laws as they currently
exist. No portion of this material may be reproduced, in any form or by any means, without permission in writing from the publisher.

7–74.  The Rankine body is defined by the source and sink,


4 m/s y
each having a strength of 0.2 m2 >s. If the velocity of the
uniform flow is 4 m>s, determine the equation in Cartesian
coordinates that defines the boundary of the body.
x

0.5 m 0.5 m

Solution
We consider ideal fluid flow.
The stream function of the flow around the Rankine oval is given by
q 2ay
c = Uy - tan-1a 2 b

. We or
2p x + y2 - a2

m W ina g

b)
ed e n
in
no W iss ea s

itt id tio
Since the boundary of the Rankine oval contains the stagnation point where y = 0,

is e D t w
t p or em ch
d th g. in t la
then this equation gives

an on in rs h
k g rn to rig
q 2(a)(0)
c = U(0) - tan-1a 2 b = 0
or in a uc y
w d le tr p

er ld
2p x + 02 - a2
e lu nt ns co
th inc de f i es

Thus, the equation that describes the boundary of the Rankine oval is
of rk ( stu e o tat

q 2ay
ity o g us d S

Uy - tan-1a 2 b = 0
2p x + y2 - a2
te is ss th ite
in f th se for Un

Here, U = 4 m>s, q = 0.2 m2 >s and a = 0.5 m.


gr w in e
th t o a ly by

0.2 2(0.5)y
y ar d le d

4y - tan-1 c 2 d = 0
ro p an o te

2p x + y2 - 0.52
st ny s d s ec
de f a rse de ot

y
ill o u vi pr

1
4y - tan-1 c 2 d = 0
w le co ro is

10p 2
x + y - 0.25
sa eir is p rk
th d wo

y
= tan 40py Ans.
an his

2 2
x + y - 0.25
T

Ans:
y
2 2
= tan 40py
x + y - 0.25

813
© 2014 Pearson Education, Inc., Upper Saddle River, NJ. All rights reserved. This material is protected under all copyright laws as they currently
exist. No portion of this material may be reproduced, in any form or by any means, without permission in writing from the publisher.

7–75.  A fluid has a uniform velocity of U = 10 m>s. A


source q = 15 m2 >s is at x = 2 m, and a sink q = 15 m2 >s is
at x = 2 m. Graph the Rankine body that is formed, and
determine the magnitudes of the velocity and the pressure
at point (0, 2 m). The pressure within the uniform flow is
40 kPa. Take r = 850 kg>m3.

Solution
We consider ideal fluid flow.
The major and minor axes of the Rankine oval can be determined from
1
q 2
b = a a + a2 b
Up
1
15 m2 >s 2
= ec d (2 m) + (2 m)2 f
(10 m>s)p

. We or
m W ina g

b)
ed e n
in
no W iss ea s

itt id tio
is e D t w
= 2.23 m

t p or em ch
d th g. in t la
h2 - a2 2pUh

an on in rs h
tan a b
k g rn to rig
h =
2a q or in a uc y
w d le tr p

er ld
e lu nt ns co

h2 - (2 m)2 2p ( 10 m>s ) h
th inc de f i es

h = tan c d
2(2 m) 15 m2 >s
of rk ( stu e o tat
ity o g us d S

h2 - 4 4p
te is ss th ite

h = tan a hb
4 3
in f th se for Un
gr w in e
th t o a ly by

Solving numerically, and noting that q>2U = 15> 32(10) 4 = 0.75, we start with
y ar d le d

h = 0.74 and adjust this until we find that


ro p an o te
st ny s d s ec

h = 0.609 m
de f a rse de ot

s
ill o u vi pr

The resulting Rankine oval is shown in Fig. a


w le co ro is
sa eir is p rk

Since the flow around a Rankine oval is irrotational, Bernoulli’s equation can be
th d wo

applied between two points on different streamlines where these two points are
an his

point A, within the uniform flow and point B (0, 2 m). Since the flow occurs in the
T

horizontal plane, the gravity term can be excluded.


Here, VA = U = 10 m>s and the velocity components at B are

q x + a x - a
uB = U + £ - §
2p (x + a)2 + y2 (x - a)2 + y2

15 m2 >s 0 + 2m 0 - 2m
= 10 m>s + a b£ - §
2p (0 + 2 m)2 + (2 m)2 (0 - 2 m)2 + (2 m)2
= 11.19 m>s

q y y
vB = £ 2 2
- §
2p (x + a) + y (x - a)2 + y2

15 m2 >s 2m 2m
= £ 2 2
- §
2p (0 + 2 m) + (2 m) (0 - 2 m)2 + (2 m)2

= 0

814
© 2014 Pearson Education, Inc., Upper Saddle River, NJ. All rights reserved. This material is protected under all copyright laws as they currently
exist. No portion of this material may be reproduced, in any form or by any means, without permission in writing from the publisher.

7–75. Continued

Thus,
VB = u B = 11.19 m>s = 11.2 m>s Ans.
Bernoulli’s equation written between points A and B is
pA VA2 pB VB2
+ = +
r 2 r 2
r
pB = pA + ( VA2 - VB2 )
2

. We or
m W ina g
850 kg>m3

b)
ed e n
in
no W iss ea s
= 40 ( 103 ) Pa + ° ¢ c ( 10 m>s ) 2 - ( 11.19 m>s ) 2 d

itt id tio
is e D t w
t p or em ch
2

d th g. in t la
an on in rs h
= 29.24 ( 103 ) Pa = 29.2 kPa
k g rn to rig
Ans.
or in a uc y
w d le tr p

er ld
e lu nt ns co
th inc de f i es

y (m)
of rk ( stu e o tat
ity o g us d S
te is ss th ite
in f th se for Un

0.609 m
gr w in e
th t o a ly by

x (m)
y ar d le d

0.609 m
ro p an o te
st ny s d s ec
de f a rse de ot

s
ill o u vi pr

2.23 m 2.23 m
w le co ro is

(a)
sa eir is p rk
th d wo
an his

e
T

Ans:
h = 0.609 m
V = 11.2 m>s
p = 29.2 kPa

815
© 2014 Pearson Education, Inc., Upper Saddle River, NJ. All rights reserved. This material is protected under all copyright laws as they currently
exist. No portion of this material may be reproduced, in any form or by any means, without permission in writing from the publisher.

*7–76.  Integrate the pressure distribution, Eq. 7–67, over


the surface of the cylinder in Fig. 7–33b, and show that the
resultant force is equal to zero.

Solution
We consider ideal fluid flow.
The pressure distribution around a cylinder is given by

1 2
p = p0 + rU ( 1 - 4 sin2 u )
2
The force that the pressure exerts on the differential area dA = (adu)L = aLdu is

1 1
dF = pdA = c p0 + rU 2 ( 1 - 4 sin2 u ) d (aLdu) = p0aLdu + raLU 2 ( 1 - 4 sin2 u ) du
2 2

. We or
m W ina g

b)
ed e n
in
no W iss ea s
Equating the resultant forces along the x and y axes shown in Fig. a,

itt id tio
is e D t w
t p or em ch
d th g. in t la
2p

L0
an on in rs h
+
S ( FR ) x = ΣFx;   
( FR ) x = - dF cos u

k g rn to rig
or in a uc y
w d le tr p

er ld
e lu nt ns co
2p

L0
1
= - c p0aLdu + raLU 2 ( 1 - 4 sin2 u ) du d cos u
th inc de f i es

2
of rk ( stu e o tat
ity o g us d S

2p 2p

L0 L0 2
1
= - p0aL cos udu - raLU 2 ( cos u - 4 sin2 u cos u ) du
te is ss th ite
in f th se for Un
gr w in e

1 4 sin3 u 2p
th t o a ly by

= - p0aL(sin u) 2p
0 - raLU 2 asin u - b`
y ar d le d

2 3 0
ro p an o te
st ny s d s ec

= 0
de f a rse de ot

s
ill o u vi pr

2p

L0
w le co ro is

+ c ( FR ) y = ΣFy;     ( FR ) y = - dF sin u
sa eir is p rk
th d wo

2p

L0
1
an his

= - c p0aLdu + raLU 2 ( 1 - 4 sin2 u ) du d sin u


T

2
2p 2p

L0 L0 2
1
= - p0aL sin udu - raLU 2 ( sin u - 4 sin3 u ) du

2p
1 4
= - p0aL ( cos u ) 2p
0 - raLU 2 c -cos u - c - cos u ( sin2 u + 2 ) d d `
2 3 0

= 0

Therefore,

FR = 2 ( FR ) x2 + ( FR ) y2 = 0 (Q.E.D.)

(FR)y

ds = ad
dF
a
d
(FR)x

(a)

816
© 2014 Pearson Education, Inc., Upper Saddle River, NJ. All rights reserved. This material is protected under all copyright laws as they currently
exist. No portion of this material may be reproduced, in any form or by any means, without permission in writing from the publisher.

7–77.  The tall rotating cylinder is subjected to a uniform


horizontal airflow of 3 ft>s. If the radius of the cylinder is
4 ft, determine the location of the stagnation points and the
lift per unit length. The circulation around the cylinder is 3 ft/s
18 ft 2 >s. Take r = 2.35 1 10-3 2 slug>ft 3.
4 ft

Solution
We consider ideal fluid flow.
The lift can be determined by using

Fy = rUΓ = 3 2.35 ( 10-3 ) slug>ft3 4 ( 3 ft>s )( 18 ft2 >s )


= 0.127 lb>ft Ans.

For this case,

Γ 18 ft 2 >s
sin u = = = 0.1194
4p ( 3 ft>s ) (4 ft)

. We or
4pUa

m W ina g

b)
ed e n
in
no W iss ea s
u = 6.86° and 173° Ans.

itt id tio
is e D t w
t p or em ch
d th g. in t la
Since the solution has two roots, there are two stagnation points on the surface of

an on in rs h
k g rn to rig
the cylinder.
or in a uc y
w d le tr p

er ld
e lu nt ns co
th inc de f i es
of rk ( stu e o tat
ity o g us d S
te is ss th ite
in f th se for Un
gr w in e
th t o a ly by
y ar d le d
ro p an o te
st ny s d s ec
de f a rse de ot

s
ill o u vi pr
w le co ro is
sa eir is p rk
th d wo
an his

e
T

Ans:
Fy = 0.127 lb>ft
u = 6.86° and 173°

817
© 2014 Pearson Education, Inc., Upper Saddle River, NJ. All rights reserved. This material is protected under all copyright laws as they currently
exist. No portion of this material may be reproduced, in any form or by any means, without permission in writing from the publisher.

7–78.  The 0.5-m-diameter bridge pier is subjected to the


uniform flow of water at 4 m>s. Determine the maximum
and minimum pressures exerted on the pier at a depth of 2 m.

Solution
We consider ideal fluid flow.
For the flow around a cylinder, the pressure at any point on the boundary can be
determined by using

1
p = p0 + rU 2 ( 1 - 4 sin2 u )

. We or
2

m W ina g

b)
ed e n
in
no W iss ea s

itt id tio
is e D t w
t p or em ch
At the depth of h = 2 m,

d th g. in t la
an on in rs h
p0 = rgh = ( 1000 kg>m3 )( 9.81 m>s2 ) (2 m) = 19.62 ( 103 ) Pa
k g rn to rig
or in a uc y
w d le tr p

er ld
dp
e lu nt ns co

The pressure extremes occur when = 0. Thus,


th inc de f i es

du
of rk ( stu e o tat

dp
ity o g us d S

1
= 0 + rU 2(0 - 8 sin u cos u) = 0
du 2
te is ss th ite
in f th se for Un

8 sin u cos u = 0
gr w in e
th t o a ly by

4 sin 2u = 0
y ar d le d
ro p an o te

Solving,
st ny s d s ec
de f a rse de ot

s
ill o u vi pr

u = 0, 90°, 180°...
w le co ro is

The maximum pressure occurs when u = 0° or 180°. Thus,


sa eir is p rk
th d wo

1
( 1000 kg>m3 )( 4 m>s ) 2 3 1 - 4 sin2 0° 4
an his

pmax = 19.62 ( 103 ) Pa +


e

2
T

= 27.62 ( 103 ) Pa = 27.6 kPa Ans.

The minimum pressure occurs when u = 90°. Thus,


1
pmin = 19.62 ( 103 ) Pa + ( 1000 kg>m3 ) (4 m>s)2 31 - 4 sin2 90°4
2
= - 4.38 ( 103 ) Pa = - 4.38 kPa Ans.

The negative sign indicates that suction occurs.

Ans:
pmax = 27.6 kPa
pmin = - 4.38 kPa

818
© 2014 Pearson Education, Inc., Upper Saddle River, NJ. All rights reserved. This material is protected under all copyright laws as they currently
exist. No portion of this material may be reproduced, in any form or by any means, without permission in writing from the publisher.

7–79.  Air flows around the cylinder such that the pressure,
measured at A, is pA = - 4 kPa. Determine the velocity U
of the flow if r = 1.22 kg>m3. Can this velocity be A
determined if instead the pressure at B is measured? 45
U B

0.2 m

Solution
We consider ideal fluid flow.
The pressure at a point removed from the cylinder is atmospheric. Thus, p0 = 0.
At point A, u = 45°

1
pA = p0 + rU 2 ( 1 - 4 sin2 u )
2
1
-4 ( 103 ) N>m2 = 0 + ( 1.22 kg>m2 )( U 2 )( 1 - 4 sin2 45°)
2

. We or
U = 80.98 m>s = 81.0 m>s Ans.

m W ina g

b)
ed e n
in
no W iss ea s

itt id tio
is e D t w
t p or em ch
If the pressure is measured at B, the velocity of the uniform flow can be determined

d th g. in t la
using the same equation with u = 180°.

an on in rs h
k g rn to rig
or in a uc y
w d le tr p

er ld
e lu nt ns co
th inc de f i es
of rk ( stu e o tat
ity o g us d S
te is ss th ite
in f th se for Un
gr w in e
th t o a ly by
y ar d le d
ro p an o te
st ny s d s ec
de f a rse de ot

s
ill o u vi pr
w le co ro is
sa eir is p rk
th d wo
an his

e
T

Ans:
81.0 m>s; yes

819
© 2014 Pearson Education, Inc., Upper Saddle River, NJ. All rights reserved. This material is protected under all copyright laws as they currently
exist. No portion of this material may be reproduced, in any form or by any means, without permission in writing from the publisher.

*7–80.  The 200-mm-diameter cylinder is subjected to a


uniform horizontal flow having a velocity of 6 m>s. At a 6 m/s
distance far away from the cylinder, the pressure is 150 kPa.
Plot the variations of the velocity and pressure along the
r
radial line r, at u = 90°, and specify their values at r = 0.1 m,
u
0.2 m, 0.3 m, 0.4 m, and 0.5 m. Take r = 1.5 Mg>m3.
0.1 m

Solution
We consider ideal fluid flow.
The r and u components of velocity of the uniform flow around a cylinder can be r(m)
determined using
0.5
a 2 (0.1 m)2 0.06 0.4
vr = U a1 - b cos u = ( 6 m>s ) £ 1 - § cos u = c a6 - b cos u d m>s
r2 r2 r2
0.3
2
a2 (0.1 m) 0.06 0.2
vu = - U a1 + b sin u = -(6 m>s) c 1 + d sin u = c - a6 + 2 b sin u d m>s

. We or
2
r r2 r

m W ina g
0.1

b)
ed e n
in
no W iss ea s

itt id tio
is e D t w
t p or em ch
When u = 90°, v(m s)

d th g. in t la
0 6.24 12

an on in rs h
0.06 7.5

k g rn to rig
vr = 0  vu = - a6 + b m>s
or in a uc y 6.375 6.67
r2
w d le tr p

er ld
e lu nt ns co
(a)
Thus, the magnitude of the velocity when u = 90° is
th inc de f i es

r(m)
of rk ( stu e o tat

0.06 2 0.06
ity o g us d S

V = 2vr2 + vu2 2
= 0 + c - a6 + 2 b = a6 + 2 b m>s (1)
B r r 0.5
te is ss th ite
in f th se for Un

Flow around a cylinder is irrotational since the potential function exists. Therefore, 0.4
gr w in e
th t o a ly by

the Bernoulli equation is applicable. Neglecting the elevation terms,


0.3
y ar d le d
ro p an o te

pO VO2 p V2
st ny s d s ec

+ = + 0.2
de f a rse de ot

r 2 r 2
s
ill o u vi pr

N 0.06 2 0.1
w le co ro is

150 ( 103 ) a6 + b
( 6 m>s ) 2
sa eir is p rk

m2 p r2 p(kPa)
th d wo

+ = +
1500 kg>m3 2 1500 kg>m3 2 0 69 135
148
an his

144
0.06 2 147
T

p = c 177 ( 103 ) - 750 a6 + b d Pa (2) (b)


r2
The values of V and P at r = 0.1 m, 0.2 m, 0.3 m, 0.4 m, and 0.5 m can be evaluated
using Eqs. 1 and 2, respectively, and are tabulated below.
r(m) 0.1 0.2 0.3 0.4 0.5
V ( m>s )
12 7.5 6.67 6.375 6.24 Ans.
Eq. (1)
p(kPa)
69 135 144 147 148
Eq. (2)

The plot of V vs. r and p vs. r are shown in Figs. a and b.

820
© 2014 Pearson Education, Inc., Upper Saddle River, NJ. All rights reserved. This material is protected under all copyright laws as they currently
exist. No portion of this material may be reproduced, in any form or by any means, without permission in writing from the publisher.

7–81.  The 200-mm-diameter cylinder is subjected to a


uniform flow having a velocity of 6 m>s. At a distance far 6 m/s
away from the cylinder, the pressure is 150 kPa. Plot the
variation of the velocity and pressure along the radial line r,
r
at u = 0°, and specify their values at r = 0.1 m, 0.2 m, 0.3 m,
u
0.4 m, and 0.5 m. Take r = 1.5 Mg>m3.
0.1 m

Solution
We consider ideal fluid flow.
The r and u components of velocity of the uniform flow around a cylinder can be v(m s)
determined using 5.76
5.625
a2 (0.1 m)2 0.06
vr = U a1 - 2
b cos u = ( 6 m>s ) c 1 - d cos u = c a6 - 2 b cos u d m>s 5.33
r r2 r 4.50

a2 (0.1 m)2 0.06


vu = - U a1 + 2
bsin u = - ( 6 m>s ) c 1 + 2
d sin u = c - a6 + 2 b sin u d m>s
r r r

. We or
When u = 0°,

m W ina g

b)
ed e n
in
no W iss ea s

itt id tio
is e D t w
t p or em ch
d th g. in t la
0.06
vr = a6 - b m>s   vu = 0

an on in rs h
r2 r(m)

k g rn to rig
or in a uc y
w d le tr p 0 0.1 0.2 0.3 0.4 0.5

er ld
Thus, the magnitude of the velocity when u = 0° is
e lu nt ns co
(a)
th inc de f i es

0.06 2 0.06
V = 2vr2 + vu2 = a6 - b + 02 = a6 - 2 b m>s (1)
of rk ( stu e o tat

B r2 r
ity o g us d S

P(kPa)
162 177
te is ss th ite

The flow around a cylinder is irrotational since the potential function exists. Therefore,
in f th se for Un

the Bernoulli equation is applicable. Neglecting the elevation terms, 156


gr w in e
th t o a ly by

153
pO VO2 p V2 152
y ar d le d

+ = +
ro p an o te

r 2 r 2
st ny s d s ec
de f a rse de ot

0.06 2
s

N
ill o u vi pr

150 ( 103 ) a6 - b
( 6 m>s ) 2
w le co ro is

m2 p r2
+ = +
sa eir is p rk

1500 kg>m3 2 1500 kg>m3 2


th d wo
an his

0.06 2
p = c 177 ( 103 ) - 750 a6 - b d Pa (2) r(m)
T

r2 0 0.1 0.2 0.3 0.4 0.5


(b)
The values of V and P at r = 0.1 m, 0.2 m, 0.3 m, 0.4 m, and 0.5 m can be evaluated
using Eqs. 1 and 2, respectively, and are tabulated below.
r(m) 0.1 0.2 0.3 0.4 0.5

V(m>s) 0 4.50 5.33 5.625 5.76 Ans.

p(kPa) 177 162 156 153 152


The plot of V vs. r and p vs. r are shown in Figs. a and b.

Ans:
r (m) 0.1 0.2 0.3 0.4 0.5
V (m>s) 0 4.50 5.33 5.625 5.76
p (kPa) 177 162 156 153 152

821
© 2014 Pearson Education, Inc., Upper Saddle River, NJ. All rights reserved. This material is protected under all copyright laws as they currently
exist. No portion of this material may be reproduced, in any form or by any means, without permission in writing from the publisher.

7–82.  Air is flowing at U = 30 m>s past the Quonset hut y


of radius R = 3 m. Find the velocity and absolute pressure
distribution along the y axis for 3 m … y … ∞ . The U
absolute pressure within the uniform flow is p0 = 100 kPa.
A
Take ra = 1.23 kg>m3.

R
u
x
Solution
We consider ideal fluid flow.
The velocity components of the flow around the building are

a2 a2
vr = U a1 - bcos u    vu = - U a1 + bsin u
r2 r2
Here U = 30 m>s and a = 3 m. Along the y axis u = 90°. Then

(3 m)2
vr = ( 30 m>s ) c 1 - d cos 90° = 0
r2

. We or
m W ina g

b)
ed e n
in
no W iss ea s
(3 m)2

itt id tio
is e D t w
vu = - ( 30 m>s ) c 1 +

t p or em ch
d sin 90°

d th g. in t la
r2

an on in rs h
k g rn to rig
9
= - 30 a1 + bm>s or in a uc y
r2
w d le tr p

er ld
e lu nt ns co
th inc de f i es

Thus, the velocity distribution along the y axis is


of rk ( stu e o tat
ity o g us d S

9
V = vu = 30 a1 + b m>s Ans.
r2
te is ss th ite
in f th se for Un

Since the flow is irrotational, Bernoulli’s equation can be applied between two
gr w in e
th t o a ly by

points on different streamlines. Here they are point O within the uniform flow and
y ar d le d
ro p an o te

a point along the y axis,


st ny s d s ec
de f a rse de ot

p0 VO2 p V2
s
ill o u vi pr

+ + gz0 = + + gz
ra 2 ra 2
w le co ro is
sa eir is p rk

Since the density of air is small, the gravitational terms can be neglected. Here,
th d wo

V0 = U = 30 m>s. Then
an his

e
T

9 2
c 30 a1 + bd
100 ( 103
) N>m2
( 30 m>s ) 2
p r2
+ + 0 = + + 0
1.23 kg>m3 2 1.23 kg>m3 2

9 2
p = c 100 553.5 - 553.5 a1 + b d Pa Ans.
r2
Notice that at r = ∞ ,

p = 100 553.5 - 553.5(1 + 0)2


= 100 ( 103 ) Pa = p0

Ans:
9
V = 30 a1 + b m>s
r2
9 2
p = c 100 553.5 - 553.5 a1 + b d Pa
r2

822
© 2014 Pearson Education, Inc., Upper Saddle River, NJ. All rights reserved. This material is protected under all copyright laws as they currently
exist. No portion of this material may be reproduced, in any form or by any means, without permission in writing from the publisher.

7–83.  The Quonset hut of radius R is subjected to a y


uniform wind having a velocity U. Determine the resultant
vertical force caused by the pressure that acts on the hut if U
it has a length L. The density of air is r.
A

R
u
x
Solution
We consider ideal fluid flow.
For the flow around a cylinder, the pressure distribution on the boundary is
described by

1
rU 2 ( 1 - 4 sin2 u )
p = p0 +
2
Here, p0 is atmospheric pressure. Thus, the net pressure on the boundary is gauge
pressure, which is

. We or
1
pg = p - p0 = rU 2 ( 1 - 4 sin2 u )

m W ina g

b)
ed e n
in
2

no W iss ea s

itt id tio
is e D t w
t p or em ch
d th g. in t la
The force that the gauge pressure exerts on the differential area dA = (Rd u)L

an on in rs h
k g rn to rig
= RLd u is
or in a uc y
w d le tr p

er ld
e lu nt ns co

1 1
dF = pgdA = rU 2 ( 1 - 4 sin2 u ) (RLdu) = rRLU 2 ( 1 - 4 sin2 u ) du
th inc de f i es

2 2
of rk ( stu e o tat

Equating the forces along the y axis shown in Fig. a,


ity o g us d S
te is ss th ite

L0
in f th se for Un

+ c (FR)y = ΣFy; (FR)y = - dFd sin u


gr w in e
th t o a ly by

p
y ar d le d

L 2
1
rRLU 2 ( 1 - 4 sin2 u ) sin u du
ro p an o te

= -
st ny s d s ec
de f a rse de ot

0
s
ill o u vi pr

L0
1
w le co ro is

= - rRLU 2 ( sin u - 4 sin3 u ) du


sa eir is p rk

2
th d wo
an his

1 4 p
= - rRLU 2 c - cos u - c - cos u ( sin2 u + 2 ) d d
T

2 3 0

5
= rRLU 2 Ans.
3

(FR(y
ds = Rd¨
dF

(FR(x R R
= ¨

(a)

Ans:
5
(FR)y = rRLU 2
3

823
© 2014 Pearson Education, Inc., Upper Saddle River, NJ. All rights reserved. This material is protected under all copyright laws as they currently
exist. No portion of this material may be reproduced, in any form or by any means, without permission in writing from the publisher.

*7–84.  The Quonset hut of radius R is subjected to a y


uniform wind having a velocity U. Determine the speed of
the wind and the gage pressure at point A. The density of U
air is r.
A

R
u
x
Solution
We consider ideal fluid flow.
p
At point A, r = R and u = rad. Thus,
2
a2 R2 p
vr = U a1 - bcos u = U a1 - bcos = 0
r2 R2 2
a2 R2 p
vu = - U a1 + bsin u = - U a1 + bsin = -2U
r2 R2 2

. We or
Thus, the magnitude of the velocity at point A is

m W ina g

b)
ed e n
in
no W iss ea s

itt id tio
is e D t w
t p or em ch
d th g. in t la
VA = 2vr2 + vu2 = 202 + ( - 2U)2 = 2U Ans.

an on in rs h
k g rn to rig
Here, VA is directed towards the positive x axis.
or in a uc y
For the flow around a cylinder, the pressure distribution on the boundary is
w d le tr p

er ld
e lu nt ns co

described by
th inc de f i es
of rk ( stu e o tat

1
p = p0 + rU 2 ( 1 - 4 sin2 u )
ity o g us d S

2
te is ss th ite
in f th se for Un

Here, p0 is atmospheric pressure. Thus, the net pressure on the boundary is gauge
gr w in e

pressure, which is
th t o a ly by
y ar d le d

1
ro p an o te

pg = p - p0 = rU 2 ( 1 - 4 sin2 u )
st ny s d s ec

2
de f a rse de ot

s
ill o u vi pr

p
At point A, u = rad. Then,
w le co ro is

2
sa eir is p rk
th d wo

1 p 3
( pg ) A = rU 2 c 1 - 4 sin2 a b d = - rU 2 Ans.
an his

2 2 2
T

The negative sign indicates that suction occurs at A.

824
© 2014 Pearson Education, Inc., Upper Saddle River, NJ. All rights reserved. This material is protected under all copyright laws as they currently
exist. No portion of this material may be reproduced, in any form or by any means, without permission in writing from the publisher.

7–85.  Water flows toward the circular column with a


uniform speed of 3 ft>s. If the outer radius of the column is A
4 ft, and the pressure within the uniform flow is 6 lb>in2, 6 ft
determine the pressure at point A. Take rw = 1.94 slug>ft 3. 3 ft/s

4 ft

Solution
We consider ideal fluid flow.
The velocity components of the flow around the structure are

a2 a2
vr = Ua1 - 2
bcos u    vu = - U a1 + 2 bsin u
r r

Here U = 3 ft>s, and a = 4 ft. For point A, r = 6 ft and u = 90°. Then


(4 ft)2
vr = ( 3 ft>s ) c 1 - d cos 90° = 0
(6 ft)2

. We or
m W ina g

b)
ed e n
in
no W iss ea s
(4 ft)2

itt id tio
is e D t w
t p or em ch
vu = - ( 3 ft>s ) c 1 + d sin 90° = - 4.333 ft>s

d th g. in t la
(6 ft)2

an on in rs h
k g rn to rig
Thus, the magnitude of the velocity at A is or in a uc y
w d le tr p

er ld
e lu nt ns co

VA = vu = 4.333 ft>s
th inc de f i es
of rk ( stu e o tat

Since the flow is irrotational, Bernoulli’s equation can be applied between two points
ity o g us d S

on different streamlines. Here, they are point O in the uniform flow and point A.
te is ss th ite

p0 VO2 pA VA2
in f th se for Un

+ + gz0 = + + gzA
gr w in e

r 2 r 2
th t o a ly by
y ar d le d

Since the flow is in the horizontal plane, zD = zA = z. Here, V0 = U = 3 ft>s. Then


ro p an o te
st ny s d s ec

p0 V0 2 pA VA2
de f a rse de ot

+ + gz = + + gz
s
ill o u vi pr

r 2 r 2
w le co ro is
sa eir is p rk

r
pA = p0 + ( V0 2 - VA2 )
th d wo

2
an his

lb 12 in 2 1.94 slug>ft 3
= a6 ba 3 ( 3 ft>s ) 2 - ( 4.333 ft>s ) 2 4
T

2
b +
in 1 ft 2
lb 1 ft 2
= a854.52 2
ba b
ft 12 in
= 5.93 psi Ans.

Ans:
5.93 psi

825
© 2014 Pearson Education, Inc., Upper Saddle River, NJ. All rights reserved. This material is protected under all copyright laws as they currently
exist. No portion of this material may be reproduced, in any form or by any means, without permission in writing from the publisher.

7–86.  The tall circular building is subjected to a uniform


wind having a velocity of 150 ft>s. Determine the location u
of the window that is subjected to the smallest pressure.
What is this pressure? Take ra = 0.00237 slug>ft 3.
u

85 ft

Solution
We consider ideal fluid flow.
This is a case of flow around a cylinder where the velocity components are

a2 a2
vr = U a1 - 2
b cos u  vu = - U a1 + 2 b sin u
r r
On the surface of the building where r = a = 85 ft,

. We or
vr = 0  vu = - 2U sin u

m W ina g

b)
ed e n
in
no W iss ea s

itt id tio
is e D t w
t p or em ch
d th g. in t la
Thus, the velocity of the wind on the surface of the building is

an on in rs h
k g rn to rig
V = vu = - 2U sin u
or in a uc y
w d le tr p

er ld
e lu nt ns co

The minimum pressure occurs at the point where the magnitude of velocity is
maximum, that is when
th inc de f i es
of rk ( stu e o tat

sin u = 1 or  sin u = - 1


ity o g us d S
te is ss th ite

Then u = 90° or 270° Ans.


in f th se for Un
gr w in e

Therefore
th t o a ly by
y ar d le d

Vmax = 2U = 2 ( 150 ft>s ) = 300 ft>s


ro p an o te
st ny s d s ec

Since the flow is irrotational, Bernoulli’s equation can be applied between two
de f a rse de ot

s
ill o u vi pr

points on different streamlines, such as between a point within the uniform flow
w le co ro is

and a point on the building. Since the flow occurs in the horizontal plane, the gravity
sa eir is p rk

term can be excluded.


th d wo
an his

V0 2 2
e

p0 pmin Vmax
+ = +
T

r 2 r 2

Here p0 = 0 and V0 = U = 150 ft>s. Then

( 150 ft>s ) 2 pmin ( 300 ft>s ) 2


0 + = 3
+
2 0.00237 slug>ft 2

lb 1 ft 2
pmin = a - 79.99 2
ba b = - 0.555 psi Ans.
ft 12 in

Ans:
u = 90° or 270°
pmin = - 0.555 psi

826
© 2014 Pearson Education, Inc., Upper Saddle River, NJ. All rights reserved. This material is protected under all copyright laws as they currently
exist. No portion of this material may be reproduced, in any form or by any means, without permission in writing from the publisher.

7–87.  The tall circular building is subjected to a uniform


wind having a velocity of 150 ft>s. Determine the pressure
and the velocity of the wind on its walls at u = 0°, 90°, and
150°. Take ra = 0.00237 slug>ft 3.
u

85 ft

Solution
We consider ideal fluid flow.
This is a case of flow around a cylinder where the velocity components are

a2 a2
vr = U a1 - b cos u  vu = - U a1 + b sin u
r2 r2
On the surface of the building where r  =  a  =  85 ft,

. We or
vr = 0  vu = - 2U sin u

m W ina g

b)
ed e n
in
no W iss ea s

itt id tio
is e D t w
t p or em ch
d th g. in t la
Thus, the velocity of the wind on the surface of the building is

an on in rs h
k g rn to rig
V = vu = - 2U sin u
or in a uc y
w d le tr p

er ld
At u = 0°, 90° and 150°,
e lu nt ns co
th inc de f i es

V  u = 0° = 2 ( 150 ft>s ) sin 0° = 0 Ans.


of rk ( stu e o tat
ity o g us d S

V  u = 90° = 2 ( 150 ft>s ) sin 90° = 300 ft>s Ans.


te is ss th ite

V  u = 150° = 2 ( 150 ft>s ) sin 150° = 150 ft>s Ans.


in f th se for Un
gr w in e
th t o a ly by

Since the flow is irrotational, Bernoulli’s equation can be applied between two
y ar d le d

points on different streamlines, such as between a point within the uniform flow
ro p an o te
st ny s d s ec

and a point on the building. Since the flow occurs in the horizontal plane, the gravity
de f a rse de ot

term can be excluded.


s
ill o u vi pr
w le co ro is

p0 V0 2 p V2
sa eir is p rk

+ = +
2 2
th d wo

r r
an his

p 2
p = p0 + ( V0 - V 2 )
T

2
Here p0 = 0 and V0 = 150 ft>s. Then
0.00237 slug>ft 3
P  u=0° = 0 + ° ¢ 3 ( 150 ft>s ) 2 - 0 4
2

lb 1ft 2
= a26.66 2
ba b = 0.185 psi Ans.
ft 12 in

0.00237 slug>ft 3
P  u=90° = 0 + ° ¢ 3 ( 150 ft>s ) 2 - ( 300 ft>s ) 2 4
2

lb 1ft 2
= a - 79.99 2
ba b = - 0.555 psi Ans. Ans:
ft 12 in
V  u = 0° = 0
V  u = 90° = 300 ft>s
0.00237 slug>ft 3 V  u = 150° = 150 ft>s
Pu = 150° = 0 + ° ¢ 3 ( 150 ft>s ) 2 - ( 150 ft>s ) 2 4 = 0 Ans. p u = 0° = 0.185
2
p u = 90° = - 0.555 psi
p u = 150° = 0

827
© 2014 Pearson Education, Inc., Upper Saddle River, NJ. All rights reserved. This material is protected under all copyright laws as they currently
exist. No portion of this material may be reproduced, in any form or by any means, without permission in writing from the publisher.

*7–88.  The pipe is built from four quarter segments that


8 m/s
are glued together. If it is exposed to a uniform air flow
having a velocity of 8 m>s, determine the resultant force the B
90
pressure exerts on the quarter segment AB per unit length
of the pipe. Take r = 1.22 kg>m3. A

0.1 m

Solution
We consider ideal fluid flow.
Here, the pressure at point O removed from the cylinder is atmospheric. Thus, the
net pressure on the surface of the cylinder is the gauge pressure given by
1
pg = p - p0 = rU 2 ( 1 - 4 sin2 u )
2
(FR)y
The force that the gauge pressure exerts on the differential area dA = (adu)(1) = adu is ds = ad¨
1 1 dF
dF = pgdA = rU 2 ( 1 - 4 sin2 u )( adu ) = raU 2 ( 1 - 4 sin2 u ) du
2 2 a
(FR)x
=

. We or
Equating the forces along the x and y axes shown in Fig. a,

m W ina g

b)
ed e n
in
no W iss ea s
¨

itt id tio
p (a)

is e D t w
Lp>2

t p or em ch
+ ( FR ) x = ΣFx;    ( FR ) x =

d th g. in t la
d dF cos u

an on in rs h
k g rn to rig
p or in a uc y
Lp>2 2
1
w d le tr p
raU 2 ( 1 - 4 sin2 u ) cos udu

er ld
=
e lu nt ns co
th inc de f i es
of rk ( stu e o tat

Lp>2 2
1
= raU 2 ( cos u - 4 sin2 u cos u ) du
ity o g us d S
te is ss th ite
in f th se for Un

1 4 p
= raU 2 asin u - sin3 u b `
gr w in e
th t o a ly by

2 3 p>2
y ar d le d
ro p an o te

1
st ny s d s ec

= raU 2
6
de f a rse de ot

s
ill o u vi pr

Lp>2
w le co ro is

+ c ( FR ) y = ΣFy;   ( FR ) y = - dF sinu
sa eir is p rk
th d wo

Lp>2
an his

1
e

= - raU 2 ( 1 - 4 sin2 u ) sinudu


T

2
p

Lp>2 2
1
= - raU 2 ( sin u - 4 sin3 u ) du

1 4 p
= - raU 2 c - cos u - c - cos u ( sin2 u + 2 ) d d `
2 3 p>2

5
= raU 2
6

Here, U = 8 m>s, r = 1.22 kg>m3, and a = 0.1 m,


1
( FR ) x = ( 1.22 kg>m3 )( 0.1 m )( 8 m>s ) 2 = 1.3013 N>m
6
5
( FR ) y = ( 1.22 kg>m3 )( 0.1m )( 8 m>s ) 2 = 6.507 N>m
6
Thus, per unit length the magnitude of the resultant force on the quarter-segments is

FR = 2 ( FR ) x2 + ( FR ) y2 = 2 ( 1.3013 N ) 2 + ( 6.507 N ) 2
= 6.64 N>m outward (suction) Ans.

828
© 2014 Pearson Education, Inc., Upper Saddle River, NJ. All rights reserved. This material is protected under all copyright laws as they currently
exist. No portion of this material may be reproduced, in any form or by any means, without permission in writing from the publisher.

7–89.  The 1-ft-diameter cylinder is rotating at v = 5 rad>s


4 ft/s A
while it is subjected to a uniform flow having a velocity of
4 ft>s. Determine the lift force on the cylinder per unit 1 ft B
length. Take r = 2.38 1 10-3 2 slug>ft 3.
v u  90

0.5 ft

Solution
For the corresponding free vortex, at r = 0.5 ft,

vu = vr = ( 5 rad>s )( 0.5 ft ) = 2.5 ft>s

vr = 0

Thus, the circulation around the cylinder can be determined using


2p 2p
V # ds =
C L0 L0
Γ = vur du = ( 2.5 ft>s )( 0.5 ft ) du = 1.25u  2p 2
0 = 2.5p ft >s

. We or
m W ina g
The uplift force on the cylinder can be determined using

b)
ed e n
in
no W iss ea s

itt id tio
is e D t w
t p or em ch
d th g. in t la
Fy = rUΓ = 3 2.38 ( 10-3 ) slug>ft3 4 ( 4 ft>s )( 2.5p ft2 >s )

an on in rs h
k g rn to rig
= 0.0748 lb>ft or in a uc y
w d le tr p Ans.

er ld
e lu nt ns co
th inc de f i es
of rk ( stu e o tat
ity o g us d S
te is ss th ite
in f th se for Un
gr w in e
th t o a ly by
y ar d le d
ro p an o te
st ny s d s ec
de f a rse de ot

s
ill o u vi pr
w le co ro is
sa eir is p rk
th d wo
an his

e
T

Ans:
0.0748 lb>ft

829
© 2014 Pearson Education, Inc., Upper Saddle River, NJ. All rights reserved. This material is protected under all copyright laws as they currently
exist. No portion of this material may be reproduced, in any form or by any means, without permission in writing from the publisher.

7–90.  The 1-ft-diameter cylinder is rotating at v = 8 rad>s


4 ft/s A
while it is subjected to a flow having a uniform horizontal
velocity of 4 ft>s. If the pressure within the uniform flow is 1 ft B
80 lb>ft 2, determine the pressure on the surface B of the
cylinder at u = 90°, and at A, where r = 1 ft, u = 90°. Also v u  90
find the resultant force acting per unit length of the cylinder.
0.5 ft
Take r = 1.94 slug>ft 3.

Solution
We consider ideal fluid flow.
For the corresponding free vortex, at r = 0.5 ft, vu = vr = ( 8 rad>s ) (0.5 ft) = 4 ft>s.
Thus, the circulation is
2p 2p
V # ds =
C L0 L0
Γ = v0 ( rdu ) = 4 ( 0.5 du ) = 2 ( 2p ) = 4p ft 2 >s.

The velocity components of the flow around the cylinder are


a2 a2 Γ
vr = U a1 - 2
b cos u   vu = - U a1 + 2
b sin u +
r r 2pr

. We or
m W ina g
Here, U = 4 ft>s, a = 0.5 ft and u = 90°. For point A, rA = 1 ft. Then

b)
ed e n
in
no W iss ea s

itt id tio
is e D t w
t p or em ch
d th g. in t la
( 0.5 ft ) 2
( vr ) A = ( 4 ft>s ) £ 1 - § cos 90° = 0

an on in rs h
k g rn to rig
( 1 ft ) 2
or in a uc y
w d le tr p

er ld
e lu nt ns co

( 0.5 ft ) 2 4p ft 2 >s
th inc de f i es

( vu ) A = - ( 4 ft>s ) £ 1 + § sin 90° + = -3 ft>s


( 1 ft ) 2 2p ( 1 ft )
of rk ( stu e o tat
ity o g us d S

Thus, VA = ( vu ) A = 3 ft>s.
te is ss th ite
in f th se for Un

For point B, on the surface, (vr)B = 0


gr w in e
th t o a ly by

( 0.5 ft ) 2 4p ft 2 >s
( vu ) B = - ( 4ft>s ) £ 1 +
y ar d le d

§ sin 90° + = -4ft>s


ro p an o te

( 0.5 ft ) 2 2p (0.5 ft)


st ny s d s ec
de f a rse de ot

Thus, VB = ( vu ) B = 4 ft>s.
s
ill o u vi pr
w le co ro is

Since the flow is irrotational, Bernoulli’s equation can be applied between two
sa eir is p rk
th d wo

points located on the different streamlines. Here these two points are one within the
uniform flow and the other one is point A (or B)
an his

v02
T

p0 p v2
+ + gz0 = + + gz
r 2 r 2
Since the flow occurs in the horizontal plane, z0 = z
Here, V0 = U, Thus
r
p = p0 + (U 2 - V2)
2
Here, U = 4 ft>s; p0 = 80 lb>ft 2. At point A,
1.94 slug>ft 3
pA = 80 lb>ft 2 + 3 ( 4 ft>s ) 2 - ( 3 ft>s ) 2 4
2
= 86.8 lb>ft 2 Ans.
At point B,
1.94 slug>ft 3
pB = 80 lb>ft 2 + 3 ( 4ft>s ) 2 - ( 4ft>s ) 2 4
2
= 80 lb>ft 2 Ans.
The resultant force acting on the cylinder is
Fy = - rUΓ = - ( 1.94 slug>ft 3 )( 4 ft>s )( 4p ft 2 >s ) Ans:
= - 97.5 lb>ft pA = 86.8 lb>ft 2
pB = 80 lb>ft 2
= 97.5 lb>ft T Fy = 97.5 lb>ft

830
© 2014 Pearson Education, Inc., Upper Saddle River, NJ. All rights reserved. This material is protected under all copyright laws as they currently
exist. No portion of this material may be reproduced, in any form or by any means, without permission in writing from the publisher.

7–91.  The cylinder rotates counterclockwise at 40 rad>s. If


the uniform velocity of the air is 10 m>s, and the pressure
within the uniform flow is 300 Pa, determine the maximum
and minimum pressure on the surface of the cylinder. Also, 0.6 m
what is the lift force on the cylinder? Take ra = 1.20 kg>m3. 10 m/s

40 rad/s

Solution
We consider ideal fluid flow.
For the corresponding free vortex at r = 0.6 m, vu = vr = ( 40 rad>s ) (0.6 m)
= 24 m>s. Thus, the circulation is

. We or
2p 2p
V # ds =
C L0 L0

m W ina g
24(0.6du) = 28.8p m2 >s

b)
Γ = vu(rdu) =

ed e n
in
no W iss ea s

itt id tio
is e D t w
t p or em ch
d th g. in t la
Since Γ 7 4pUa = 4p ( 10 m>s ) (0.6 m) = 24p m2 >s, the stagnation point will not

an on in rs h
k g rn to rig
be on the surface of the wheel. The pressure at a point on the surface is
or in a uc y
w d le tr p

er ld
e lu nt ns co
2
1 2 Γ
p = p0 + rU c 1 - a - 2 sin u + b d
th inc de f i es

2 2pUa
of rk ( stu e o tat

Γ
ity o g us d S

Since Γ 7 4pUa, the term - 2 sin u + is the smallest when u = 90°, which
2p va
te is ss th ite
in f th se for Un

yields the maximum pressure. Thus,


gr w in e
th t o a ly by

2
1 2 Γ
p max = p0 + rU c 1 - a - 2 + b d
y ar d le d
ro p an o te

2 2pUa
st ny s d s ec
de f a rse de ot

1 28.8p m2 >s 2
s
ill o u vi pr

= 300 Pa + ( 1.20 kg>m3 )( 10 m>s ) 2 e 1 - c -2 + d f


2 2p ( 10 m>s ) (0.6 m)
w le co ro is
sa eir is p rk
th d wo

= 350 Pa Ans.
an his

Also, we notice that the minimum pressure occurs at a point where u = 90°. Then
T

2
1 2 Γ
p min = p0 + rU c 1 - a2 + b d
2 2pUa

1 28.8p m2 >s 2
= 300 Pa + ( 1.20 kg>m3 )( 10 m>s ) 2 e 1 - c 2 + d f
2 2p ( 10 m>s ) (0.6 m)

= -802 Pa Ans.

Ans:
pmax = 350 Pa
pmin = - 802 Pa

831
© 2014 Pearson Education, Inc., Upper Saddle River, NJ. All rights reserved. This material is protected under all copyright laws as they currently
exist. No portion of this material may be reproduced, in any form or by any means, without permission in writing from the publisher.

*7–92.  A torque T is applied to the cylinder, causing it to


rotate counterclockwise with a constant angular velocity of 120 rev/min
120 rev>min. If the wind is blowing at a constant speed of
15 m>s, determine the location of the stagnation points on 200 mm
the surface of the cylinder, and find the maximum pressure.
The pressure within the uniform flow is 400 Pa. Take 15 m/s
ra = 1.20 kg>m3. T

Solution
We consider ideal fluid flow.
For the corresponding free vortex at r = 0.2 m, vu = vr
rev 2p rad 1 m
= c a120 ba ba b d (0.2 m) = 0.8p m>s . Thus, the circulation of this

. We or
min 1 rev 60 s

m W ina g

b)
ed e n
in
no W iss ea s
free vortex is

itt id tio
is e D t w
t p or em ch
d th g. in t la
an on in rs h
2p 2p
V # ds =
C L0 L0 k g rn to rig
Γ = vu(rdu) = (0.8 p)(0.2 du) = 0.32p2 m2 >s
or in a uc y
w d le tr p

er ld
e lu nt ns co

Since Γ 6 4pUa = 4p ( 15 m>s ) (0.2 m) = 12p m2 >s, there exist two stagnation
th inc de f i es
of rk ( stu e o tat

points on the surface. The location of these two points can be found using
ity o g us d S

Γ 0.32p2 m2 >s
te is ss th ite

sin u = = = 0.08378
4pUa 4p ( 15 m>s ) (0.2 m)
in f th se for Un
gr w in e
th t o a ly by

u = 4.806° = 4.81° or u = 175.19° = 176° Ans.


y ar d le d

The maximum pressure occurs at the stagnation point where V = 0. Since the flow
ro p an o te
st ny s d s ec

is irrotational, Bernoulli’s equation can be applied between two points on different


de f a rse de ot

streamlines, such as one within the uniform flow and the other one at the stagnation
s
ill o u vi pr

point. Here, the gravity term can be neglected since the flow involves air which has
w le co ro is
sa eir is p rk

a low density.
th d wo

p0 V0 2 p V2
an his

+ = +
ra 2 ra 2
T

Here V0 = U = 15 m>s, p0 = 400 Pa, p = p max and V = 0. Then

400 N>m2 ( 15 m>s ) 2 p max


3
+ = + 0
1.20 kg>m 2 1.20 kg>m3

p max = 535 Pa Ans.

832
© 2014 Pearson Education, Inc., Upper Saddle River, NJ. All rights reserved. This material is protected under all copyright laws as they currently
exist. No portion of this material may be reproduced, in any form or by any means, without permission in writing from the publisher.

7–93.  A torque T is applied to the cylinder, causing it to


rotate counterclockwise with a constant angular velocity of
120 rev>min. If the wind is blowing at a constant speed of 120 rev/min
15 m>s, determine the lift per unit length on the cylinder 200 mm
and the minimum pressure on the cylinder. The pressure
within the uniform flow is 400 Pa. Take ra = 1.20 kg>m3.
15 m/s
T

Solution
We consider ideal fluid flow.
For the corresponding free vortex at r = 0.2 m, vu = vr
rev 2p rad 1 min
= c a120 ba ba b d (0.2 m) = 0.8p m>s . Thus, the circulation of
min 1 rev 60 s
this free vortex is
2p 2p
V # ds =
C L0 L0

. We or
Γ = vu(r du) = (0.8p)(0.2 du) = 0.32p2 m2 >s

m W ina g

b)
ed e n
in
no W iss ea s

itt id tio
is e D t w
t p or em ch
d th g. in t la
The “lift” exerted on the cylinder can be determined from

an on in rs h
k g rn to rig
F = rUΓ = ( 1.20 kg>m3 ) (15 m>s) ( 0.32p2 m2 >s )
or in a uc y
w d le tr p

er ld
e lu nt ns co

= 56.8 kN>m Ans.


th inc de f i es

The pressure at a point on the surface is


of rk ( stu e o tat
ity o g us d S

2
1 2 Γ
p = p0 + rU c 1 - a - 2 sin u + b d
te is ss th ite

2 2pUa
in f th se for Un
gr w in e

We notice from this equation that p will be minimum at a point where u = -90°.
th t o a ly by

Then
y ar d le d
ro p an o te
st ny s d s ec

2
1 Γ
p = p0 + ra U 2 c 1 - a - 2 sin ( -90°) + b d
de f a rse de ot

2 2pUa
s
ill o u vi pr
w le co ro is

2
1 Γ
sa eir is p rk

= p0 + raU 2 c 1 - a2 + b d
th d wo

2 2pUa
an his

1 0.32p2 m2 >s 2
( 1.20 kg>m3 )( 15 m>s ) e 1 - c 2 +
2
T

= 400 Pa + d f
2 2p ( 15 m>s ) (0.2 m)
= - 99.3 Pa Ans.

Ans:
F = 5.68 kN>m
p = - 99.3 Pa

833
© 2014 Pearson Education, Inc., Upper Saddle River, NJ. All rights reserved. This material is protected under all copyright laws as they currently
exist. No portion of this material may be reproduced, in any form or by any means, without permission in writing from the publisher.

7–94.  Liquid is confined between a top plate having an area y


U
A and a fixed surface. A force F is applied to the plate and
gives the plate a velocity U. If this causes laminar flow, and F
the pressure does not vary, show that the Navier–Stokes and
h u
continuity equations indicate that the velocity distribution for
this flow is defined by u = U(y>h), and that the shear stress x
within the liquid is txy = F>A.

Solution
Since the flow is steady and is only along the x axis then v = w = 0. Also, the liquid
is incompressible. Thus, the continuity equation reduces to
0r 0(ru) 0(rv) 0(rw)

. We or
+ + + = 0

m W ina g
0t 0x 0y 0z

b)
ed e n
in
no W iss ea s

itt id tio
is e D t w
t p or em ch
d th g. in t la
0u
0 + r + 0 + 0 = 0

an on in rs h
0x

k g rn to rig
0u or in a uc y
w d le tr p

er ld
= 0
e lu nt ns co

0x
th inc de f i es

Integrating this equation with respect to x


of rk ( stu e o tat
ity o g us d S

u = u(y)
te is ss th ite

Using this result, when the pressure p remains constant along x axis, then Navier-Stokes
in f th se for Un

equation along x axis gives


gr w in e
th t o a ly by

0u 0u 0u 0u 0p 02u 02u 02u


y ar d le d

ra + u + v + w b = rgx - + ma 2 + 2 + 2 b
ro p an o te

0t 0x 0y 0z 0x 0x 0y 0z
st ny s d s ec
de f a rse de ot

02u
s
ill o u vi pr

0 + 0 + 0 + 0 = 0 - 0 + ma0 + + 0b
w le co ro is

0y2
sa eir is p rk
th d wo

02u
m = 0
an his

0y2
T

02u d 2u
Since u is a function of y only, = . Integrating this equation with respect to
0y2 dy2
y twice.
du
= C 1 (1)
dy

And,
u = C1y + C2 (2)
Applying the boundary condition, u = 0 at y = 0
0 = C1(0) + C2  C2 = 0

and u = U at y = h,
U
U = C1(h)  C1 =
h
Substituting these results into Eq. 1

U
u = a by (Q.E.D)
h

834
© 2014 Pearson Education, Inc., Upper Saddle River, NJ. All rights reserved. This material is protected under all copyright laws as they currently
exist. No portion of this material may be reproduced, in any form or by any means, without permission in writing from the publisher.

7–94. Continued

Applying

0u 0v
txy = ma + b
0y 0x

0v 0u U

. We or
Here, = 0 and from Eq. 1 = . Then

m W ina g
0x 0y h

b)
ed e n
in
no W iss ea s

itt id tio
is e D t w
t p or em ch
d th g. in t la
U U
txy = ma + 0b = ma b

an on in rs h
h h

k g rn to rig
or in a uc y
This shows that txy is a constant between the liquid layers. Therefore, its value is
w d le tr p

er ld
e lu nt ns co

equal to that at the bottom surface of the top plate which is


th inc de f i es

F
of rk ( stu e o tat

txy =  (Q.E.D)
ity o g us d S

A
te is ss th ite

Although not necessary, the Navier-Stokes equation along the y axis gives
in f th se for Un

0v 0v 0v 0v 0p 02v 02v 02v


gr w in e
th t o a ly by

ra + u + v + w b = rgy - + ma 2 + 2 + 2 b
0t 0x 0y 0z 0y 0x 0y 0z
y ar d le d
ro p an o te

0p
st ny s d s ec

r ( 0 + 0 + 0 + 0 ) = r( - g) - + m(0 + 0 + 0 + 0)
de f a rse de ot

0y
s
ill o u vi pr
w le co ro is

0r
= - rg
sa eir is p rk

0y
th d wo
an his

Integrating this equation with respect to y by realizing the pressure variation is


T

along y axis only,


p = - rgy + C3

Applying the boundary condition p = 0 at y = h,


0 = - rgh + C3  C3 = rgh

Then

p = rg(h - y)

835
© 2014 Pearson Education, Inc., Upper Saddle River, NJ. All rights reserved. This material is protected under all copyright laws as they currently
exist. No portion of this material may be reproduced, in any form or by any means, without permission in writing from the publisher.

7–95.  The channel for a liquid is formed by two fixed y


plates. If laminar flow occurs between the plates, show that
the Navier–Strokes and continuity equations reduce to
02u>0y2 = (1>m) 0p>0x and 0p>0y = 0. Integrate these d/2
x
equations to show that the velocity profile for the flow is
d/2
u = (1>(2m)) (dp>dx) 3 y2 - (d>2)2 4 . Neglect the effect of
gravity.

Solution
Since the flow is steady and is along the x axis only, then v = w = 0. Also, the liquid
is incompressible. Thus, the continuity equation becomes
0r 0(ru) 0(rv) 0(rw)

. We or
+ + + = 0

m W ina g
0t 0x 0y 0z

b)
ed e n
in
no W iss ea s

itt id tio
is e D t w
t p or em ch
d th g. in t la
0u
0 + r + 0 + 0 = 0

an on in rs h
0x
k g rn to rig
or in a uc y
w d le tr p

er ld
0u
e lu nt ns co

= 0
0x
th inc de f i es
of rk ( stu e o tat

Integrating this equation with respect to x,


ity o g us d S
te is ss th ite

u = u(y)
in f th se for Un
gr w in e

Using this result, the Navier-Stokes equation along the x and y axes gives
th t o a ly by
y ar d le d

0u 0u 0u 0u 0p 02u 02u 02u


ro p an o te

ra + u + v + w b = rgx - + ma 2 + 2 + 2 b
st ny s d s ec

0t 0x 0y 0z 0x 0x 0y 0z
de f a rse de ot

s
ill o u vi pr

0p 02u
r( 0 + 0 + 0 + 0 ) = 0 - + m( 0 + 2 + 0 )
w le co ro is

0x 0y
sa eir is p rk
th d wo

02u 1 0p
   (Q.E.D) (1)
an his

=
e

0y2 m 0x
T

0v 0v 0v 0v 0p 02v 02v 02v


ra + u + v + w b = rgy - + ma 2 + 2 + 2 b
0t 0x 0y 0z 0y 0x 0y 0z
0p
r(0 + 0 + 0 + 0) = 0 - + m(0 + 0 + 0)
0y
0p
= 0    (Q.E.D) (2)
0y

Integrating Eq. 2 with respect to y,

p = p(x)

02u d 2u
Since u is a function of y only and p is a function of x only, then 2 = and
0r dr 0y dy2
= . Eq. 1 becomes
0x dx

d 2u 1 dp
2
=
dy m dx

836
© 2014 Pearson Education, Inc., Upper Saddle River, NJ. All rights reserved. This material is protected under all copyright laws as they currently
exist. No portion of this material may be reproduced, in any form or by any means, without permission in writing from the publisher.

*7–95. Continued

Integrating this equation twice with respect to y,

du 1 dr
= a by + C1 (3)
dy m dx

1 dr 2
u = a by + C1y + C2 (4)
2m dx

du
Since u is maximum at y = 0. Then = 0 at y = 0. Using Eq. 3
dy

. We or
m W ina g

b)
ed e n
in
no W iss ea s
1 dr

itt id tio
is e D t w
t p or em ch
0 = a b(0) + C1  C1 = 0

d th g. in t la
m dx

an on in rs h
k g rn to rig
d or in a uc y
Also, u = 0 at y = . Using Eq. 4 with C1 = 0,
w d le tr p

er ld
2
e lu nt ns co
th inc de f i es

1 dr d 2 1 dr d 2
of rk ( stu e o tat

0 = a ba b + 0 + C2  C2 = - a ba b
2m dx 2 2m dx 2
ity o g us d S
te is ss th ite

Substituting these results into Eq. 4,


in f th se for Un
gr w in e

1 dr 2 1 dr d 2
th t o a ly by

u = a by - a ba b
2m dx 2m dx 2
y ar d le d
ro p an o te
st ny s d s ec
de f a rse de ot

1 dr d 2
s
ill o u vi pr

= a b c y2 - a b d (Q.E.D)
2m dx 2
w le co ro is
sa eir is p rk
th d wo
an his

e
T

837
© 2014 Pearson Education, Inc., Upper Saddle River, NJ. All rights reserved. This material is protected under all copyright laws as they currently
exist. No portion of this material may be reproduced, in any form or by any means, without permission in writing from the publisher.

*7–96.  Fluid having a density r and viscosity m fills the


space between the two cylinders. If the outer cylinder is
fixed, and the inner one is rotating at v, apply the v
Navier–Stokes equations to determine the velocity profile ri
assuming laminar flow.
ro r

Solution
Since the flow is steady and is along the transverse direction (u axis) only, then
vr = vz = 0. Also, the liquid is incompressible. Thus, the continuity equation
reduces to

. We or
m W ina g

b)
ed e n
in
no W iss ea s

itt id tio
is e D t w
t p or em ch
0r 1 0 ( rrvr ) 1 0 ( rvu ) 0 ( rvz )

d th g. in t la
+ + + = 0

an on in rs h
0t r 0r rr 0v0u 0z

k g rn to rig
u
0 + 0 + + 0 = 0
r 0u or in a uc y
w d le tr p

er ld
e lu nt ns co

0vu
th inc de f i es

= 0
0u
of rk ( stu e o tat
ity o g us d S

Integrating this equation with respect to u,


te is ss th ite
in f th se for Un

vu = vu(r)
gr w in e
th t o a ly by

Using this result, the Navier-Stokes equations along the u axis gives
y ar d le d
ro p an o te

0vu 0vu vu 0vu 0vu


st ny s d s ec

vrvu
ra + vr + + + vz b
de f a rse de ot

0t 0r r 0u r 0z
s
ill o u vi pr
w le co ro is

1 0p 1 0 0vu vu 1 02vu 2 0vr 02vu


sa eir is p rk

= - + rgu + mc ar b - 2 + 2 2 + 2 + d
th d wo

r 0u r 0r 0r r r 0u r 0u 0z2
an his

1 0 0vu vu
r(0 + 0 + 0 + 0 + 0) = - 0 + 0 + m c ar b - 2 + 0 + 0 + 0 d
T

r 0r 0r r
1 0 0vu vu
ar b - 2 = 0
r 0r 0r r
However, it can be shown that

0 1 0 1 0 0vu vu
c ( rv ) d = ar b - 2
0r r 0r u r 0r 0r r
Thus,

0 1 0
c ( rv ) d = 0
0r r 0r u

838
© 2014 Pearson Education, Inc., Upper Saddle River, NJ. All rights reserved. This material is protected under all copyright laws as they currently
exist. No portion of this material may be reproduced, in any form or by any means, without permission in writing from the publisher.

7–96. Continued

Since vu = vu(r), then the above equation can be written in the form of

d 1 d
c (rv ) d = 0
dr r dr u
Integrating this equation with respect to r,

1 d
( rv ) = C1
r dr u

. We or
d

m W ina g
( rv ) = C1r

b)
ed e n
in
no W iss ea s

itt id tio
dr u

is e D t w
t p or em ch
d th g. in t la
an on in rs h
Integrating again,

k g rn to rig
or in a uc y
r2
w d le tr p

er ld
rvu = C1a b + C2
e lu nt ns co

2
th inc de f i es
of rk ( stu e o tat
ity o g us d S

C1 C2
vu = r + (1)
te is ss th ite

2 r
in f th se for Un

At r = ru, vu = 0. Then Eq. 1 gives


gr w in e
th t o a ly by
y ar d le d

C1 C
(r ) + 2
ro p an o te

0 = (2)
2 0
st ny s d s ec

r0
de f a rse de ot

At r = ri, vu = vri. Then Eq. 1 gives


s
ill o u vi pr
w le co ro is

C1 C2
sa eir is p rk

vri = r + (3)
2 i ri
th d wo
an his

Solving Eq. 2 and 3,


T

2vr i2 vr i2r 02
C1 = -   C2 =
r 02 - ri 2
r 02 - r i2
Substituting these results into Eq. 1,
vr i2 vr i2r 02 1
vu = - a br + a ba b
r 02 - ri 2
r 02 - r i2 r
vr i2 r 02 - r 2
vu = a b Ans.
r 02 - r i 2 r

839
© 2014 Pearson Education, Inc., Upper Saddle River, NJ. All rights reserved. This material is protected under all copyright laws as they currently
exist. No portion of this material may be reproduced, in any form or by any means, without permission in writing from the publisher.

7–97.  A horizontal velocity field is defined by


u = 2 1 x2 - y2 2 ft>s and v = (- 4xy) ft>s. Show that these
expressions satisfy the continuity equation. Using the
Navier–Stokes equations, show that the pressure distribution
is defined by p = C - rV 2 >2 - rgz.

Solution
For the continuity equation,
0r 0(ru) 0(rv) 0(rw)
+ + +
0t 0x 0y 0z

= 0 + 4rx + ( - 4rx) + 0

= 0 (satisfied)

. We or
The Navier-Stokes equations along the x, y and z axes are

m W ina g

b)
ed e n
in
no W iss ea s

itt id tio
is e D t w
t p or em ch
0u 0u 0u 0u 0p 02u 02u 02u

d th g. in t la
ra + u + v + w b = rgx - + ma 2 + 2 + 2 b

an on in rs h
0t 0x 0y 0z 0x 0x 0y 0z

k g rn to rig
or in a uc y
0p
w d le tr p

er ld
r 3 0 + ( 2x2 - 2y2 )( 4x ) + ( - 4xy )( - 4y ) + 0 4 = 0 -
e lu nt ns co

+ m(4 - 4 + 0)
0x
th inc de f i es
of rk ( stu e o tat

0p
= - 8r ( x3 + xy2 )  (1)
ity o g us d S

0x
te is ss th ite
in f th se for Un

0v 0v 0v 0v 0p 02v 02v 02v


ra + u + v + w b = rgy - + ma 2 + 2 + 2b
gr w in e
th t o a ly by

0t 0x 0y 0z 0y 0x 0y 0z
y ar d le d
ro p an o te

0p
r 3 0 + ( 2x2 - 2y2 ) ( -4y) + ( - 4xy)( - 4x) + 0 4 = 0 -
st ny s d s ec

+ m(0 + 0 + 0)
0y
de f a rse de ot

s
ill o u vi pr

0r
w le co ro is

= - 8r ( y3 + x2y )  (2)
sa eir is p rk

0y
th d wo

0w 0w 0w 0w 0p 02w 02w 02w


an his

ra + u + v + w b = rgz - + ma 2 + 2 + 2 b
0t 0x 0y 0z 0z 0x 0y 0z
T

0p
r(0 + 0 + 0 + 0) = r( -g) - + m(0 + 0 + 0)
0z
0p
= - rg (3)
0z
Integrating Eqs. 1 with respect to x,

x4 x2y2
p = - 8r ° + ¢ + f(y) + g(z) (4)
4 2

Differentiate Eq. 4 with respect to y and equate to Eq. 2


0p
= -8r ( x2y ) + f 1(y) = - 8r ( y3 + x2y )
0y

f 1(y) = - 8ry3

840
© 2014 Pearson Education, Inc., Upper Saddle River, NJ. All rights reserved. This material is protected under all copyright laws as they currently
exist. No portion of this material may be reproduced, in any form or by any means, without permission in writing from the publisher.

7–97. Continued

Integrate this equation with respect to y,

f(y) = - 2ry4 + C1 (5)

Differentiate Eq. 4 with respect to z and equal to Eq. 3


0p
= g1(z) = - rg
0z

Integrate this equation with respect to z,

. We or
m W ina g

b)
g(z) = - rgz + C2 (6)

ed e n
in
no W iss ea s

itt id tio
is e D t w
t p or em ch
d th g. in t la
Substitute Eq. 5 and 6 into 4,

an on in rs h
k g rn to rig
x4 x2y2 or in a uc y
p = -8ra + b - 2ry4 - rgz + C
w d le tr p

er ld
4 2
e lu nt ns co
th inc de f i es

p = - 2r ( x4 + y4 + 2x2y2 ) - rgz + C
of rk ( stu e o tat
ity o g us d S

1 1 1
Since V 2 = ( u2 + v2 ) = 3 ( 2x2 - 2y2 ) 2 + ( -4xy)2 4 = 2 ( x4 + y4 + 2x2y2 ) ,
te is ss th ite

2 2 2
in f th se for Un

then the above equation becomes


gr w in e
th t o a ly by

1 2
y ar d le d

p = C - rV - rgz (Q.E.D)
ro p an o te

2
st ny s d s ec
de f a rse de ot

s
ill o u vi pr
w le co ro is
sa eir is p rk
th d wo
an his

e
T

841
© 2014 Pearson Education, Inc., Upper Saddle River, NJ. All rights reserved. This material is protected under all copyright laws as they currently
exist. No portion of this material may be reproduced, in any form or by any means, without permission in writing from the publisher.

7–98.  The sloped open channel has steady laminar flow at y


a depth h. Show that the Navier–Stokes equations reduce to
02u>0y2 = - (r g sin u)>m and 0p>0y = -r g cos u. Integrate h
these equations to show that the velocity profile is
u = [(r g sin u)>2m] 1 2hy - y2 2 and the shear-stress
distribution is txy = r g sin u (h - y).

u
x

Solution
Since the flow is steady and is along the x axis only, then v = w = 0. Also, the liquid
is incompressible. Thus, the continuity equation reduces to
0r 0(ru) 0(rv) 0(rw)
+ + + = 0
0t 0x 0y 0z

0u
0 + r + 0 + 0 = 0

. We or
0x

m W ina g

b)
ed e n
in
no W iss ea s

itt id tio
is e D t w
0u

t p or em ch
d th g. in t la
= 0
0x

an on in rs h
k g rn to rig
Integrating this equation with respect to x, or in a uc y
w d le tr p

er ld
e lu nt ns co

u = u(y)
th inc de f i es
of rk ( stu e o tat

The Navier-Stokes equations along the x and y axes give


ity o g us d S

0u 0u 0u 0u 0p 02u 02u 02u


te is ss th ite

ra + u + v + w b = rgx - + ma 2 + 2 + 2 b
in f th se for Un

0t 0x 0y 0z 0x 0x 0y 0z
gr w in e
th t o a ly by

02u
r(0 + 0 + 0 + 0) = rg sin u - 0 + m a0 + + 0b
y ar d le d
ro p an o te

0y2
st ny s d s ec
de f a rse de ot

02u rg sin u
s
ill o u vi pr

= - (Q.E.D) (1)
0y2 m
w le co ro is
sa eir is p rk

0v 0v 0v 0v 0r 02v 02v 02v


th d wo

ra + u + v + w b = rgy - + ma 2 + 2 + 2 b
0t 0x 0y 0z 0y 0x 0y 0z
an his

e
T

0p
r(0 + 0 + 0 + 0) = r( - g cos u) - + m(0 + 0 + 0)
0y
0p
= -rg cos u (Q.E.D) (2)
0y

02u 02u
Since u = u(y), then 2
= 2 . Thus Eq. (1) becomes
0y 0y
d 2u rg sin u
= -
dy2 m

Integrating this equation with respect to y twice yields


du rg sin u
= - y + C1 (3)
dy m
rg sin u 2
u = - y + C1y + C2 (4)
2m

842
© 2014 Pearson Education, Inc., Upper Saddle River, NJ. All rights reserved. This material is protected under all copyright laws as they currently
exist. No portion of this material may be reproduced, in any form or by any means, without permission in writing from the publisher.

7–98. Continued

At y = 0, u = 0. Then, Eq. 4 gives

0 = - 0 + 0 + C2  C2 = 0

du
At y = h, txy = m a b = 0. Then Eq. 3 gives
dy
rg sin u rgh sin u
0 = - (h) + C1  C1 =
m m

. We or
Substituting these results into Eq 3 and 4

m W ina g

b)
ed e n
in
no W iss ea s

itt id tio
is e D t w
t p or em ch
du rg sin u rgh sin u

d th g. in t la
= - y +
dy

an on in rs h
m m

k g rn to rig
du rg sin u or in a uc y
w d le tr p

er ld
= (h - y)
e lu nt ns co

dy m
th inc de f i es
of rk ( stu e o tat

rg sin u 2 rgh sin u


u = - y + y
ity o g us d S

2m m
te is ss th ite

rg sin u
in f th se for Un

u = ( 2hy - y2 ) (Q.E.D)
gr w in e

2m
th t o a ly by
y ar d le d
ro p an o te

The shear stress distribution is


st ny s d s ec
de f a rse de ot

du
s

= rg sin u(h - y) (Q.E.D)


ill o u vi pr

txy = m
dy
w le co ro is
sa eir is p rk
th d wo
an his

e
T

843
© 2014 Pearson Education, Inc., Upper Saddle River, NJ. All rights reserved. This material is protected under all copyright laws as they currently
exist. No portion of this material may be reproduced, in any form or by any means, without permission in writing from the publisher.

7–99.  The laminar flow of a fluid has velocity y


components u = 6x and v = -6y, where y is vertical.
Use the Navier–Stokes equations to determine the pressure
in the fluid, p = p(x, y), if at point (0, 0), p = 0. The density
of the fluid is r.
A B
x

2m 0.5 m

Solution
Since the flow is steady and the fluid is incompressible, the continuity equation is
0r 0(ru) 0(rv) 0(rw)
+ + + = 0 + 6r + ( - 6r) + 0 = 0
0t 0x 0y 0z

is indeed satisfied. Writing the Navier-Stokes equation along the x and y axes gives

0u 0u 0u 0u 0r 02u 02u 02u

. We or
ra + u + v + w b = rgx - + ma 2 + 2 + 2 b
0t 0x 0y 0z 0x

m W ina g
0x 0y 0z

b)
ed e n
in
no W iss ea s

itt id tio
is e D t w
t p or em ch
d th g. in t la
0r
r30 + 6x(6) + 0 + 04 = 0 - + 0

an on in rs h
0x

k g rn to rig
or in a uc y
w d le tr p
0r

er ld
e lu nt ns co

= - 36rx (1)
0x
th inc de f i es
of rk ( stu e o tat

0v 0v 0v 0v 0p 02v 02v 02v


ity o g us d S

ra + u + v + w b = rgy - + ma 2 + 2 + 2 b
0t 0x 0y 0z 0y 0x 0y 0z
te is ss th ite
in f th se for Un

0p
r30 + 0 + ( -6y)( - 6) + 04 = r( - g) -
gr w in e
th t o a ly by

0y
y ar d le d
ro p an o te

0p
st ny s d s ec

= -rg - 36ry (2)


0y
de f a rse de ot

s
ill o u vi pr

Integrating Eq. 1 with respect to x


w le co ro is
sa eir is p rk

r = - 18rx2 + f(y)
th d wo

(3)
an his

Takes the partial derivative of Eq. 2 with respect to y and equate it to Eq. 2
T

0p
= f 1(y) = - rg - 36ry
0y

Integrate this equation with respect to y,


f(y) = - rgy - 18ry2 + C

Substitute this result into Eq. 3

p = - 18rx2 - pgy - 18ry2 + C

p = - r ( 18x2 + 18y2 + gy ) + C (4)

At point (0, 0), p = 0. Then Eq. 4 gives

0 = r(0 + 0 + 0) + C   C = 0

Thus, the pressure distribution is

p(x, y) = - r ( 18x2 + 18y2 + gy ) Ans.

Ans:
p = - r (18x2 + 18y2 + gy )

844
© 2014 Pearson Education, Inc., Upper Saddle River, NJ. All rights reserved. This material is protected under all copyright laws as they currently
exist. No portion of this material may be reproduced, in any form or by any means, without permission in writing from the publisher.

*7–100.  The steady laminar flow of an ideal fluid y


towards  the fixed surface has a velocity of
u = 3 10 1 1 + 1> 1 8x3 2 4 m>s along the horizontal
streamline AB. Use the Navier–Stokes equations and
determine the variation of the pressure along this streamline,
and plot it for - 2.5 m … x … - 0.5 m. The pressure at A is A B
x
5 kPa, and the density of the fluid is r = 1000 kg>m3.

2m 0.5 m

Solution
We consider ideal fluid flow.
Since streamline AB is along the x axis, the velocity of the flow along this streamline
will not have components along the y and z axes; ie, v = w = 0. Also, the fluid is
ideal and the flow is steady. Writing the Navier-Stoke equation along x axis gives

0u 0u 0u 0u 0p 02u 02u 02u


ra + u + v + w b = rgx - + ma 2 + 2 + 2 b
0t 0x 0y 0z 0x 0x 0y 0z

. We or
m W ina g
0p 02u

b)
1 30

ed e n
in
no W iss ea s

itt id tio
rc 0 + 10a1 + ba - 4 b d = 0 - + 0 a 2 + 0 + 0b

is e D t w
t p or em ch
3 0x
8x 8x 0x

d th g. in t la
an on in rs h
0p 75r 75r 75r
k g rn to rig
1
= a1 + b = + or in a uc y
0x 2x 4
8x 3
2x 4
16x7
w d le tr p

er ld
e lu nt ns co

Integrating this equation with respect to x,


th inc de f i es
of rk ( stu e o tat

25r 25r
ity o g us d S

p(x) = - 3
- + C
2x 32x6
te is ss th ite
in f th se for Un

25r
gr w in e

p(x) = - ( 16x3 + 1 ) + C
th t o a ly by

32x6
y ar d le d
ro p an o te

At x = - 2.5 m, p = 5 kPa and with r = 1000 kg>m3. Then,


st ny s d s ec
de f a rse de ot

25
ill o u vi pr

5 = - 3 16 ( - 2.5 ) 3 + 14 + C
w le co ro is

32 ( - 2.5 ) 6
sa eir is p rk
th d wo

C = 4.2032r p (kPa)
an his

Thus, since r = 1000 kg>m3,


T

25
p(x) = c - ( 16x3 + 1 ) + 4.2032 d kPa Ans.
60
32x6
The plot of this pressure distribution is shown in Fig. a 50

x(m) - 2.5 - 2.0 - 1.5 - 1.0 - 0.5


40
p(kPa) 5.0 5.75 7.84 15.92 54.20
30

20

10

x(m)
–2.5 –2.0 –1.5 –1.0 –0.5

(a)

845

Вам также может понравиться